Practice Questions: Neuro (combined set)

Réussis tes devoirs et examens dès maintenant avec Quizwiz!

what nursing intervention is PRIORITY in the nursing diagnosis RISK FOR INEFFECTIVE AIRWAY CLEARANCE R/T INABILITY TO SWALLOW

***************PRIORITY************* PERFORMING ORAL SUCTIONING

after surgery what is the nurses PRIORITY assessment if the doctor has ordered a clear liquid diet?

**************PRIORITY*********** THE PATIENTS ABILITY TO SWALLOW ( GAG REFLEX)

what is the PRIORITY sign that may indicate Intracranial Pressure is increasing ?

**************PRIORITY************ SYSTOLIC PRESSURE INCREASES AND DIASTOLIC PRESSURE DECREASES

what PRIORITY should the nurse teach the caregiver of a patient with stroke who is taking the medication Coumadin?

**************PRIORITY************* SELECT ALL THAT APPLY DIETARY RESTRICTIONS MISSED DOSES BRUISING OR BLOOD IN URINE NEED FOR FREQUENT LABORATORY WORK

what is the PRIORITY nursing intervention in a patient with impaired skin integrity due to weakness and numbness in various parts of the body ?

**************PRIORITY************** change the patient's position every 2 hours

what is the nurses PRIORITY intervention after a craniotomy if the patient has periods of confusion?

*************PRIORITY******** REORIENT THE PATIENT TO PLACE AND SITUATION

what preoperative assessment is PRIORITY to document as a basis for postoperative comparison in a patient having a craniotomy?

*************PRIORITY********** MOTOR STRENGTH IN ALL EXTREMITIES

what is the PRIORITY in a patient with TIA is ready to transfer from bed to wheelchair?

*************PRIORITY************* MAKE SURE TO BRACE THE PARALYZED FOOT AND KNEE

after the nurse gathers all the information in a home health patient who is experiencing a TIA , what should she do that is most important ?

*************PRIORITY************* REFER THE PATIENT FOR IMMEDIATE MEDICAL EVALUATION

what is the PRIORITY INTERVENTION in a patient with multiple sclerosis when helping with ADL's ?

************PRIORITY************** LET THE PATIENT REST BETWEEN ACTIVITES

what is the PRIORITY EQUIPMENT when caring for a patient with Myasthenia Gravis?

***********PRIORITY********* A SUCTION MACHINE IN CASE OF COMPROMISED SWALLWOING

in assessing a patient with a cerebral aneurysm what is the order of PRIORITY?

**********PRIORITY **************** 1ST. URINE OUTPUT 2ND. MOTOR STRENGHT 3RD. VITAL SIGNS 4TH. LOC 5TH. SKIN INTEGRITY 6TH BOWEL ELIMINATION

Identify two types of hearing loss.

- Conductive (transmission of sound to inner ear is blocked) - Sensorineural (damage to eighth cranial nerve)

what is the nurses PRIORITY for controlling symptoms of patient with Myasthenia Gravis?

***************PRIORITY************* ADMINISTER MEDICATION AT THE PRECISE SCHEDULED TIME

When teaching patients who are at risk for Bell's palsy because of previous herpes simplex infection, which information should the nurse include?

"Call the doctor if pain or herpes lesions occur near the ear." rational: Pain or herpes lesions near the ear may indicate the onset of Bell's palsy and rapid corticosteroid treatment may reduce the duration of Bell's palsy symptoms. Antiviral therapy for herpes simplex does not reduce the risk for Bell's palsy. Corticosteroid therapy will be most effective in reducing symptoms if started before paralysis is complete but will still be somewhat effective when started later. Facial exercises do not prevent Bell's palsy.

Which statement by a patient who is being discharged from the emergency department (ED) after a head injury indicates a need for intervention by the nurse?

"I am going to drive home and go to bed." rational: Following a head injury, the patient should avoid operating heavy machinery. Retrograde amnesia is common after a concussion. The patient can take acetaminophen for headache and should return if symptoms of increased intracranial pressure such as dizziness or nausea occur.

When administering a mental status examination to a patient, the nurse suspects depression when the patient responds with ______________

"I don't know." rational: Answers such as "I don't know" are more typical of depression. The response "Who are those people over there?" is more typical of the distraction seen in a patient with delirium. The remaining two answers are more typical of a patient with dementia.

After teaching a patient about management of migraine headaches, the nurse determines that the teaching has been effective when the patient says, _____________

"I will try to lie down someplace dark and quiet when the headaches begin." rational: It is recommended that the patient with a migraine rest in a dark, quiet area. Topiramate (Topamax) is used to prevent migraines and must be taken for several months to determine effectiveness. Aspirin or other nonsteroidal anti-inflammatory medications can be taken with the triptans. Alcohol may precipitate migraine headaches.

A 28-year-old woman who has multiple sclerosis (MS) asks the nurse about risks associated with pregnancy. Which response by the nurse is accurate?

"MS symptoms may be worse after the pregnancy." rational: During the postpartum period, women with MS are at greater risk for exacerbation of symptoms. There is no increased risk for congenital defects in infants born of mothers with MS. Symptoms of MS may improve during pregnancy. Onset of labor is not affected by MS.

An elementary teacher who has just been diagnosed with epilepsy after having a generalized tonic-clonic seizure tells the nurse, "I cannot teach anymore, it will be too upsetting if I have a seizure at work." Which response by the nurse is best?

"Most patients with epilepsy are well controlled with antiseizure medications." rational: The nurse should inform the patient that most patients with seizure disorders are controlled with medication. The other information may be necessary if the patient seizures persist after treatment with antiseizure medications is implemented.

The health care provider recommends a carotid endarterectomy for a patient with carotid atherosclerosis and a history of transient ischemic attacks (TIAs). The patient asks the nurse to describe the procedure. Which response by the nurse is appropriate?

"The carotid endarterectomy involves surgical removal of plaque from an artery in the neck." rational: In a carotid endarterectomy, the carotid artery is incised and the plaque is removed. The response beginning, "The diseased portion of the artery in the brain is removed" describes an arterial graft procedure. The answer beginning, "A catheter with a deflated balloon is positioned at the narrow area" describes an angioplasty. The final response beginning, "A wire is threaded through the artery" describes the MERCI procedure.

When family members ask the nurse about the purpose of the ventriculostomy system being used for intracranial pressure monitoring for a patient, which response by the nurse is best?

"The monitoring system helps show whether blood flow to the brain is adequate." rational: Short and simple explanations should be given to patients and family members. The other explanations are either too complicated to be easily understood or may increase the family member's anxiety.

A 62-year-old patient is brought to the clinic by a family member who is concerned about the patient's inability to solve common problems. To obtain information about the patient's current mental status, which question should the nurse ask the patient?

"What did you have for breakfast?" rational: This question tests the patient's recent memory, which is decreased early in Alzheimer's disease (AD) or dementia. Asking the patient about birthplace tests for remote memory, which is intact in the early stages. Questions about the patient's emotions and self-image are helpful in assessing emotional status, but they are not as helpful in assessing mental state.

what should the nurse assess for first PRIORITY in a patient with head injury ?

******** PUPILLARY RESPONSES*******

what is the PRIORITY nursing intervention in a patient with Alzheimer's disease?

********* PRIORITY ************* ATTACH AN IDENTITY TAG TO THE PATIENTS CLOTHES

what is PRIORITY for the nurse for bowel retraining in a patient with spinal cord injury ?

********** ENCOURAGING PATIENT TO CONSUME HIGH FIBER DIET********

what nursing intervention is PRIORITY in a patient with hemianopia?

********************PRIORITY************** APPROACH THE PATIENT FROM THE UNEFFECTED SIDE

if patient has a seizure after EEG what is the nurses priority?

****** PRIORITY*****PLACING PATIENT ON SIDE*****

after administering MANNITOL to a patient with head injury what is PRIORITY in reporting the status of patient to the physician

******* URINE OUTPUT*****

what is the nurses PRIORITY when caring for a patient who is deceased?

**********PRIORITY********** to allow the family see the patient this is important PLACE A ROLLED TOWEL UNDER THE CHIN UNTIL THE FAMILY ARRIVES

after doing a postoperative assessment on a patient who had surgery 6 hours ago the nurses notes that the dressing is moist, what is the nurses PRIORITY ?

**********PRIORITY*********** REINFORCE THE DRESSING

what postoperative PRIORITY should the nurse take in a patient who had a diskectomy with spinal fusion ?

**********PRIORITY************ LOGROLL THE PATIENT FROM SIDE TO SIDE

to reduce the risk of liability what is the nurses PRIORITY in a patient with head injury ?

********PRIORITY***** REMAIN WITH THE ACCIDNET VICTIM UNTIL PARAMEDICS ARRIVE

what is the nurses PRIORITY when caring for a patient in a Myasthenia Gravis crisis ?

********PRIORITY************ BREATHING

what nursing action is the PRIORITY in a patient with a head injury ?

*******PRIORITY***** MAINTAIN THE PATIENTS PRESENT POSITION

PRIORITY nursing intervention in the postictal phase of a seizure?

******ASSESS THE PATIENT'S BREATHING PATTERN*******

what is the PRIORITY TEACHING PLAN in a patient with Parkinson's disease?

******PRIORITY******* THE NEED TO REMOVE ALL SAFETY HAZARDS

what is the highest PRIORITY in a patient with T5 injury ?

******STRENGTHENING THE CLIENT'S UPPER BODY MUSCLES*****

when prepping a patient for EEG what is most important?

*****PRIORITY*** ASSIST WITH SHAMPOOING THE CLIENT'S HAIR

what is the nursing priority if she finds a patient having a tonic -clonic seizure?

*****PRIORITY**** ROLLING THE PATIENTS BODY TO THE SIDE REMOVING ENVIRONMENTAL HAZARDS TO PROTECT THE PATIENT CALLING THE RESPIRATORY THERAPY DEPARTMENT

what should the nurses priority document for a patient with seizure?

****PRIORITY*** THE DURATION OF THE SEIZURE other things to include: time the seizure started duration of the seizure client's mood just before the seizure client's comments after the seizure

what discharge instruction is PRIORITY after having a PET scan?

****PRIORITY********** INCREASE FLUID INTAKE

what nursing action is important for seizure precautions?

***PRIORITY*** KEEP BED IN LOWEST POSITION****

HESI HINT: A woman who has had a stroke 2 days earlier has left-sided paralysis. She has begun to regain some movement in her left side. What can the nurse tell the family about the client's recovery period?

- "The quicker movement is recovered, the better the prognosis is for full or improved recovery." - She will need patience and understanding from her family as she tries to cope with the stroke - Mood swings can be expected during the recovery period and bouts of depression and tearfulness are likely

HESI HINT: What is drug therapy for MS clients?

- ACTH - Cortisone - Cyclophosphamide (Cytoxan) - Other immunosuppressive drugs - Use Interferon-beta products (Betaseron, Rebif, and Avonex) when MS relapses *Focus on the prevention of infection with these drugs

HESI HINT: What is a residual feeding? What does it indicate?

- Amount of previous feeding still in the stomach - Presence of 100 mL of residual in an adult usually indicates poor gastric emptying and the feeding should be withheld

In your own words, describe the Glasgow Coma scale

- An objective assessment of the level of consciousness based on a score of 3-15, with scores of 7 or less indicative of coma

HESI HINT: What heart rhythm causes a high incidence of thrombus formation?

- Atrial flutter and fibrillation - They cause a turbulence of blood flow through all valves and heart chambers

HESI HINT: Why should you try to avoid restraints? Why should you avoid narcotics?

- Avoid restraints bc they increase restlessness - Avoid narcotics bc they mask level of responsiveness

HESI HINT: What often relieves symptoms in its with myasthenia gravis? What is often a recurring problem?

- Bed rest often relieves symptoms - Bladder and respiratory infections are often a recurring problem There is a need for health promotion teachings

HESI HINT: What are benign brain tumors and how do they affect the brain?

- Benign tumors continue to grow and take up space in the confined area of the cranium, causing neural and vascular compromise in the brain, increased ICP, and necrosis of brain tissue - Even benign tumors must be treated bc they may have malignant effects

A neighbor calls the neighborhood nurse stating that he was knocked hard to the floor by his very hyperactive dog. He is wondering what sxs would indicate the need to visit an ED. What should the nurse tell him to do?

- Call his HCP now and inform him or her of the fall. Sxs that need medical attention: - Vertigo or confusion - Any subtle behavioral change - Headache - Vomiting - Ataxia (Imbalance) - Seizure

HESI HINT: What can restlessness indicate?

- Can indicate a return to consciousness - Can indicate anoxia, distended bladder, covert bleeding, or increasing cerebral anoxia *Do not overstate and report any sxs of restlessness

Write four nursing interventions for the care of the blind person and four nursing interventions for the care of the deaf person.

- Care of blind: Announce presence clearly, call by name, orient carefully to surroundings, guide by walking in front of client with his or her hand in your elbow - Care of deaf: Reduce distraction before beginning conversation, look and listen to client, give client full attention if he or she is a lip reader, face client

What activities and situations that increase ICP should be avoided?

- Change in bed position - Extreme hip flexion - Endotracheal suctioning - Compression of jugular veins - Coughing, vomiting, and straining of any kind

HESI HINT: What is a cataract?

- Condition characterized by the opacity of the lens - Leading cause of blindness in the world

HESI HINT: What are common craniotomy prep medications?

- Corticosteroids reduce swelling - Agents and osmotic diuretics reduce secretions (atropine, glycopyrrolate/robinul) - Agents to reduce seizures (phenytoin) - Prophylactic ABX

HESI HINT: What are the three main parts of the ear? What are their functions?

- External ear - Middle ear - Inner ear Inner ear: Involved in CNS disorders; not helped with hearing aids External and middle ear: Issues may result from infection, trauma, or wax buildup; can be treated with hearing aid

HESI HINT: What medication is used to treat glaucoma?

- Eye drops are used to cause pupil constriction bc movement of the muscles to constrict the pupil also allows aqueouss humor to flow out - This decreases eye pressure - Pilocarpine is also used - Caution client that vision may be blurred 1-2 hours after administration (d/t pupillary constriction)

HESI HINT: What does CSF leakage carry the risk of?

- It carries the risk for meningitis and indicates a deteriorating condition - Because of CSF leakage, the usual sxs of increased ICP may not occur

HESI HINT: What are nursing considerations for any client on bed rest or immobilized?

- Must have ROM exercises often and very frequent position changes - Do not leave the client in any one position for longer than 2 hours - Any position that dec. venous return, like sitting with dependent extremities for long periods, is dangerous

HESI HINT: What medical condition is common in comatose clients? How is this treated?

- Paralytic ileus - Gastric tube aids in gastric decompression

What are the classifications of the commonly prescribed eye drops for the glaucoma?

- Parasympathetic for pupillary constriction - Beta-adrenergic receptor blocking agents to inhibit formation of aqueous humor - Carbonic anhydrase inhibitors to reduce aqueous humor production - Prostaglandin agonists to increase aqueous humor outflow

Who is at risk for stroke?

- Persons with histories of HTN - Previous TIAs - Cardiac disease (atrial flutter or fibrillation) - Diabetes - Oral contraceptive use - Older adults

HESI HINT: What should physical assessment concentrate on? What clients is this important for?

- Physical assessment should concentrate on resp. status, esp. in clients with injury at C3-C5. The cervical plexus (c3-c5) innervates diaphragm, which helps with breathing

HESI HINT: Other Meds for Glaucoma?

- Pilocarpine - Beta-Adrenergic Receptor Blocking Agents ("lol"s) - Carbonic Anhydrase Inhibitors (-"lamide"s) - Alpha Agonists (-"dine"s) - Prostaglandin Antagonists (-"prost"s)

State four independent nursing interventions to maintain adequate respiration, airway, and oxygenation in the unconscious client.

- Position for maximum ventilation (prone or semiprone or slightly to one side) - Insert airway if tongue is obstructing, suction airway efficiently, monitor arterial pO2 and pCO2 - Hyperventilate with 100% O2 before suctioning

HESI HINT: What are safety features for immobilized clients?

- Prevent skin breakdown by frequent turning - Maintain adequate nutrition - Prevent aspiration with slow, small feedings or NG feedings - Monitor neurologic sxs to detect the first sxs that ICP may be increasing - Provide ROM exercises to prevent deformities - Prevent respiratory complications; frequent turning and positioning provide optimal drainage

HESI HINT: What are ways to communicate with older adults who are hearing impaired?

- Speak in a low-pitched voice, slowly and distinctly - Stand in front of the person, with the light source behind the client - Use visual aids if available

HESI HINT: Nursing interventions for when temperature elevates?

- Take quick measures to decrease it bc fever increases cerebral metabolism and can increase cerebral edema

HESI HINT: Where do sxs involving motor function usually begin?

- They begin in the upper extremities with weakness progressing to spastic paralysis - Bowel and bladder dysfunction occurs in 90% of cases - MS is more common in women - Progression is not "orderly"

HESI HINT: Why are steroids administered after a stroke?

- They decrease cerebral edema and retard permanent disability - H2 inhibitors are administered to prevent peptic ulcers

HESI HINT: What are common features of Parkinson disease? What is a common nursing focus for this patient?

- Tremors (coarse between finger and thumb - pill rolling) - Rigidity - Hypertonicity - Masklike facial expression - Stooped posture Focus on safety!

The spouse of a male patient with early stage Alzheimer's disease (AD) tells the nurse, "I am just exhausted from the constant worry. I don't know what to do." Which action is best for the nurse to take next:

-Offer ideas for ways to distract or redirect the patient. -Educate the spouse about the availability of adult day care as a respite. -Ask the spouse what she knows and has considered about dementia care options. rational: The stress of being a caregiver can be managed with a multicomponent approach. This includes respite care, learning ways to manage challenging behaviors, and further assessment of what the spouse may already have considered. The patient is in the early stages and does not need long-term placement. Antianxiety medications may be appropriate but other measures should be tried first.

When preparing to admit a patient who has been treated for status epilepticus in the emergency department, which equipment should the nurse have available in the room:

-Siderail pads -Oxygen mask -Suction tubing rational: The patient is at risk for further seizures, and oxygen and suctioning may be needed after any seizures to clear the airway and maximize oxygenation. The bed's side rails should be padded to minimize the risk for patient injury during a seizure. Insertion of a nasogastric (NG) tube is not indicated because the airway problem is not caused by vomiting or abdominal distention. Use of tongue blades during a seizure is contraindicated.

When caring for a patient who experienced a T1 spinal cord transsection 2 days ago, which collaborative and nursing actions will the nurse include in the plan of care:

-Urinary catheter care -Continuous cardiac monitoring -Avoidance of cool room temperature -Administration of H2 receptor blockers rational: The patient is at risk for bradycardia and poikilothermia caused by sympathetic nervous system dysfunction and should have continuous cardiac monitoring and maintenance of a relatively warm room temperature. Gastrointestinal (GI) motility is decreased initially and NG suctioning is indicated. To avoid bladder distention, a urinary retention catheter is used during this acute phase. Stress ulcers are a common complication but can be avoided through the use of the H2 receptor blockers such as famotidine.

A patient with Parkinson's disease is admitted to the hospital for treatment of an acute infection. Which nursing interventions will be included in the plan of care:

-Use an elevated toilet seat -Cut patient's food into small pieces -Place an arm chair at the patient's bedside rational: Since the patient with Parkinson's has difficulty chewing, food should be cut into small pieces. An armchair should be used when the patient is seated so that the patient can use the arms to assist with getting up from the chair. An elevated toilet seat will facilitate getting on and off the toilet. High protein foods will decrease the effectiveness of L-dopa. Parkinson's is a steadily progressive disease without acute exacerbations.

A patient with Parkinson's disease has lost 35 pounds over two months. A swallowing study shows that the patient is able to swallow and does not aspirate. What suggestion should the nurse discuss with the patient and spouse to improve nutrition? 1 Allow adequate time for the patient to eat meals. 2 Administer prescribed carbidopa/levodopa with a protein drink. 3 Encourage the patient to eat at least every two hours while awake. 4 Include foods that are chewy so the patient builds up the jaw muscles.

1 Allow adequate time for the patient to eat meals. Allowing adequate time for the patient to eat will limit frustration and improve overall intake. Six small feedings may improve intake, but eating every two hours would exhaust the patient. Absorption of levodopa is impaired by protein, so it is best to avoid large amounts of protein when carbidopa/levodopa is administered. Foods should be easily chewable to increase the overall intake.

A patient with a family history of Huntington's disease suffers from a psychiatric disorder and is treated with sertraline and paroxetine. Tetrabenazine is added to the patient's prescription. Which assessment finding supports the addition of tetrabenazine to the prescription? 1 Chorea 2 Anxiety 3 Depression 4 Social withdrawal

1 Chorea A patient with a family history of Huntington's disease has a 50 percent chance of inheriting the disease. The early stages of the disease are characterized by psychiatric symptoms. Movement disorders, such as chorea, characterized by abnormal and excessive involuntary movements of the face, the limbs, and the body, may appear at later stages of the disease. These are treated successfully with tetrabenazine. Anxiety can be managed with sertraline and paroxetine. Depression is a psychiatric symptom and can also be managed with sertraline and paroxetine. Social withdrawal is a psychiatric disorder symptom managed with sertraline and paroxetine,

A patient being treated for multiple sclerosis complains of flu-like symptoms and reports feeling depressed. The patient also reports having frequent thoughts of committing suicide. Which drug in the patient's prescription may be responsible for these symptoms? 1 β-interferon 2 Natalizumab 3 Mitoxantrone 4 Dalfampridine

1 β-interferon β-interferon is an immunomodulator prescribed in the initial treatment of multiple sclerosis. It is known to cause flu-like symptoms, depression, and suicidal ideations. Natalizumab is prescribed for patients with active and aggressive forms of multiple sclerosis. This drug is not part of initial treatment, and the major risk factor associated with this drug therapy is progressive multifocal leukoencephalopathy. Mitoxantrone is also prescribed for patients with active and aggressive forms of multiple sclerosis, not in the initial treatment. The risk factors associated with this drug are cardiotoxicity, leukemia, and infertility. Dalfampridine is prescribed to improve walking speed.

A patient is advised to use diazepam for multiple sclerosis. What patient teaching is important for those taking this drug? Select all that apply. 1 Avoid driving while on the drug. 2 Do not stop the drug abruptly. 3 Avoid taking alcohol with the drug. 4 Monitor blood pressure regularly. 5 Avoid contact with large crowds.

1 Avoid driving while on the drug. 2 Do not stop the drug abruptly. 3 Avoid taking alcohol with the drug. Diazepam is used as a muscle relaxant. Patients on diazepam should avoid driving and any such activities requiring the patient to be alert because of the sedative effects of the drug. They should not stop the drug abruptly without consulting the health care provider. The drug should not be taken along with alcohol because alcohol can potentiate the drug's effect. Monitoring blood pressure is a general health care measure and is not specific to diazepam. Because the drug has no effect on the immune system, avoiding crowds is not necessary.

The nurse is caring for a patient with a spastic bladder secondary to multiple sclerosis. What manifestations of spastic bladder should the nurse expect to find in the patient? Select all that apply. 1 Bladder contractions are unchecked. 2 The patient experiences urinary retention. 3 The bladder has a small capacity for urine. 4 The patient experiences incontinence and dribbling. 5 There is no sensation of urge to void or no desire to void.

1 Bladder contractions are unchecked. 3 The bladder has a small capacity for urine. 4 The patient experiences incontinence and dribbling. A patient with spastic bladder may experience unchecked bladder contraction, and the bladder may have a small capacity for urine. Both of these factors may result in urine urgency and frequency and cause incontinence and dribbling. Sensation of an urge to void is present in patients with spastic bladder but absent in patients with flaccid bladder, who may have urinary retention due to a large bladder capacity for urine.

what nursing intervention should be done when a patient with multiple sclerosis develops the side effect of nausea and vomiting from taking interferon beta-1a?

provide small easy to digest meals

The nurse is caring for a patient with Parkinson's disease. What adjustments should the nurse make in the dietary habits of the patient to prevent malnutrition and constipation? Select all that apply. 1 Cut food into bite-size pieces. 2 Serve hot foods on a warmed plate. 3 Include whole grains and fruits in the diet. 4 Include plenty of food items high in protein. 5 Provide three large meals rather than six small meals.

1 Cut food into bite-size pieces. 2 Serve hot foods on a warmed plate. 3 Include whole grains and fruits in the diet. Patients with Parkinson's disease are predisposed to malnutrition and constipation, owing to inadequate food intake caused by difficulty in chewing and swallowing. To promote adequate nutrition, the nurse should include whole grains and fruits in the diet, which will prevent constipation. The food should be cut into bite size pieces so that chewing and swallowing are easy. Serving hot foods on a warmed plate makes the food more appealing. Food items high in protein should be limited in the diet because they can interfere with the absorption of carbidopa-levodopa, the most common drug used in the treatment of Parkinson's disease. Six small meals, rather than three large meals, would be less exhausting for the patients.

A patient with myasthenia gravis is admitted to the hospital in respiratory failure and diagnosed with myasthenic crisis. Which of the factors in the patient's current life situation led to the development of this myasthenic crisis? Select all that apply. 1 Omitted pyridostigmine previous day. 2 Attended funeral of family member earlier in week. 3 Took antibiotic prescribed for urinary tract infection. 4 Diagnosed with urinary tract infection previous week. 5 Continued taking prescribed corticosteroid every other day.

1 Omitted pyridostigmine previous day. 2 Attended funeral of family member earlier in week. 3 Took antibiotic prescribed for urinary tract infection. 4 Diagnosed with urinary tract infection previous week. Myasthenia gravis is an autoimmune disease in which acetylcholine receptors at the neuromuscular junction are destroyed. Patients experience muscular weakness that improves with rest. A myasthenia crisis is an acute exacerbation of muscle weakness that often involves the respiratory muscles. Exacerbations and crises can be precipitated by many factors including infection (patient's urinary tract infection), emotional stress (funeral of family member), reaction to medications (antibiotic prescribed for urinary tract infection), and inadequate anticholinesterase medications. Pyridostigmine is an anticholinesterase medication that prolongs acetylcholine present in the neuromuscular junction to improve muscle strength. Taking corticosteroid medications as prescribed suppresses immunity to decrease the myasthenic effect of muscular weakness.

The nurse is planning discharge teaching for a patient with myasthenia gravis. What instructions should the nurse include in the plan? Select all that apply. 1 Plan activities with periods of rest. 2 Practice hobbies such as playing golf. 3 Include liquid rather than solid foods in the diet. 4 Eat a balanced diet that can be easily chewed and swallowed. 5 Schedule drugs so that the peak effect of the drug is at mealtime.

1 Plan activities with periods of rest. 4 Eat a balanced diet that can be easily chewed and swallowed. 5 Schedule drugs so that the peak effect of the drug is at mealtime. The discharge teaching should focus on the neurologic deficits and their effect on daily living. Teach the patient about a balanced diet that can easily be chewed and swallowed. Help the patient plan activities of daily living to avoid fatigue. Scheduling doses of drugs so that peak action is reached at mealtime may make eating less difficult. Semisolid foods may be easier to eat than solids or liquids. Arrange diversional activities that require little physical effort and match the patient's interests. Playing golf may be too exhausting.

A patient with a family history of Huntington's disease (HD) who underwent genetic testing has positive results. What can the nurse infer from this finding? Select all that apply. 1 There is no cure for this disease. 2 The patient would not be treated completely with the help of drugs. 3 The patient will develop a progressive, degenerative brain disorder. 4 The onset of disease usually occurs between 30 and 50 years of age. 5 The patient will be a carrier of the mutated gene but will not develop the disease.

1 There is no cure for this disease. 3 The patient will develop a progressive, degenerative brain disorder. 4 The onset of disease usually occurs between 30 and 50 years of age. Huntington's disease is an autosomal dominant disorder caused by a mutation in the HTT gene located on chromosome 4. Genetic testing, or DNA testing, is useful in diagnosing the disease but is not helpful in predicting the onset of symptoms. There is no cure for the disease. The onset of disease usually occurs between 30 and 50 years of age. Huntington's disease is a progressive, degenerative brain disorder. The symptoms of behavioral problems and movement disorder can be controlled with drugs. Negative test results would indicate that the person does not carry the mutated gene and will not develop the disease.

how far apart do you space the injections for the medication interferon beta-1a ( Avenox) when alternating sites subcutaneously ?

1 inch

A 58-year-old patient who began experiencing right-sided arm and leg weakness is admitted to the emergency department. In which order will the nurse implement these actions included in the stroke protocol?

1) Administer oxygen to keep O2 saturation >95% 2) Use National Institute of Health Stroke Scale to assess patient 3) Obtain CT scan without contrast. 4) Infuse tissue plasminogen activator (tPA). rational: The initial actions should be those that help with airway, breathing, and circulation. Baseline neurologic assessments should be done next. A CT scan will be needed to rule out hemorrhagic stroke before tPA can be administered.

In which order will the nurse perform the following actions when caring for a patient with possible C6 spinal cord trauma who is admitted to the emergency department?

1) Immobilize the patient's head, neck, and spine. 2) Administer O2 using a non-rebreather mask. 3) Monitor cardiac rhythm and blood pressure. 4) Infuse normal saline at 150 mL/hr. 5) Transfer the patient to radiology for spinal computed tomography (CT). rational: The first action should be to prevent further injury by stabilizing the patient's spinal cord. Maintenance of oxygenation by administration of 100% O2 is the second priority. Because neurogenic shock is a possible complication, monitoring of heart rhythm and BP are indicated, followed by infusing normal saline for volume replacement. A CT scan to determine the extent and level of injury is needed once initial assessment and stabilization are accomplished.

10. Which type of seizure is most likely to cause death for the patient? a. Subclinical seizures b. Myoclonic seizures c. Psychogenic seizures d. Tonic-clonic status epilepticus

10. d. Tonic-clonic status epilepticus is most dangerous because the continuous seizing can cause respiratory insufficiency, hypoxemia, cardiac dysrhythmia, hyperthermia, and systemic acidosis, which can all be fatal. Subclinical seizures may occur in a patient who is sedated, so there is no physical movement. Myoclonic seizures may occur in clusters and have a sudden, excessive jerk of the body that may hurl the person to the ground. Psychogenic seizures are psychiatric in origin and diagnosed with videoelectroencephalography (EEG) monitoring. They occur in patients with a history of emotional abuse or a specific traumatic episode.

A patient with a head injury opens the eyes to verbal stimulation, curses when stimulated, and does not respond to a verbal command to move but attempts to remove a painful stimulus. The nurse records the patient's Glasgow Coma Scale score as _______

11 rational: The patient has a score of 3 for eye opening, 3 for best verbal response, and 5 for best motor response.

What types of drugs are used in tx of myasthenia gravis?

Anti cholinesterase drugs They inhibit action of cholinesterase at nerve endings to promote accumulation of acetylcholine at receptor sites This should improve neuronal transmission to muscles

11. A patient admitted to the hospital following a generalized tonic-clonic seizure asks the nurse what caused the seizure. What is the best response by the nurse? a. "So many factors can cause epilepsy that it is impossible to say what caused your seizure." b. "Epilepsy is an inherited disorder. Does anyone else in your family have a seizure disorder?" c. "In seizures, some type of trigger causes sudden, abnormal bursts of electrical brain activity." d. "Scar tissue in the brain alters the chemical balance, creating uncontrolled electrical discharges."

11. c. A seizure is a paroxysmal, uncontrolled discharge of neurons in the brain, which interrupts normal function, but the factor that causes the abnormal firing is not clear. Seizures may be precipitated by many factors and although scar tissue may make the brain neurons more likely to fire, it is not the usual cause of seizures. Epilepsy is established only by a pattern of spontaneous, recurring seizures.

what assessment finding indicates if the client has developed ineffective breathing in a patient with Guillain-Barre syndrome?

pulse oximetry of 82%

12. A patient with a seizure disorder is being evaluated for surgical treatment of the seizures. The nurse recognizes that what is one of the requirements for surgical treatment? a. Identification of scar tissue that is able to be removed b. An adequate trial of drug therapy that had unsatisfactory results c. Development of toxic syndromes from long-term use of antiseizure drugs d. The presence of symptoms of cerebral degeneration from repeated seizures

12. b. Most patients with seizure disorders maintain seizure control with medications but if surgery is considered, three requirements must be met: the diagnosis of epilepsy must be confirmed, there must have been an adequate trial with drug therapy without satisfactory results, and the electroclinical syndrome must be defined. The focal point must be localized but the presence of scar tissue is not required.

13. The nurse teaches the patient taking antiseizure drugs that this method is most commonly used to measure compliance and monitor for toxicity. a. A daily seizure log b. Urine testing for drug levels c. Blood testing for drug levels d. Monthly electroencephalography (EEG)

13. c. Serum levels of antiseizure drugs are monitored regularly to maintain therapeutic levels of the drug, above which patients are likely to experience toxic effects and below which seizures are likely to occur. Many newer drugs do not require drug level monitoring because of large therapeutic ranges. A daily seizure log and urine testing for drug levels will not measure compliance or monitor for toxicity. EEGs have limited value in diagnosis of seizures and even less value in monitoring seizure control.

14. Priority Decision: When teaching a patient with a seizure disorder about the medication regimen, what is it most important for the nurse to emphasize? a. The patient should increase the dosage of the medication if stress is increased. b. Most over-the-counter and prescription drugs are safe to take with antiseizure drugs. c. Stopping the medication abruptly may increase the intensity and frequency of seizures. d. If gingival hypertrophy occurs, the drug should be stopped and the health care provider notified.

14. c. If antiseizure drugs are discontinued abruptly, seizures can be precipitated. Missed doses should be made up if the omission is remembered within 24 hours and patients should not adjust medications without professional guidance because this also can increase seizure frequency and may cause status epilepticus. Antiseizure drugs have numerous interactions with other drugs and the use of other medications should be evaluated by health professionals. If side effects occur, the physician should be notified and drug regimens evaluated.

HESI HINT: What should you be alert for with patients that have myasthenia gravis?

Be alert for changes in resp. status -> Most severe involvement may result in respiratory failure

HESI HINT: Why is a client with an altered state of consciousness fed via enteral routes?

Because the likelihood of aspiration is high with oral feedings

Following a generalized tonic-clonic seizure, the patient is tired and sleepy. What care should the nurse provide? a. Suction the patient before allowing him to rest. b. Allow the patient to sleep as long as he feels sleepy. c. Stimulate the patient to increase his level of consciousness. d. Check the patient's level of consciousness every 15 minutes for an hour.

16. b. In the postictal phase of generalized tonic-clonic seizures, patients are usually very tired and may sleep for several hours and the nurse should allow the patient to sleep as long as necessary. Suctioning is performed only if needed and decreased level of consciousness is not a problem postictally unless a head injury has occurred during the seizure.

17. During the diagnosis and long-term management of a seizure disorder, what should the nurse recognize as one of the major needs of the patient? a. Managing the complicated drug regimen of seizure control b. Coping with the effects of negative social attitudes toward epilepsy c. Adjusting to the very restricted lifestyle required by a diagnosis of epilepsy d. Learning to minimize the effect of the condition in order to obtain employment

17. b. One of the most common complications of a seizure disorder is the effect it has on the patient's lifestyle. This is because of the social stigma attached to seizures, which causes patients to hide their diagnosis and to prefer not to be identified as having epilepsy. Medication regimens usually require only once- or twice-daily dosing and the major restrictions of lifestyle usually involve driving and high-risk environments. Job discrimination against the handicapped is prevented by federal and state laws and patients only need to identify their disease in case of medical emergencies.

18. A patient at the clinic for a routine health examination mentions that she is exhausted because her legs bother her so much at night that she cannot sleep. The nurse questions the patient further about her leg symptoms with what knowledge about restless legs syndrome? a. The condition can be readily diagnosed with EMG. b. Other more serious nervous system dysfunctions may be present. c. Dopaminergic agents are often effective in managing the symptoms. d. Symptoms can be controlled by vigorous exercise of the legs during the day.

18. c. Restless legs syndrome that is not related to other pathologic processes, such as diabetes mellitus or rheumatic disorders, may be caused by a dysfunction in the basal ganglia circuits that use the neurotransmitter dopamine, which controls movements. Dopamine precursors and dopamine agonists, such as those used for parkinsonism, are effective in managing sensory and motor symptoms. Polysomnography studies during sleep are the only tests that have diagnostic value and although exercise should be encouraged, excessive leg exercise does not have an effect on the symptoms.

Complications of immobility include the potential for thrombus development. State three nursing interventions to prevent thrombi.

Frequent range of motion exercises Frequent (every 2 hours) position changes Avoidance of positions that dec. venous return

what should the nurse encourage in the diet of a patient with Parkinson's due to risk for constipation ?

Fresh fruits

19. Which chronic neurologic disorder involves a deficiency of the neurotransmitters acetylcholine and γ aminobutyric acid (GABA) in the basal ganglia and extrapyramidal system? a. Myasthenia gravis b. Parkinson's disease c. Huntington's disease d. Amyotrophic lateral sclerosis (ALS)

19. c. Huntington's disease (HD) involves deficiency of acetylcholine and γ-aminobutyric acid (GABA) in the basal ganglia and extrapyramidal system that causes the opposite symptoms of parkinsonism. Myasthenia gravis involves autoimmune antibody destruction of cholinergic receptors at the neuromuscular junction. Amyotrophic lateral sclerosis (ALS) involves degeneration of motor neurons in the brainstem and spinal cord.

Which neurodegenerative disorder is characterized by rigidity and bradykinesia? 1 Multiple sclerosis 2 Parkinson's disease 3 Lou Gehrig's disease 4 Huntington's disease

2 Parkinson's disease Parkinson's disease is a chronic, progressive neurodegenerative disorder characterized by an increased muscle tone (known as rigidity), slowness in the initiation and execution of movement (known as bradykinesia), tremors, and gait disturbances. Multiple sclerosis is a chronic progressive degenerative disorder of the central nervous system characterized by progressive, chronic demyelination of nerve fibers of the spinal cord and brain. Lou Gehrig's disease is a rare progressive neurologic disorder that involves degeneration of motor neurons in the spinal cord and brain. It is characterized by limb weakness, dysarthria, and dysphagia. Huntington's disease is a genetically transmitted, autosomal dominant disorder characterized by chorea movements and cognitive and psychiatric disorders.

Which disease is characterized by a degeneration of dopamine-producing neurons and decreased levels of dopamine? 1 Multiple sclerosis 2 Parkinson's disease 3 Lou Gehrig's disease 4 Huntington's disease

2 Parkinson's disease Parkinson's disease is characterized by a lack of dopamine because of the degeneration of dopamine-producing neurons; this disrupts the normal balance between acetylcholine and dopamine in the brain. Multiple sclerosis is due to the demyelination of the nerve fibers of the brain and spinal cord. Characteristics of this disease are not associated with a deficiency of neurotransmitters. Lou Gehrig's disease is a rare, progressive neurologic disorder that involves the degeneration of motor neurons in the brain and spinal cord, not the degeneration of neurons producing dopamine. Huntington's disease involves a deficiency of the neurotransmitters acetylcholine and γ-aminobutyric acid. This disease is characterized by increased levels of dopamine.

A patient has been diagnosed with amyotrophic lateral sclerosis (ALS). What nursing intervention is most important to help prevent a common cause of death for patients with ALS? 1 Reduce fat intake 2 Reduce the risk of aspiration 3 Decrease injury related to falls 4 Decrease pain secondary to muscle weakness

2 Reduce the risk of aspiration Reducing the risk of aspiration can help prevent respiratory infections that are a common cause of death from deteriorating muscle function. Reducing fat intake may reduce cardiovascular disease, but this is not a common cause of death for patients with ALS. Decreasing injury related to falls and decreasing pain secondary to muscle weakness are important nursing interventions for patients with ALS but are unrelated to causes of death for these patients.

A patient is diagnosed with multiple sclerosis (MS) and is prescribed interferon. What should the nurse include in medication teaching? 1 The medication should be taken before meals on an empty stomach. 2 The medication often causes patients to experience flu-like symptoms. 3 The medication is given during exacerbation of symptoms to promote remission. 4 The medication alters carbohydrate metabolism and elevates serum glucose levels.

2 The medication often causes patients to experience flu-like symptoms. Interferon is an immunomodulator medication that treats the disease process and prevents relapses. The medication often causes flu-like symptoms, achiness, and headache, which are treated effectively with mild analgesics such as acetaminophen or ibuprofen. The medication is administered subcutaneously every other day. Exacerbations of MS are treated with corticosteroid medications. Corticosteroids alter carbohydrate metabolism and elevate serum glucose levels.

The nurse is performing a physical examination on a patient with Parkinson's disease. What manifestations of Parkinson's disease is the nurse likely to find? Select all that apply. 1 Nystagmus 2 Drooling of saliva 3 Patchy blindness 4 Decreased arm swing 5 Shuffling, propulsive gait

2 Drooling of saliva 4 Decreased arm swing 5 Shuffling, propulsive gait The patient may manifest drooling of saliva; shuffling, propulsive gait; and decreased arm swing. These symptoms are due to the combination of tremors, rigidity of muscles, and bradykinesia. Patchy blindness and nystagmus are not found in Parkinson's disease.

A patient with Parkinson's disease is prescribed sinemet. For which side effects of the drug should the nurse monitor in the patient? Select all that apply. 1 Dizziness 2 Dyskinesia 3 Severe headache 4 Involuntary eye movements 5 Severe nausea and vomiting

2 Dyskinesia 4 Involuntary eye movements 5 Severe nausea and vomiting Sinemet is a combination of levodopa and carbidopa that is prescribed to patients suffering from parkinsonism. The drug has few side effects. These side effects include dyskinesia due to increased dopamine availability. Severe nausea and vomiting is another important side effect, because the dopaminergic pathway is the major pathway involved in emesis. Increased dopamine levels in the body many trigger the sensation of nausea and vomiting. Involuntary eye movements are due to the increased levels of the neurotransmitter dopamine in the body. Dizziness or fainting, due to orthostatic hypotension, is a side effect of the drug bromocriptine. A severe headache is also a side effect of bromocriptine.

A patient complains of difficulty with eye and eyelid movement, chewing, swallowing, speaking, and breathing. Which findings in the patient's diagnostic results would indicate the presence of myasthenia gravis? Select all that apply. 1 Alteration of one copy of gene in DNA test 2 Improved muscle contractibility during the Tensilon test 3 Presence of three signs of the classic triad: tremor, rigidity, and bradykinesia 4 Decreased response to repeated stimulation of hand muscles during EMG test 5 Increased immunoglobulin G levels in the cerebrospinal fluid during CSF analysis test

2 Improved muscle contractibility during the Tensilon test 4 Decreased response to repeated stimulation of hand muscles during EMG test Patients with myasthenia gravis experience improved muscle contractibility after intravenous administration of the anticholinesterase agent edrophonium chloride. This test aids in diagnosis of myasthenia gravis. EMG testing may show a decreased response to repeated stimulation of muscles in the hands, which would indicate muscle fatigue. Single-fiber EMG is a confirmative test for myasthenia gravis. The alteration of one copy of a gene in a DNA test is a diagnostic test for Huntington's disease, a genetically transmitted autosomal dominant disorder. The presence of three signs of the classic triad, tremor, rigidity, and bradykinesia, is a confirmed diagnostic test for Parkinson's disease. Increased immunoglobulin G levels in the cerebrospinal fluid during CSF analysis test confirms multiple sclerosis, not myasthenia gravis.

20. A 38-year-old woman has newly diagnosed multiple sclerosis (MS) and asks the nurse what is going to happen to her. What is the best response by the nurse? a. "You will have either periods of attacks and remissions or progression of nerve damage over time." b. "You need to plan for a continuous loss of movement, sensory functions, and mental capabilities." c. "You will most likely have a steady course of chronic progressive nerve damage that will change your personality." d. "It is common for people with MS to have an acute attack of weakness and then not to have any other symptoms for years."

20. a. Most patients with multiple sclerosis (MS) have remissions and exacerbations of neurologic dysfunction or a relapsing-remitting initial course followed by progression with or without occasional relapses, minor remissions, and plateaus that progressively cause loss of motor, sensory, and cerebellar functions. Intellectual function generally remains intact but patients may experience anger, depression, or euphoria. A few people have chronic progressive deterioration and some may experience only occasional and mild symptoms for several years after onset.

21. During assessment of a patient admitted to the hospital with an acute exacerbation of MS, what should the nurse expect to find? a. Tremors, dysphasia, and ptosis b. Bowel and bladder incontinence and loss of memory c. Motor impairment, visual disturbances, and paresthesias d. Excessive involuntary movements, hearing loss, and ataxia

21. c. Specific neurologic dysfunction of MS is caused by destruction of myelin and replacement with glial scar tissue at specific areas in the nervous system. Motor, sensory, cerebellar, and emotional dysfunctions, including paresthesias as well as patchy blindness, blurred vision, pain radiating along the dermatome of the nerve, ataxia, and severe fatigue, are the most common manifestations of MS. Constipation and bladder dysfunctions, short-term memory loss, sexual dysfunction, anger, and depression or euphoria may also occur. Excessive involuntary movements and tremors are not seen in MS.

23. Mitoxantrone (Novantrone) is being considered as treatment for a patient with progressive-relapsing MS. The nurse explains that a disadvantage of this drug compared with other drugs used for MS is what? a. It must be given subcutaneously every day. b. It has a lifetime dose limit because of cardiac toxicity. c. It is an anticholinergic agent that causes urinary incontinence. d. It is an immunosuppressant agent that increases the risk for infection.

23. b. Mitoxantrone (Novantrone) cannot be used for more than 2 to 3 years because it is an antineoplastic drug that causes cardiac toxicity, leukemia, and infertility. It is a monoclonal antibody given IV monthly when patients have inadequate responses to other drugs. It increases the risk of progressive multifocal leukoencephalopathy.

if a patient has trigeminal neuralgia which causes paroxysmal pain what intervention should the nurse include in the care plan?

this involves the trigeminal nerve avoid care that involves touching the patient's face

Priority Decision: A patient with MS has a nursing diagnosis of self-care deficit related to muscle spasticity and neuromuscular deficits. In providing care for the patient, what is most important for the nurse to do? a. Teach the family members how to care adequately for the patient's needs. b. Encourage the patient to maintain social interactions to prevent social isolation. c. Promote the use of assistive devices so the patient can participate in self-care activities. d. Perform all activities of daily living (ADLs) for the patient to conserve the patient's energy.

24. c. The main goal in care of the patient with MS is to keep the patient active and maximally functional and promote self-care as much as possible to maintain independence. Assistive devices encourage independence while preserving the patient's energy. No care activity that the patient can do for himself or herself should be performed by others. Involvement of the family in the patient's care and maintenance of social interactions are also important but are not the priority in care.

25. A patient with newly diagnosed MS has been hospitalized for evaluation and initial treatment of the disease. Following discharge teaching, the nurse realizes that additional instruction is needed when the patient says what? a. "It is important for me to avoid exposure to people with upper respiratory infections." b. "When I begin to feel better, I should stop taking the prednisone to prevent side effects." c. "I plan to use vitamin supplements and a high-protein diet to help manage my condition." d. "I must plan with my family how we are going to manage my care if I become more incapacitated."

25. b. Corticosteroids used in treating acute exacerbations of MS should not be abruptly stopped by the patient because adrenal insufficiency may result and prescribed tapering doses should be followed. Infections may exacerbate symptoms and should be avoided and high-protein diets with vitamin supplements are advocated. Long-term planning for increasing disability is also important.

26. The classic triad of manifestations associated with Parkinson's disease is tremor, rigidity, and bradykinesia. What is a consequence related to rigidity? a. Shuffling gait b. Impaired handwriting c. Lack of postural stability d. Muscle soreness and pain

26. d. The degeneration of dopamine-producing neurons in the substantia nigra of midbrain and basal ganglia lead to this triad of signs. Muscle soreness, pain, and slowness of movement are patient function consequences related to rigidity. Shuffling gait, lack of postural stability, absent arm swing while walking, absent blinking, masked facial expression, and difficulty initiating movement are all related to bradykinesia. Impaired handwriting and hand activities are related to the tremor of Parkinson's disease (PD).

27. A patient with a tremor is being evaluated for Parkinson's disease. The nurse explains to the patient that Parkinson's disease can be confirmed by a. CT and MRI scans. b. relief of symptoms with administration of dopaminergic agents. c. the presence of tremors that increase during voluntary movement. d. cerebral angiogram that reveals the presence of cerebral atherosclerosis.

27. b. Although clinical manifestations are characteristic in PD, no laboratory or diagnostic tests are specific for the condition. A diagnosis is made when at least two of the three signs of the classic triad are present and it is confirmed with a positive response to antiparkinsonian medication. Research regarding the role of genetic testing and MRI to diagnose PD is ongoing. Essential tremors increase during voluntary movement whereas the tremors of PD are more prominent at rest.

When performing a physical assessment of a patient with myasthenia gravis, what clinical manifestations is the nurse likely to find? Select all that apply. 1 Muscle atrophy 2 Abnormal reflexes 3 Impaired facial mobility 4 Difficulty in swallowing food 5 Voice fading after a long conversation

3 Impaired facial mobility 4 Difficulty in swallowing food 5 Voice fading after a long conversation Myasthenia gravis is characterized by a fluctuating weakness of skeletal muscles. The patient may have impaired facial mobility and expressions. Chewing and swallowing food may become difficult. Speech may be affected, and the voice fades after a long conversation. Apart from this muscle weakness, there is no other dysfunction such as muscle atrophy or abnormal reflexes.

28. Which observation of the patient made by the nurse is most indicative of Parkinson's disease? a. Large, embellished handwriting b. Weakness of one leg resulting in a limping walk c. Difficulty rising from a chair and beginning to walk d. Onset of muscle spasms occurring with voluntary movement

28. c. The bradykinesia of PD prevents automatic movements and activities such as beginning to walk, rising from a chair, or even swallowing saliva cannot be executed unless they are consciously willed. Handwriting is affected by the tremor and results in the writing trailing off at the end of words. Specific limb weakness and muscle spasms are not characteristic of PD.

29. A patient with Parkinson's disease is started on levodopa. What should the nurse explain about this drug? a. It stimulates dopamine receptors in the basal ganglia. b. It promotes the release of dopamine from brain neurons. c. It is a precursor of dopamine that is converted to dopamine in the brain. d. It prevents the excessive breakdown of dopamine in the peripheral tissues.

29. c. Peripheral dopamine does not cross the blood-brain barrier but its precursor, levodopa, is able to enter the brain, where it is converted to dopamine, increasing the supply that is deficient in PD. Other drugs used to treat PD include bromocriptine, which stimulates dopamine receptors in the basal ganglia, and amantadine, which blocks the reuptake of dopamine into presynaptic neurons. Carbidopa is an agent that is usually administered with levodopa to prevent the levodopa from being metabolized in peripheral tissues before it can reach the brain.

Which nursing diagnosis is likely to be a priority in the care of a patient with myasthenia gravis (MG)? 1 Acute confusion 2 Bowel incontinence 3 Activity intolerance 4 Disturbed sleep pattern

3 Activity intolerance The primary feature of MG is fluctuating weakness of skeletal muscle. Bowel incontinence and confusion are unlikely signs of MG, and although sleep disturbance is likely, activity intolerance is usually of primary concern.

A patient with a diagnosis of Parkinson's disease (PD) has been admitted recently to a long-term care facility. Which action should the health care team take to promote adequate nutrition for this patient? 1 Provide multivitamins with each meal. 2 Provide a diet that is low in complex carbohydrates and high in protein. 3 Provide small, frequent meals throughout the day that are easy to chew and swallow. 4 Provide the patient with a minced or pureed diet that is high in potassium and low in sodium.

3 Provide small, frequent meals throughout the day that are easy to chew and swallow. Nutritional support is a priority in the care of individuals with PD. Such patients may benefit from meals that are smaller and more frequent than normal and that are easy to chew and swallow. Multivitamins are not necessary at each meal, and vitamin intake, along with protein intake, must be monitored to prevent contraindications with medications. It is likely premature to introduce a minced or pureed diet, and a low-carbohydrate diet is not indicated.

Which criterion is included to establish that a patient has Parkinson's disease? Select all that apply. 1 Decreased serum dopamine levels. 2 Tumor present in the thymus gland. 3 Positive response to antiparkinsonian medications. 4 Magnetic resonance imaging (MRI) shows areas of plaque on cranial nerves. 5 Presence of two of the three classic features: rigidity, bradykinesia, and tremor.

3 Positive response to antiparkinsonian medications. 5 Presence of two of the three classic features: rigidity, bradykinesia, and tremor. Presently there is no specific test to diagnose Parkinson's disease. Diagnosis is made when the patient demonstrates two of the three classic signs: rigidity (increased resistance to passive motion as a cogwheel); bradykinesia (slowed and loss of automatic coordinated movement); and tremor (a tremor that is more severe at rest and pill-rolling hand tremor). The diagnosis is confirmed with positive response to medications used to treat the disease such as carbidopa/levodopa. Parkinson's disease is caused by decreased dopamine levels in the brain and is not measured in the serum. Tumors of the thymus gland are associated with myasthenia gravis. Areas of plaque are seen on MRI of patients with multiple sclerosis.

A patient with multiple sclerosis is under treatment with β-interferon. What patient teaching would be appropriate for this patient? Select all that apply. 1 Do not drink grape juice. 2 Monitor vital signs regularly. 3 Rotate injection sites with each dose. 4 Know that flu-like symptoms are common. 5 Wear sunscreen while exposed to sunlight.

3 Rotate injection sites with each dose. 4 Know that flu-like symptoms are common. 5 Wear sunscreen while exposed to sunlight. Patients on β-Interferon should wear sunscreen when exposed to sunlight because the drug may cause photosensitivity. The injection site should be rotated with each dose to prevent lipodystrophy. The nurse should let the patient know that flu-like symptoms are common with β-interferon. The symptoms usually subside on their own; if they do not, they can be treated with nonsteroidal antiinflammatory drugs. The drug does not interact with grape juice; therefore, grape juice can be consumed. Monitoring of vital signs is not a specific teaching related to the drug.

31. A patient with myasthenia gravis is admitted to the hospital with respiratory insufficiency and severe weakness. When is a diagnosis of cholinergic crisis made? a. The patient's respiration is impaired because of muscle weakness. b. Administration of edrophonium (Tensilon) increases muscle weakness. c. Administration of edrophonium (Tensilon) results in improved muscle contractility. d. EMG reveals decreased response to repeated stimulation of muscles.

31. b. The reduction of the acetylcholine (ACh) effect in myasthenia gravis (MG) is treated with anticholinesterase drugs, which prolong the action of ACh at the neuromuscular synapse, but too much of these drugs will cause a cholinergic crisis with symptoms very similar to those of MG. To determine whether the patient's manifestations are due to a deficiency of ACh or to too much anticholinesterase drug, the anticholinesterase drug edrophonium chloride (Tensilon) is administered. If the patient is in cholinergic crisis, the patient's symptoms will worsen; if the patient is in a myasthenic crisis, the patient will improve.

Priority Decision: During care of a patient in myasthenic crisis, maintenance of what is the nurse's first priority for the patient? a. Mobility b. Nutrition c. Respiratory function d. Verbal communication

32. c. The patient in myasthenic crisis has severe weakness and fatigability of all skeletal muscles, affecting the patient's ability to breathe, swallow, talk, and move. However, the priority of nursing care is monitoring and maintaining adequate ventilation.

33. When providing care for a patient with ALS, the nurse recognizes what as one of the most distressing problems experienced by the patient? a. Painful spasticity of the face and extremities b. Retention of cognitive function with total degeneration of motor function c. Uncontrollable writhing and twisting movements of the face, limbs, and body d. Knowledge that there is a 50% chance the disease has been passed to any offspring

33. b. In ALS there is gradual degeneration of motor neurons with extreme muscle wasting from lack of stimulation and use. However, cognitive function is not impaired and patients feel trapped in a dying body. Chorea manifested by writhing, involuntary movements is characteristic of HD. As an autosomal dominant genetic disease, HD also has a 50% chance of being passed to each offspring.

34. In providing care for patients with chronic, progressive neurologic disease, what is the major goal of treatment that the nurse works toward? a. Meet the patient's personal care needs. b. Return the patient to normal neurologic function. c. Maximize neurologic functioning for as long as possible. d. Prevent the development of additional chronic diseases.

34. c. Many chronic neurologic diseases involve progressive deterioration in physical or mental capabilities and have no cure, with devastating results for patients and families. Health care providers can only attempt to alleviate physical symptoms, prevent complications, and assist patients in maximizing function and self-care abilities for as long as possible.

What is the major complication associated with myasthenic crisis? 1 Speech alteration 2 Difficulty chewing 3 Impaired facial mobility 4 Respiratory insufficiency

4 Respiratory insufficiency Myasthenic crisis is the acute exacerbation of myasthenia gravis. This is a life-threatening condition that occurs when the muscles that control breathing and swallowing become too weak to perform their functions. The clinical manifestations of myasthenia gravis include speech alteration due to muscle weakness related to speech. However, this is not a major complication associated with myasthenic crisis. Difficulty in chewing is due to the weakness of muscles related to chewing but is not a major complication associated with myasthenic crisis. Impaired facial mobility and expression are the clinical manifestations of myasthenia gravis due to the weakness of facial muscles but are not a major complication associated with myasthenic crisis.

4. What drug therapy is included for acute migraine and cluster headaches that appears to alter the pathophysiologic process for these headaches? a. β-Adrenergic blockers such as propranolol (Inderal) b. Serotonin antagonists such as methysergide (Sansert) c. Tricyclic antidepressants such as amitriptyline (Elavil) d. Specific serotonin receptor agonists such as sumatriptan (Imitrex)

4. d. Triptans (sumatriptan [Imitrex]) affect selected serotonin receptors that decrease neurogenic inflammation of the cerebral blood vessels and produce vasoconstriction. Both migraine headaches and cluster headaches appear to be related to vasodilation of cranial vessels and drugs that cause vasoconstriction are useful in treatment of migraine and cluster headaches. Methysergide blocks serotonin receptors in the central and peripheral nervous systems and is used for prevention of migraine and cluster headaches. β adrenergic blockers and tricyclic antidepressants are used prophylactically for migraine headaches but are not effective for cluster headaches.

After receiving change-of-shift report on the following four patients, which patient should the nurse see first?

A patient with right-sided weakness who has an infusion of tPA prescribed rational: tPA needs to be infused within the first few hours after stroke symptoms start in order to be effective in minimizing brain injury. The other medications also should be given as quickly as possible, but timing of the medications is not as critical.

5. What is a nursing intervention that is appropriate for the patient with a nursing diagnosis of anxiety related to lack of knowledge of the etiology and treatment of headache? a. Help the patient to examine lifestyle patterns and precipitating factors. b. Administer medications as ordered to relieve pain and promote relaxation. c. Provide a quiet, dimly lit environment to reduce stimuli that increase muscle tension and anxiety. d. Support the patient's use of counseling or psychotherapy to enhance conflict resolution and stress reduction.

5. a. When the anxiety is related to a lack of knowledge about the etiology and treatment of a headache, helping the patient to identify stressful lifestyle patterns and other precipitating factors and ways of avoiding them are appropriate nursing interventions for the anxiety. Interventions that teach alternative therapies to supplement drug therapy also give the patient some control over pain and are appropriate teaching regarding treatment of the headache. The other interventions may help to reduce anxiety generally but they do not address the etiologic factor of the anxiety.

A patient has been taking phenytoin (Dilantin) for 2 years. Which action will the nurse take when evaluating for adverse effects of the medication? a. Inspect the oral mucosa. b. Listen to the lung sounds. c. Auscultate the bowel tones. d. Check pupil reaction to light.

ANS: A Phenytoin can cause gingival hyperplasia, but does not affect bowel tones, lung sounds, or pupil reaction to light.

how far should a catheter be inserted in a male patient?

6-8 inches

6. Delegation Decision: The nurse is preparing to admit a newly diagnosed patient experiencing tonic-clonic seizures. What could the nurse delegate to unlicensed assistive personnel (UAP)? a. Complete the admission assessment. b. Explain the call system to the patient. c. Obtain the suction equipment from the supply cabinet. d. Place a padded tongue blade on the wall above the patient's bed.

6. c. The unlicensed assistive personnel (UAP) is able to obtain equipment from the supply cabinet or department. The RN may need to provide a list of necessary equipment and should set up the equipment and ensure proper functioning. The RN is responsible for the initial history and assessment as well as teaching the patient about the room's call system. Padded tongue blades are no longer used and no effort should be made to place anything in the patient's mouth during a seizure.

7. How do generalized seizures differ from focal seizures? a. Focal seizures are confined to one side of the brain and remain focal in nature. b. Generalized seizures result in loss of consciousness whereas focal seizures do not. c. Generalized seizures result in temporary residual deficits during the postictal phase. d. Generalized seizures have bilateral synchronous epileptic discharges affecting the whole brain at onset of the seizure.

7. d. Generalized seizures have bilateral synchronous epileptic discharge affecting the entire brain at onset of the seizure. Loss of consciousness is also characteristic but many focal seizures also include an altered consciousness. Focal seizures begin in one side of the brain but may spread to involve the entire brain. Focal seizures that start with a local focus and spread to the entire brain, causing a secondary generalized seizure, are associated with a transient residual neurologic deficit postictally known as Todd's paralysis.

An unconscious patient with a traumatic head injury has a blood pressure of 126/72 mm Hg, and an intracranial pressure of 18 mm Hg. The nurse will calculate the cerebral perfusion pressure as ____________________.

72 mm Hg (The formula for calculation of cerebral perfusion pressure is [(Systolic pressure + Diastolic blood pressure × 2)/3] = intracranial pressure.)

9. The patient is diagnosed with complex focal seizures. Which characteristics are related to complex focal seizures (select all that apply)? a. Formerly known as grand mal seizure b. Often accompanied by incontinence or tongue or cheek biting c. Psychomotor seizures with repetitive behaviors and lip smacking d. Altered memory, sexual sensations, and distortions of visual or auditory sensations e. Loss of consciousness and stiffening of the body with subsequent jerking of extremities f. Often involves behavioral, emotional, and cognitive functions with altered consciousness

9. c, d, f. Complex focal seizures are psychomotor seizures with automatisms such as lip smacking. They cause altered consciousness or loss of consciousness producing a dreamlike state and may involve behavioral, emotional, or cognitive experiences without memory of what was done during the seizure. In generalized tonic-clonic seizures (previously known as grand mal seizures) there is loss of consciousness and stiffening of the body with subsequent jerking of extremities. Incontinence or tongue or cheek biting may also occur.

Which of these patients is most appropriate for the intensive care unit (ICU) charge nurse to assign to an RN who has floated from the medical unit?

A 44-year-old receiving IV antibiotics for meningococcal meningitis rational: An RN who works on a medical unit will be familiar with administration of IV antibiotics and with meningitis. The postcraniotomy patient, patient with an ICP monitor, and the patient on a ventilator should be assigned to an RN familiar with the care of critically ill patients.

What is the most important indicator of increased ICP?

A change in the level of responsiveness

Define stroke.

A disruption of blood supply to a part of the brain, which results in sudden loss of brain function

A nurse is interviewing a patient who is seeking relief for frequent headaches. Which description is consistent with symptoms of a migraine headache? 1 Extreme tenseness in the area of the neck and shoulders. 2 Tears flow from one eye and nasal drainage occurs with the headache. 3 The pain of the headache wakes the patient from sleep. 4 The pain throbs and is synchronous with the patient's pulse.

A migraine headache is caused by a series of neurovascular events that result from some trigger stimulus. The pain usually is one-sided, throbbing in nature, and synchronous with the patient's pulse. Palpable tenseness in the neck and shoulders occurs with a tension headache. A cluster headache awakens the patient from sleep and involves tearing of one eye with nasal drainage on the same side. Text Reference - p. 1414 4

After the emergency department nurse has received a status report on the following patients who have been admitted with head injuries, which patient should the nurse assess first?

A patient whose right pupil is 10 mm and unresponsive to light rational: The dilated and nonresponsive pupil may indicate an intracerebral hemorrhage and increased intracranial pressure. The other patients are not at immediate risk for complications such as herniation.

What precautions should the nurse follow when feeding a patient with a stroke on the left side? Select all that apply. 1 Place the patient in high Fowler's position. 2 Place the patient in a chair with the head flexed forward. 3 Place the patient in low Fowler's position. 4 Place the patient in Trendelenburg's position. 5 Place the patient in right lateral position.

A person in a Fowler's position is sitting straight up or leaning slightly back. Their legs may either be straight or bent. A high Fowler's position is someone who is sitting upright. This helps in feeding as well as swallowing for the patient. Sitting in a chair with the head flexed forward also serves a similar purpose. Low Fowler's, Trendelenburg's, and right lateral positions are not appropriate for feeding. Text Reference - p. 1406 1,2

Identify a causative factor associated with a sensorineural hearing loss. 1 Otosclerosis 2 Impacted cerumen 3 Perforation of the tympanic membrane 4 Ototoxicity from chemotherapeutic medications

A sensorineural hearing loss involves damage to the neural component of hearing. Within the inner ear there are sensory hair cells in the cochlea that pick up impulses and initiate nerve impulse to the brain via the acoustic branch of the eighth cranial nerve. Medications, including some chemotherapy, can damage this nerve pathway of hearing. Otosclerosis is a hereditary condition causing a conductive hearing loss because of bony growth that limits the stapes from vibrating in response to sound. Impacted cerumen (earwax) causes a conductive hearing loss because the sound vibrations are blocked by excessive earwax in the ear canal. A perforated tympanic membrane (ruptured eardrum) causes a conductive hearing loss because the tympanic membrane is no longer intact to adequately conduct sound vibrations. Text Reference - p. 407 4

The nurse organizes the plan of care with regard to normal brain alterations associated with age, which are (select all that apply): 1. decrease in brain weight. 2. pigmentation of brain with lipofuscin. 3. present of amyloid. 4. tiny clot formation. 5. tangled nerve fibers

ANS: 1, 2, 3, 5 All listed are expected changes that affect the older adult's neurologic function except for tiny clot formations, which are pathologic changes

The nurse explains that neurotransmitters, which support smooth neural transmission, are (select all that apply): 1. acetylcholine. 2. cerebrospinal fluid. 3. dopamine. 4. dendrite. 5. epinephrine.

ANS: 1, 3, 5 Acetylcholine, dopamine, and epinephrine are neurotransmitters. CSF bathes the brain and spinal cord but has no transmission activity; the dendrite is the locus of the synapse.

The nurse assesses a K + level of 6.2 mEq/L and becomes alert for the associated sign of (se-lect all that apply): 1. pain on movement. 2. contractions of the hands. 3. twitching of muscles. 4. lethargy. 5. trembling of limbs

ANS: 3, 5 Increased levels of K + cause twitching and tremor related to the altered depolarization that provides for smooth organized function

A 64-year-old patient who has amyotrophic lateral sclerosis (ALS) is hospitalized with pneumonia. Which nursing action will be included in the plan of care? a. Assist with active range of motion (ROM). b. Observe for agitation and paranoia. c. Give muscle relaxants as needed to reduce spasms. d. Use simple words and phrases to explain procedures.

ANS: A ALS causes progressive muscle weakness, but assisting the patient to perform active ROM will help maintain strength as long as possible. Psychotic manifestations such as agitation and paranoia are not associated with ALS. Cognitive function is not affected by ALS, and the patient's ability to understand procedures will not be impaired. Muscle relaxants will further increase muscle weakness and depress respirations.

A hospitalized 31-year-old patient with a history of cluster headache awakens during the night with a severe stabbing headache. Which action should the nurse take first? a. Start the ordered PRN oxygen at 6 L/min. b. Put a moist hot pack on the patient's neck. c. Give the ordered PRN acetaminophen (Tylenol). d. Notify the patient's health care provider immediately.

ANS: A Acute treatment for cluster headache is administration of 100% oxygen at 6 to 8 L/min. If the patient obtains relief with the oxygen, there is no immediate need to notify the health care provider. Cluster headaches last only 60 to 90 minutes, so oral pain medications have minimal effect. Hot packs are helpful for tension headaches but are not as likely to reduce pain associated with a cluster headache.

After change-of-shift report, which patient should the nurse assess first? a. Patient with myasthenia gravis who is reporting increased muscle weakness b. Patient with a bilateral headache described as "like a band around my head" c. Patient with seizures who is scheduled to receive a dose of phenytoin (Dilantin) d. Patient with Parkinson's disease who has developed cogwheel rigidity of the arms

ANS: A Because increased muscle weakness may indicate the onset of a myasthenic crisis, the nurse should assess this patient first. The other patients should also be assessed, but do not appear to need immediate nursing assessments or actions to prevent life-threatening complications.

A 31-year-old woman who has multiple sclerosis (MS) asks the nurse about risks associated with pregnancy. Which response by the nurse is accurate? a. "MS symptoms may be worse after the pregnancy." b. "Women with MS frequently have premature labor." c. "MS is associated with an increased risk for congenital defects." d. "Symptoms of MS are likely to become worse during pregnancy."

ANS: A During the postpartum period, women with MS are at greater risk for exacerbation of symptoms. There is no increased risk for congenital defects in infants born of mothers with MS. Symptoms of MS may improve during pregnancy. Onset of labor is not affected by MS.

The nurse advises a patient with myasthenia gravis (MG) to a. perform physically demanding activities early in the day. b. anticipate the need for weekly plasmapheresis treatments. c. do frequent weight-bearing exercise to prevent muscle atrophy. d. protect the extremities from injury due to poor sensory perception.

ANS: A Muscles are generally strongest in the morning, and activities involving muscle activity should be scheduled then. Plasmapheresis is not routinely scheduled, but is used for myasthenia crisis or for situations in which corticosteroid therapy must be avoided. There is no decrease in sensation with MG, and muscle atrophy does not occur because although there is muscle weakness, they are still used.

A 46-year-old patient tells the nurse about using acetaminophen (Tylenol) several times every day for recurrent bilateral headaches. Which action will the nurse plan to take first? a. Discuss the need to stop taking the acetaminophen. b. Suggest the use of biofeedback for headache control. c. Describe the use of botulism toxin (Botox) for headaches. d. Teach the patient about magnetic resonance imaging (MRI).

ANS: A The headache description suggests that the patient is experiencing medication overuse headache. The initial action will be withdrawal of the medication. The other actions may be needed if the headaches persist.

A patient reports feeling numbness and tingling of the left arm before experiencing a tonic-clonic seizure. The nurse determines that this history is consistent with what type of seizure? a. Focal b. Atonic c. Absence d. Myoclonic

ANS: A The initial symptoms of a focal seizure involve clinical manifestations that are localized to a particular part of the body or brain. Symptoms of an absence seizure are staring and a brief loss of consciousness. In an atonic seizure, the patient loses muscle tone and (typically) falls to the ground. Myoclonic seizures are characterized by a sudden jerk of the body or extremities.

The nurse observes a patient ambulating in the hospital hall when the patient's arms and legs suddenly jerk and the patient falls to the floor. The nurse will first a. assess the patient for a possible head injury. b. give the scheduled dose of divalproex (Depakote). c. document the timing and description of the seizure. d. notify the patient's health care provider about the seizure.

ANS: A The patient who has had a myoclonic seizure and fall is at risk for head injury and should first be evaluated and treated for this possible complication. Documentation of the seizure, notification of the seizure, and administration of antiseizure medications are also appropriate actions, but the initial action should be assessment for injury.

A patient with Parkinson's disease is admitted to the hospital for treatment of pneumonia. Which nursing interventions will be included in the plan of care (select all that apply)? a. Use an elevated toilet seat. b. Cut patient's food into small pieces. c. Provide high-protein foods at each meal. d. Place an armchair at the patient's bedside. e. Observe for sudden exacerbation of symptoms.

ANS: A, B, D Because the patient with Parkinson's has difficulty chewing, food should be cut into small pieces. An armchair should be used when the patient is seated so that the patient can use the arms to assist with getting up from the chair. An elevated toilet seat will facilitate getting on and off the toilet. High-protein foods will decrease the effectiveness of L-dopa. Parkinson's is a steadily progressive disease without acute exacerbations.

A 27-year-old patient who has been treated for status epilepticus in the emergency department will be transferred to the medical nursing unit. Which equipment should the nurse have available in the patient's assigned room (select all that apply)? a. Side-rail pads b. Tongue blade c. Oxygen mask d. Suction tubing e. Urinary catheter f. Nasogastric tube

ANS: A, C, D The patient is at risk for further seizures, and oxygen and suctioning may be needed after any seizures to clear the airway and maximize oxygenation. The bed's side rails should be padded to minimize the risk for patient injury during a seizure. Use of tongue blades during a seizure is contraindicated. Insertion of a nasogastric (NG) tube is not indicated because the airway problem is not caused by vomiting or abdominal distention. A urinary catheter is not required unless there is urinary retention.

The home health registered nurse (RN) is planning care for a patient with a seizure disorder related to a recent head injury. Which nursing action can be delegated to a licensed practical/vocational nurse (LPN/LVN)? a. Make referrals to appropriate community agencies. b. Place medications in the home medication organizer. c. Teach the patient and family how to manage seizures. d. Assess for use of medications that may precipitate seizures.

ANS: B LPN/LVN education includes administration of medications. The other activities require RN education and scope of practice.

Which intervention will the nurse include in the plan of care for a patient with primary restless legs syndrome (RLS) who is having difficulty sleeping? a. Teach about the use of antihistamines to improve sleep. b. Suggest that the patient exercise regularly during the day. c. Make a referral to a massage therapist for deep massage of the legs. d. Assure the patient that the problem is transient and likely to resolve.

ANS: B Nondrug interventions such as getting regular exercise are initially suggested to improve sleep quality in patients with RLS. Antihistamines may aggravate RLS. Massage does not alleviate RLS symptoms and RLS is likely to progress in most patients.

A 73-year-old patient with Parkinson's disease has a nursing diagnosis of impaired physical mobility related to bradykinesia. Which action will the nurse include in the plan of care? a. Instruct the patient in activities that can be done while lying or sitting. b. Suggest that the patient rock from side to side to initiate leg movement. c. Have the patient take small steps in a straight line directly in front of the feet. d. Teach the patient to keep the feet in contact with the floor and slide them forward.

ANS: B Rocking the body from side to side stimulates balance and improves mobility. The patient will be encouraged to continue exercising because this will maintain functional abilities. Maintaining a wide base of support will help with balance. The patient should lift the feet and avoid a shuffling gait.

Which action will the nurse plan to take for a 40-year-old patient with multiple sclerosis (MS) who has urinary retention caused by a flaccid bladder? a. Decrease the patient's evening fluid intake. b. Teach the patient how to use the Credé method. c. Suggest the use of adult incontinence briefs for nighttime only. d. Assist the patient to the commode every 2 hours during the day.

ANS: B The Credé method can be used to improve bladder emptying. Decreasing fluid intake will not improve bladder emptying and may increase risk for urinary tract infection (UTI) and dehydration. The use of incontinence briefs and frequent toileting will not improve bladder emptying.

When a 74-year-old patient is seen in the health clinic with new development of a stooped posture, shuffling gait, and pill rolling-type tremor, the nurse will anticipate teaching the patient about a. oral corticosteroids. b. antiparkinsonian drugs. c. magnetic resonance imaging (MRI). d. electroencephalogram (EEG) testing.

ANS: B The diagnosis of Parkinson's is made when two of the three characteristic manifestations of tremor, rigidity, and bradykinesia are present. The confirmation of the diagnosis is made on the basis of improvement when antiparkinsonian drugs are administered. This patient has symptoms of tremor and bradykinesia. The next anticipated step will be treatment with medications. MRI and EEG are not useful in diagnosing Parkinson's disease, and corticosteroid therapy is not used to treat it.

A high school teacher who has just been diagnosed with epilepsy after having a generalized tonic-clonic seizure tells the nurse, "I cannot teach anymore, it will be too upsetting if I have a seizure at work." Which response by the nurse is best? a. "You might benefit from some psychologic counseling." b. "Epilepsy usually can be well controlled with medications." c. "You will want to contact the Epilepsy Foundation for assistance." d. "The Department of Vocational Rehabilitation can help with work retraining."

ANS: B The nurse should inform the patient that most patients with seizure disorders are controlled with medication. The other information may be necessary if the seizures persist after treatment with antiseizure medications is implemented.

Following a thymectomy, a 62-year-old male patient with myasthenia gravis receives the usual dose of pyridostigmine (Mestinon). An hour later, the patient complains of nausea and severe abdominal cramps. Which action should the nurse take first? a. Auscultate the patient's bowel sounds. b. Notify the patient's health care provider. c. Administer the prescribed PRN antiemetic drug. d. Give the scheduled dose of prednisone (Deltasone).

ANS: B The patient's history and symptoms indicate a possible cholinergic crisis. The health care provider should be notified immediately, and it is likely that atropine will be prescribed. The other actions will be appropriate if the patient is not experiencing a cholinergic crisis.

A hospitalized patient complains of a bilateral headache, 4/10 on the pain scale, that radiates from the base of the skull. Which prescribed PRN medications should the nurse administer initially? a. Lorazepam (Ativan) b. Acetaminophen (Tylenol) c. Morphine sulfate (Roxanol) d. Butalbital and aspirin (Fiorinal)

ANS: B The patient's symptoms are consistent with a tension headache, and initial therapy usually involves a nonopioid analgesic such as acetaminophen, which is sometimes combined with a sedative or muscle relaxant. Lorazepam may be used in conjunction with acetaminophen but would not be appropriate as the initial monotherapy. Morphine sulfate and butalbital and aspirin would be more appropriate for a headache that did not respond to a nonopioid analgesic.

The health care provider is considering the use of sumatriptan (Imitrex) for a 54-year-old male patient with migraine headaches. Which information obtained by the nurse is most important to report to the health care provider? a. The patient drinks 1 to 2 cups of coffee daily. b. The patient had a recent acute myocardial infarction. c. The patient has had migraine headaches for 30 years. d. The patient has taken topiramate (Topamax) for 2 months.

ANS: B The triptans cause coronary artery vasoconstriction and should be avoided in patients with coronary artery disease. The other information will be reported to the health care provider, but none of it indicates that sumatriptan would be an inappropriate treatment.

The nurse will assess a 67-year-old patient who is experiencing a cluster headache for a. nuchal rigidity. b. unilateral ptosis. c. projectile vomiting. d. throbbing, bilateral facial pain.

ANS: B Unilateral eye edema, tearing, and ptosis are characteristic of cluster headaches. Nuchal rigidity suggests meningeal irritation, such as occurs with meningitis. Although nausea and vomiting may occur with migraine headaches, projectile vomiting is more consistent with increased intracranial pressure (ICP). Unilateral sharp, stabbing pain, rather than throbbing pain, is characteristic of cluster headaches.

When obtaining a health history and physical assessment for a 36-year-old female patient with possible multiple sclerosis (MS), the nurse should a. assess for the presence of chest pain. b. inquire about urinary tract problems. c. inspect the skin for rashes or discoloration. d. ask the patient about any increase in libido.

ANS: B Urinary tract problems with incontinence or retention are common symptoms of MS. Chest pain and skin rashes are not symptoms of MS. A decrease in libido is common with MS.

Which assessment is most important for the nurse to make regarding a patient with myasthenia gravis? a. Pupil size b. Grip strength c. Respiratory effort d. Level of consciousness

ANS: C Because respiratory insufficiency may be life threatening, it will be most important to monitor respiratory function. The other data also will be assessed but are not as critical.

While the nurse is transporting a patient on a stretcher to the radiology department, the patient begins having a tonic-clonic seizure. Which action should the nurse take? a. Insert an oral airway during the seizure to maintain a patent airway. b. Restrain the patient's arms and legs to prevent injury during the seizure. c. Time and observe and record the details of the seizure and postictal state. d. Avoid touching the patient to prevent further nervous system stimulation.

ANS: C Because the diagnosis and treatment of seizures frequently are based on the description of the seizure, recording the length and details of the seizure is important. Insertion of an oral airway and restraining the patient during the seizure are contraindicated. The nurse may need to move the patient to decrease the risk of injury during the seizure.

A 49-year-old patient with multiple sclerosis (MS) is to begin treatment with glatiramer acetate (Copaxone). Which information will the nurse include in patient teaching? a. Recommendation to drink at least 4 L of fluid daily b. Need to avoid driving or operating heavy machinery c. How to draw up and administer injections of the medication d. Use of contraceptive methods other than oral contraceptives

ANS: C Copaxone is administered by self-injection. Oral contraceptives are an appropriate choice for birth control. There is no need to avoid driving or drink large fluid volumes when taking glatiramer.

A 40-year-old patient is diagnosed with early Huntington's disease (HD). When teaching the patient, spouse, and children about this disorder, the nurse will provide information about the a. use of levodopa-carbidopa (Sinemet) to help reduce HD symptoms. b. prophylactic antibiotics to decrease the risk for aspiration pneumonia. c. option of genetic testing for the patient's children to determine their own HD risks. d. lifestyle changes of improved nutrition and exercise that delay disease progression.

ANS: C Genetic testing is available to determine whether an asymptomatic individual has the HD gene. The patient and family should be informed of the benefits and problems associated with genetic testing. Sinemet will increase symptoms of HD because HD involves an increase in dopamine. Antibiotic therapy will not reduce the risk for aspiration. There are no effective treatments or lifestyle changes that delay the progression of symptoms in HD.

The nurse determines that teaching about management of migraine headaches has been effective when the patient says which of the following? a. "I can take the (Topamax) as soon as a headache starts." b. "A glass of wine might help me relax and prevent a headache." c. "I will lie down someplace dark and quiet when the headaches begin." d. "I should avoid taking aspirin and sumatriptan (Imitrex) at the same time."

ANS: C It is recommended that the patient with a migraine rest in a dark, quiet area. Topiramate (Topamax) is used to prevent migraines and must be taken for several months to determine effectiveness. Aspirin or other nonsteroidal antiinflammatory medications can be taken with the triptans. Alcohol may precipitate migraine headaches.

A 22-year-old patient seen at the health clinic with a severe migraine headache tells the nurse about having other similar headaches recently. Which initial action should the nurse take? a. Teach about the use of triptan drugs. b. Refer the patient for stress counseling. c. Ask the patient to keep a headache diary. d. Suggest the use of muscle-relaxation techniques.

ANS: C The initial nursing action should be further assessment of the precipitating causes of the headaches, quality, and location of pain, etc. Stress reduction, muscle relaxation, and the triptan drugs may be helpful, but more assessment is needed first.

List four rationales for the appearance of restlessness in the unconscious client

Anoxia Distended bladder Covert bleeding Return to consciousness

Which medication taken by a patient with restless legs syndrome should the nurse discuss with the patient? a. Multivitamin (Stresstabs) b. Acetaminophen (Tylenol) c. Ibuprofen (Motrin, Advil) d. Diphenhydramine (Benadryl)

ANS: D Antihistamines can aggravate restless legs syndrome. The other medications will not contribute to restless legs syndrome.

Which information about a 60-year-old patient with MS indicates that the nurse should consult with the health care provider before giving the prescribed dose of dalfampridine (Ampyra)? a. The patient has relapsing-remitting MS. b. The patient walks a mile a day for exercise. c. The patient complains of pain with neck flexion. d. The patient has an increased serum creatinine level.

ANS: D Dalfampridine should not be given to patients with impaired renal function. The other information will not impact whether the dalfampridine should be administered.

A 76-year-old patient is being treated with carbidopa/levodopa (Sinemet) for Parkinson's disease. Which information is most important for the nurse to report to the health care provider? a. Shuffling gait b. Tremor at rest c. Cogwheel rigidity of limbs d. Uncontrolled head movement

ANS: D Dyskinesia is an adverse effect of the Sinemet, indicating a need for a change in medication or decrease in dose. The other findings are typical with Parkinson's disease.

A 62-year-old patient who has Parkinson's disease is taking bromocriptine (Parlodel). Which information obtained by the nurse may indicate a need for a decrease in the dose? a. The patient has a chronic dry cough. b. The patient has four loose stools in a day. c. The patient develops a deep vein thrombosis. d. The patient's blood pressure is 92/52 mm Hg.

ANS: D Hypotension is an adverse effect of bromocriptine, and the nurse should check with the health care provider before giving the medication. Diarrhea, cough, and deep vein thrombosis are not associated with bromocriptine use.

Which information about a 72-year-old patient who has a new prescription for phenytoin (Dilantin) indicates that the nurse should consult with the health care provider before administration of the medication? a. Patient has generalized tonic-clonic seizures. b. Patient experiences an aura before seizures. c. Patient's most recent blood pressure is 156/92 mm Hg. d. Patient has minor elevations in the liver function tests.

ANS: D Many older patients (especially with compromised liver function) may not be able to metabolize phenytoin. The health care provider may need to choose another antiseizure medication. Phenytoin is an appropriate medication for patients with tonic-clonic seizures, with or without an aura. Hypertension is not a contraindication for phenytoin therapy.

Which nursing diagnosis is of highest priority for a patient with Parkinson's disease who is unable to move the facial muscles? a. Activity intolerance b. Self-care deficit: toileting c. Ineffective self-health management d. Imbalanced nutrition: less than body requirements

ANS: D The data about the patient indicate that poor nutrition will be a concern because of decreased swallowing. The other diagnoses may also be appropriate for a patient with Parkinson's disease, but the data do not indicate that they are current problems for this patient.

Which prescribed intervention will the nurse implement first for a patient in the emergency department who is experiencing continuous tonic-clonic seizures? a. Give phenytoin (Dilantin) 100 mg IV. b. Monitor level of consciousness (LOC). c. Obtain computed tomography (CT) scan. d. Administer lorazepam (Ativan) 4 mg IV.

ANS: D To prevent ongoing seizures, the nurse should administer rapidly acting antiseizure medications such as the benzodiazepines. A CT scan is appropriate, but prevention of any seizure activity during the CT scan is necessary. Phenytoin will also be administered, but it is not rapidly acting. Patients who are experiencing tonic-clonic seizures are nonresponsive, although the nurse should assess LOC after the seizure.

A patient with possible cerebral edema has a serum sodium level of 115 mEq/L (115 mmol/L) and a decreasing level of consciousness (LOC) and complains of a headache. Which of these prescribed interventions should the nurse implement first?

Administer 5% hypertonic saline intravenously. rational: The patient's low sodium indicates that hyponatremia may be causing the cerebral edema, and the nurse's first action should be to correct the low sodium level. Acetaminophen (Tylenol) will have minimal effect on the headache because it is caused by cerebral edema and increased intra-cranial pressure (ICP). Drawing ABGs and obtaining a CT scan may add some useful information, but the low sodium level may lead to seizures unless it is addressed quickly.

Which of these prescribed interventions will the nurse implement first for a hospitalized patient who is experiencing continuous tonic-clonic seizures?

Administer lorazepam (Ativan) 4 mg IV. rational: To prevent ongoing seizures, the nurse should administer rapidly acting antiseizure medications such as the benzodiazepines. A CT scan is appropriate, but prevention of any seizure activity during the CT scan is necessary. Phenytoin also will be administered, but it is not rapidly acting. Patients who are experiencing tonic-clonic seizures are nonresponsive, although the nurse should assess LOC after the seizure.

Which of these nursing actions included in the care of a patient who has been experiencing stroke symptoms for 60 minutes can the nurse delegate to an LPN/LVN?

Administer the prescribed clopidogrel (Plavix). rational: Administration of oral medications is included in LPN education and scope of practice. The other actions require more education and scope of practice and should be done by the RN.

Following a head injury, an unconscious 32-year-old patient is admitted to the emergency department (ED). The patient's spouse and children stay at the patient's side and constantly ask about the treatment being given. What action is best for the nurse to take?

Allow the family to stay with the patient and briefly explain all procedures to them. rational: The need for information about the diagnosis and care is very high in family members of acutely ill patients, and the nurse should allow the family to observe care and explain the procedures. A pastor or counseling service can offer some support, but research supports information as being more effective. Asking the family to stay in the waiting room will increase their anxiety.

HESI HINT: What are nursing diagnoses for severely neurologically impaired persons?

Almost every NANDA dx is applicable bc these patients require total care

A 32-year-old patient has a stroke resulting from a ruptured aneurysm and subarachnoid hemorrhage. Which intervention will be included in the care plan?

Applying intermittent pneumatic compression stockings rational: The patient with a subarachnoid hemorrhage usually has minimal activity to prevent cerebral vasospasm or further bleeding and is at risk for venous thromboemboism (VTE). Activities such as coughing and sitting up that might increase intracranial pressure (ICP) or decrease cerebral blood flow are avoided. Because there is no indication that the patient is unconscious, an oropharyngeal airway is inappropriate.

what type of climate should a patient with multiple sclerosis avoid?

Areas with predominantly hot weather

A patient who sustained a spinal cord injury a week ago becomes angry, telling the nurse "I want to be transferred to a hospital where the nurses know what they are doing!" Which reaction by the nurse is best?

Ask for the patient's input into the plan for care. rational: The patient is demonstrating behaviors consistent with the anger phase of the mourning process, and the nurse should allow expression of anger and seek the patient's input into care. Expression of anger is appropriate at this stage and should be tolerated by the nurse. Reassurance about the competency of the staff will not be helpful in responding to the patient's anger. Ignoring the patient's comments will increase the patient's anger and sense of helplessness.

A patient seen at the health clinic with a severe migraine headache tells the nurse about having four similar headaches in the last 3 months. Which initial action should the nurse take?

Ask the patient to keep a headache diary. rational: The initial nursing action should be further assessment of the precipitating causes of the headaches, quality, and location of pain, etc. Stress reduction, muscle relaxation, and the triptan drugs may be helpful, but more assessment is needed first.

Which action will the nurse include in the plan of care when caring for a patient who is experiencing trigeminal neuralgia?

Assess intake and output and dietary intake. rational: The patient with an acute episode of trigeminal neuralgia may be unwilling to eat or drink, so assessment of nutritional and hydration status is important. Because stimulation by touch is the precipitating factor for pain, relaxation of the facial muscles will not improve symptoms. Application of ice is likely to precipitate pain. The patient will not want to engage in conversation, which may precipitate attacks.

When caring for a patient who was admitted 24 hours previously with a C5 spinal cord injury, which nursing action has the highest priority?

Assessment of respiratory rate and depth rational: Edema around the area of injury may lead to damage above the C4 level, so the highest priority is assessment of the patient's respiratory function. The other actions also are appropriate but are not as important as assessment of respiratory effort.

Several weeks after a stroke, a patient has urinary incontinence resulting from an impaired awareness of bladder fullness. For an effective bladder training program, which nursing intervention will be best to include in the plan of care?

Assist the patient onto the bedside commode every 2 hours. rational: Developing a regular voiding schedule will prevent incontinence and may increase patient awareness of a full bladder. A 1200 mL fluid restriction may lead to dehydration. Intermittent catheterization and use of a condom catheter are appropriate in the acute phase of stroke but should not be considered solutions for long-term management because of the risks for urinary tract infection (UTI) and skin breakdown.

The nurse identifies the nursing diagnosis of imbalanced nutrition: less than body requirements related to impaired self-feeding ability for a patient with right-sided hemiplegia. Which intervention should be included in the plan of care?

Assist the patient to eat with the left hand. rational: Because the nursing diagnosis indicates that the patient's imbalanced nutrition is related to the right-sided hemiplegia, the appropriate interventions will focus on teaching the patient to use the left hand for self-feeding. The other interventions are appropriate for patients with other etiologies for the imbalanced nutrition.

A patient with amyotrophic lateral sclerosis (ALS) is hospitalized with pneumonia. Which nursing action will be included in the plan of care?

Assist with active range of motion. rational: ALS causes progressive muscle weakness, but assisting the patient to perform active ROM will help to maintain strength as long as possible. Psychotic symptoms such as agitation and paranoia are not associated with ALS. Cognitive function is not affected by ALS, and the patient's ability to understand procedures will not be impaired. Muscle relaxants will further increase muscle weakness and depress respirations.

After suctioning, the nurse notes that the intracranial pressure for a patient with a traumatic head injury has increased from 14 to 16 mm Hg. Which action should the nurse take first?

Assure that the patient's neck is not in a flexed position. rational: Since suctioning will cause a transient increase in intracranial pressure, the nurse should initially check for other factors that might be contributing to the increase and observe the patient for a few minutes. Documentation is needed, but this is not the first action. There is no need to notify the health care provider about this expected reaction to suctioning. Propofol is used to control patient anxiety or agitation; there is no indication that anxiety has contributed to the increase in intracranial pressure.

A patient found in a tonic-clonic seizure reports afterward that the seizure was preceded by numbness and tingling of the arm. The nurse knows that this finding indicates what type of seizure?

Atonic rational: The initial symptoms of a partial seizure involve clinical manifestations that are localized to a particular part of the body or brain. Symptoms of an absence seizure are staring and a brief loss of consciousness. In an atonic seizure, the patient loses muscle tone and (typically) falls to the ground. Myoclonic seizures are characterized by a sudden jerk of the body or extremities.

A patient with a head injury has admission vital signs of blood pressure 128/68, pulse 110, and respirations 26. Which of these vital signs, if taken 1 hour after admission, will be of most concern to the nurse?

Blood pressure 156/60, pulse 55, respirations 12 rational: Systolic hypertension with widening pulse pressure, bradycardia, and respiratory changes represent Cushing's triad and indicate that the intracranial pressure (ICP) has increased, and brain herniation may be imminent unless immediate action is taken to reduce ICP. The other vital signs may indicate the need for changes in treatment, but they are not indicative of an immediately life-threatening process.

Which drug helps to increase the bioavailability of levodopa in the brain by inhibiting the enzyme dopa-decarboxylase? 1 Carbidopa 2 Tolcapone 3 Rasagiline 4 Entacapone

Carbidopa is a dopamine decarboxylase inhibitor. This enzyme breaks down the dopamine peripheral tissues before reaching the brain, decreasing the bioavailability of the drug. Levodopa is the dopamine precursor converted to dopamine in the basal ganglia. Carbidopa, when administered along with levodopa, increases the bioavailability of the drug in the brain. This prevents peripheral tissue breakdown of the drug by the enzyme dopa-decarboxylase. Tolcapone and entacapone blocks the enzyme COMT that helps to increase the availability of drug levodopa. Rasagiline is a monoamine oxidase type B inhibitor that helps increase the half-life of the drug levodopa.

The care plan for a patient who has increased intracranial pressure and a ventriculostomy includes the following nursing actions. Which action can the nurse delegate to nursing assistive personnel (NAP) who regularly work in the intensive care unit?

Check capillary blood glucose level every 6 hours. rational: Experienced NAP can obtain capillary blood glucose levels when they have been trained and evaluated in the skill. Monitoring and documentation of cerebrospinal fluid (CSF) color and intracranial pressure (ICP) require RN-level education and scope of practice. Although repositioning patients is frequently delegated to NAP, repositioning a patient with a ventriculostomy is complex and should be done by the RN.

A patient with a history of a T2 spinal cord injury tells the nurse, "I feel awful today. My head is throbbing, and I feel sick to my stomach." Which action should the nurse take first?

Check the blood pressure (BP). rational: The BP should be assessed immediately in a patient with an injury at the T6 level or higher who complains of a headache to determine whether autonomic dysreflexia is occurring. Notification of the patient's health care provider is appropriate after the BP is obtained. Administration of an antiemetic is indicated after autonomic dysreflexia is ruled out as the cause of the nausea. The nurse may assess for a fecal impaction, but this should be done after checking the BP and lidocaine jelly should be used to prevent further increases in the BP.

After noting that a patient with a head injury has clear nasal drainage, which action should the nurse take?

Check the nasal drainage for glucose. rational: Clear nasal drainage in a patient with a head injury suggests a dural tear and cerebrospinal fluid (CSF) leakage. If the drainage is CSF, it will test positive for glucose. Fluid leaking from the nose will have normal nasal flora, so culture and sensitivity will not be useful. Blowing the nose is avoided to prevent CSF leakage.

A patient with left-sided hemiparesis arrives by ambulance to the emergency department. Which action should the nurse take first?

Check the respiratory rate. rational: The initial nursing action should be to assess the airway and take any needed actions to ensure a patent airway. The other activities should take place quickly after the ABCs (airway, breathing, circulation) are completed.

A patient complains of a decrease in the sense of smell. What could be the possible reasons for this symptom? Select all that apply. Correct 1 Heavy smoking Correct 2 Basilar skull fracture 3 Damage to vagus nerve 4 Damage to the trochlear nerve 5 Damage to glossopharyngeal nerve

Chemicals in cigarette smoke may damage the olfactory receptor cells, thus affecting the ability to smell. The basilar skull fracture may damage the olfactory fibers as they pass through the delicate cribriform plate of the skull, thus affecting the ability to smell. The vagus nerve has sensory functions on the viscera of the thorax and abdomen. The trochlear nerve is associated with eye movement. The glossopharyngeal nerve is associated with taste sensation and motor activity of the superior pharyngeal muscles. Text Reference - p. 1346 1,2

Scientific Concepts/Neurology Which of the following neurotransmitters is decreased in early Parkinson's disease? A. Dopamine B. Serotonin C. Norepinephrine D. Acetylcholine

Explanations (c) A. Decreased dopamine levels are typical of Parkinson's disease even early in the course of the disease. (u) B. See A for explanation. (u) C. See A for explanation. (u) D. See A for explanation.

A patient presenting with stroke symptoms is being considered for fibrinolytic therapy. What assessment data would be important to communicate promptly to the prescribing health care provider? 1 History of transient ischemic attack (TIA) six months ago. 2 Presence of indwelling urinary catheter. 3 Removal of soft tissue tumor from back three weeks ago. 4 Colonoscopy for evaluation of blood in the stools one week ago.

Contraindications for fibrinolytic therapy include recent history of gastrointestinal bleeding, stroke, or head trauma within three months or major surgery within 14 days. It would be most important for the nurse to inform the prescriber of the history of blood in the stools. Patients who have experienced a TIA are at greater risk of having a stroke. A history of TIA or presence of an indwelling urinary catheter are not contraindications for fibrinolytic therapy. Removal of a soft tissue tumor from the back does not fall in the category of major surgery, and this patient's surgery is beyond the 14 day window. Text Reference - p. 1398 4

A patient reports, "While I was walking I got something in my eye." What nursing intervention is most appropriate for a patient with a suspected foreign object in the eye? 1 Beginning irrigation with sterile normal saline solution 2 Attempting to remove the object without causing further damage to the eye 3 Refraining from doing anything until the patient can be seen by an ophthalmologist 4 Loosely covering the eye with a sterile patch and referring the patient to emergency care

Covering the eye loosely with a sterile patch with referral for emergency care is the safest option for this patient. Eye irrigation and attempting to remove the object are not appropriate in this healthcare setting. The nurse should never attempt to remove a foreign object from the eye, because this could cause further damage. The patient should be seen by the an eye specialist, but covering the eye with an eye patch will prevent further trauma and irritation. Text Reference - p. 390 4

A nurse is caring for a patient who becomes agitated in the evening. What nursing interventions are most helpful? Select all that apply. 1 Creating a calm environment 2 Allowing the patient to sleep for long hours 3 Isolating the patient to minimize stimuli 4 Limiting caffeine intake 5 Consulting with the health care provider

Creating a calm environment reduces agitation in the patient; restraining or threatening the patient can worsen the problem. Caffeine is a stimulant, so limiting caffeine intake would help in reducing agitation. Healthcare providers should be consulted if antianxiety drugs or sedatives need to be prescribed. Do not allow the patient to sleep for long hours at night, and limit daytime naps. Maximize the exposure to daylight rather than isolating the patient. Text Reference - p. 1454 1,4,5

The health care provider prescribes these interventions for a patient with possible botulism poisoning. Which one will the nurse question?

Give magnesium citrate 8 oz now. rational: Magnesium is contraindicated because it may worsen the neuromuscular blockade. The other orders are appropriate for the patient.

what is the most prominent symptom of MYASTHENIA GRAVIS?

DROOPING EYELIDS

What is the action of hyper osmotic agents (osmotic diuretics) used to treat ICP?

Dehydrate the brain and reduce cerebral edema by holding water in the renal tubules to prevent reabsorption and by drawing fluid from the extravascular spaces into the plasma

HESI HINT: What is multiple sclerosis?

Demyelinating dz resulitng in destruction of CNS myelin and consequent disruption in the transmission of nerve impulses

what medication should the nurse have in emergency for status epilepticus patient?

Diazepam

HESI HINT: What is dysarthria?

Difficulty articulating

A patient with a stroke experiences right-sided arm and leg paralysis and facial drooping on the right side. When admitting the patient, which clinical manifestation will the nurse expect to find?

Difficulty in understanding commands rational: Right-sided paralysis indicates a left-brain stroke, which will lead to difficulty with comprehension and use of language. The left-side reflexes are likely to be intact. Impulsive behavior and neglect are more likely with a right-side stroke.

A patient who has right-sided weakness after a stroke is attempting to use the left hand for feeding and other activities. The patient's wife insists on feeding and dressing him, telling the nurse, "I just don't like to see him struggle." Which nursing diagnosis is most appropriate for the patient?

Disabled family coping related to inadequate understanding by patient's spouse rational: The information supports the diagnosis of disabled family coping because the wife does not understand the rehabilitation program. There are no data supporting low self-esteem, and the patient is attempting independence. The data do not support an interruption in family processes because this may be a typical pattern for the couple. There is no indication that the patient has impaired nutrition.

A patient tells the nurse about using acetaminophen (Tylenol) several times every day for recurrent bilateral headaches. Which action will the nurse plan to take first?

Discuss the need to stop taking the acetaminophen. rational: The headache description suggests that the patient is experiencing medication overuse headache. The initial action will be withdrawal of the medication. The other actions may be needed if headaches persist.

HESI HINT: What is an important aspect treatment for Parkinsons?

Drug therapy Pathophysiology of Parkinsons involves an imbalance bw acetylcholine and dopamine, so symptoms can be controlled by admin a dopamine precursor (levodopa)

HESI HINT: What is dysphagia?

Dysfunctional swallowing

A patient who has bacterial meningitis is disoriented and anxious. Which nursing action will be included in the plan of care?

Encourage family members to remain at the bedside. rational: Patients with meningitis and disorientation will be calmed by the presence of someone familiar at the bedside. Restraints should be avoided because they increase agitation and anxiety. The patient requires frequent assessment for complications; the use of touch and a soothing voice will decrease anxiety for most patients. The patient will have photophobia, so the light should be dim.

a patient with multiple sclerosis is having an acute exacerbation after being without symptoms for 6 months. What can the nurse do to help the patient deal with personal fears at this time?

Encourage the patient to verbalize feelings

What is the most important principle in a bowel management program for a client with a neurologic deficits?

Establishment of regularity

Which action should the nurse take when assessing a patient with trigeminal neuralgia?

Examine the mouth and teeth thoroughly. rational: Oral hygiene is frequently neglected because of fear of triggering facial pain. Having the patient clench the facial muscles will not be useful because the sensory branches of the nerve are affected by trigeminal neuralgia. Light touch and palpation may be triggers for pain and should be avoided.

A patient who has a history of a transient ischemic attack (TIA) has an order for aspirin 160 mg daily. When the nurse is administering the medications, the patient says, "I don't need the aspirin today. I don't have any aches or pains." Which action should the nurse take?

Explain that the aspirin is ordered to decrease stroke risk. rational: Aspirin is ordered to prevent stroke in patients who have experienced TIAs. Documentation of the patient's refusal to take the medication is an inadequate response by the nurse. There is no need to clarify the order with the health care provider. The aspirin is not ordered to prevent aches and pains.

Health Maintenance/Neurology Which of the following is a milestone usually achieved by a 15-month old infant? A. walks alone B. puts three words together C. feeds self well with spoon D. builds tower of seven cubes

Explanations (c) A. A 15-month-old infant should be able to walk alone. (u) B. The ability to put three words together, feed oneself well with a spoon and build a tower of seven cubes does not occur until 24 months. (u) C. See B for explanation. (u) D. See B for explanation.

Diagnostic Studies/Neurology A patient involved in a minor motor vehicle crash is brought to the emergency department by a family member who was riding in the car, but was unhurt. The family member states that the patient was unconscious for about 2 minutes, but seems "okay" now. What diagnostic procedure would be most helpful in assessing this patient? A. head CT scan B. lumbar puncture C. skull radiographs D. electroencephalogram

Explanations (c) A. A head CT scan would provide evidence of fractures and demonstrate intracranial hemorrhage and cerebral edema if present. (u) B. A lumbar puncture would not be immediately indicated, but if the patient worsens it might be a consideration. (u) C. Skull radiographs would only reveal skull fracture and not any potentially lethal intracranial injuries. (u) D. An electroencephalogram is not needed because there is no history of seizures.

Diagnosis/Neurology You are examining a patient with right-sided extremity weakness and left-sided weakness of the face. Where is the lesion? A. Brainstem B. Cerebral hemisphere C. Cerebellum D. Basal ganglia

Explanations (c) A. A patient with a unilateral sensory or weakness finding on one side of the body and contralateral finding of weakness or sensory loss of the face has a brainstem lesion. (u) B. Patients with cerebral hemisphere lesions may present with motor, sensory, visual or auditory findings depending on the lesion. (u) C. Patients with cerebellum lesions present with ataxia, intention tremor, and dysmetria. (u) D. Patients with basal ganglia lesions present with bradykinesia, akinesia, and loss of postural reflexes.

Scientific Concepts/Neurology Which of the following animals is the major cause of human rabies in the United States and, therefore, poses the highest risk? A. bats B. rabbits C. rodents D. dogs

Explanations (c) A. Bat rabies is found in practically every state and is the most common cause of human rabies in the U.S. (u) B. Rabbits, rodents, and dogs are uncommon causes of human rabies in the United States. (u) C. See B for explanation. (u) D. See B for explanation.

Diagnostic Studies/Neurology A 21 year-old male college student is admitted to the hospital with suspected meningitis. A lumbar puncture is performed. The results of the cerebrospinal fluid (CSF) analysis reveals an elevated white blood cell count of 5,000/mcL with over 90% neutrophils, a decreased glucose level of 35 mg/dL, and elevated protein level of 150 mg/dL. What is the most likely diagnosis based on these results? A. Bacterial meningitis B. Viral meningitis C. Fungal meningitis D. Tuberculous meningitis

Explanations (c) A. CSF results with bacterial meningitis reveal an elevated white count with predominance of neutrophils, a low glucose, and an elevated protein level. (u) B. CSF values in patients with viral meningitis are lymphocytic pleocytosis with normal glucose and normal or slightly elevated protein. (u) C. CSF findings in fungal meningitis include lymphocytic pleocytosis, elevated protein, and decreased glucose. (u) D. CSF findings with TB meningitis reveals elevated pressure, lymphocytic pleocytosis, elevated protein, and decreased glucose. Ref: (1)

Clinical Therapeutics/Neurology Cognitive loss in Alzheimer's dementia may be delayed with which of the following medications? A. donepezil (Aricept) B. haloperidol (Haldol) C. risperidone (Risperdal) D. zolpidem (Ambien)

Explanations (c) A. Donepezil is a reversible cholinesterase inhibitor that leads to increased acetylcholine, which is necessary for learning and memory. (u) B. Haloperidol, risperidone, and other antipsychotics may be used to treat the agitation and behavioral symptoms in patients with dementia, but have significant side effects. (u) C. See B for explanation. (u) D. Zolpidem is used to treat insomnia.

Diagnostic Studies/Neurology A 2 month-old infant has had a single, generalized tonic-clonic convulsion lasting 4 to 5 minutes. There is no history of trauma and the infant had been well previously. Physical findings include a temperature of 39.6 degrees C (103.2 degrees F), a bulging tympanic membrane on the right, and an inflamed pharynx. The next most appropriate step is to A. perform a lumbar puncture. B. order x-ray studies of the skull. C. obtain an electroencephalogram. D. send home with antibiotics and an anticonvulsant.

Explanations (c) A. Febrile convulsions are uncommon under the age of 3 months. The physical findings suggest the possibility of meningitis, so lumbar puncture with CSF analysis is indicated. (u) B. See A for explanation. (u) C. See A for explanation. (h) D. See A for explanation. In addition, anticonvulsant therapy is not indicated for febrile seizures.

Scientific Concepts/Neurology The source of pain experienced during a migraine headache is a result of activation of which nerve? A. Trigeminal B. Vagus C. Optic D. Occulomotor

Explanations (c) A. Headache may result in release of neuropeptides acting as neurotransmitters at trigeminal nerve branches. (u) B. See A for explanation. (u) C. See A for explanation. (u) D. See A for explanation.

Diagnostic Studies/Neurology A 27 year-old female presents to your office for evaluation of weakness, visual loss, and sensory loss over the right great toe. These symptoms have occurred during three episodes approximately three months apart with each episode lasting about three days. Which of the following tests would be most useful in further evaluating this patient? A. MRI of the brain B. Electromyograph C. Glucose tolerance test D. Electroencephalograph

Explanations (c) A. Multiple sclerosis typically presents with relapsing weakness of the limbs, sensory loss, paresthesias, and visual changes. Diagnosis is based on history and either abnormal brain or spinal cord MRI, or visual, auditory, or somatosensory evoked electrical response. (u) B. See A for explanation. (u) C. Glucose tolerance test is used in the evaluation of diabetes mellitus. (u) D. Electroencephalograph is used to evaluate patients for possible seizure disorder.

52. Health Maintenance/Neurology What is the recommendation for primary prevention of stroke in a patient under sixty years of age with atrial fibrillation? A. No therapy is needed B. Aspirin C. Warfarin D. Maze procedure

Explanations (c) A. No therapy is recommended for primary stroke prevention in this patient. (u) B. See A for explanation. (u) C. See A for explanation. (u) D. See A for explanation.

Clinical Therapeutics/Neurology A 22 year-old female presents with onset of right eye vision loss 24 hours ago which has partially improved. She has experienced various widespread paresthesias, regional weakness and fatigue over the past six months. Episodes vary in location, severity and duration but invariably lead to recovery. Her exam is completely normal but MRI reveals numerous areas of periventricular gadolinium uptake. For this condition, what treatment has shown the greatest benefit with long term use? A. Interferon beta 1a (Avonex) B. Ethosuximide (Zarontin) C. Methylprednisolone (Solumedrol) D. Pyridostigmine (Mestinon)

Explanations (c) A. The interferon agents have been the longest used and best studied medications that provide the best long term benefits in multiple sclerosis. Steroids do play a role in acute exacerbations but not as long term agents. (u) B. See A for explanation. (u) C. See A for explanation. u) D. See A for explanation.

Diagnosis/Neurology A 28 year-old female presents to the clinic complaining of a "prickly sensation" that started bilaterally in her feet two days ago and difficulty walking. She now has the dysesthesia from her mid-thigh down to her toes. On physical examination she has diminished pain and temperature sensation, absent reflexes, loss of proprioception in her legs bilaterally, and muscle strength is 1+/5+ in the lower extremities and 5+/5+ in the upper extremities. What is the most likely diagnosis? A. Guillain-Barré syndrome B. Multiple sclerosis C. Myasthenia gravis D. Spinal cord compression

Explanations (c) A. The pattern of sensory, motor and reflex findings is consistent with the pathophysiology of peripheral nerve demyelination that occurs in Guillain-Barré syndrome. (u) B. Multiple sclerosis does not present as a symmetrical ascending paralysis. (u) C. Patients with myasthenia gravis tend to have intermittent symptoms that affect proximal and extraocular muscles most notably and it also lacks sensory involvement. (u) D. Although the exact type of cord transection can alter the pattern of motor and sensory findings a patient with spinal cord compression who is not in spinal shock would have hyperreflexia instead of areflexia.

Clinical Therapeutics/Neurology A 50 year-old female with a history of coronary artery disease presents to the office requesting medication for an exacerbation of her migraine headaches. She complains of migraines approximately once a month. What medication is contraindicated in this patient? A. Sumatriptan (Imitrex) B. Propranolol (Inderal) C. Droperidol(Inapsine) D. Naproxensodium(Anaprox)

Explanations (c) A. The triptans are contraindicated in patients with coronary artery disease or peripheral vascular disease and should be avoided in patients with an increased risk for stroke. All other drugs listed are not contraindicated. (u) B. See A for explanation. (u) C. See A for explanation. (u) D. See A for explanation.

Clinical Therapeutics/Neurology A 37 year-old female presents to the clinic for follow-up regarding her recently diagnosed tonic-clonic epilepsy. She reports no seizures or side effects since starting valproate (Depakote) at her last visit one month ago. What diagnostic study would you order to monitor this patient's treatment? Answers A. Serum amylase B. Serum creatinine C. Liver function tests D. Electroencephalogram

Explanations (u) A. See C for explanation. (u) B. See C for explanation. (c) C. Valproate may be toxic to the liver as well as cause thrombocytopenia. (u) D. See C for explanation.

Clinical Intervention/Neurology A 50 year-old male presents to the ER with a complaint of severe headaches. These headaches are unilateral and he describes the headache pain as steady and non-throbbing. He also complains of nasal congestion and rhinorrhea. He also mentions that alcohol often triggers these headaches. What do you recommend for this patient? A. High flow oxygen B. Massage C. Ibuprofen D. Propranolol

Explanations (c) A. This a description of a cluster headache (migrainous neuralgia), these headaches respond to oxygen by mask 7-10 L/min for 15 minutes. (u) B. Massage may be effective for patients with tension headaches. (u) C. Ibuprofen is helpful for the treatment of tension type headaches. (u) D. Propranolol is a preventative medication used for migraine headaches.

Diagnosis/Neurology A 55 year-old male presents with a three month history of progressive weakness in the extremities without associated sensory changes or deficits. Examination reveals widespread muscle atrophy, fasciculations and bilateral hyperactive reflexes with Babinski sign. Which of the following is the most likely diagnosis? A. Amyotrophic lateral sclerosis B. Polymyalgia rheumatica C. Myasthenia gravis D. Multiple sclerosis

Explanations (c) A. Though some variability in presentation does occur the characteristic progressive weakness without sensory changes and upper and lower motor nerve dysfunction is a hallmark of amyotrophic lateral sclerosis (ALS). (u) B. See A for explanation. (u) C. See A for explanation. (u) D. See A for explanation.

Diagnostic Studies/Neurology A 37 year-old right-handed male presents with daily headaches for the past 2 months. They are worse in the morning and awaken him from sleep. In the past week, he has noticed a tendency to drop things. Neurologic examination reveals upper extremity strength of 4/5 on the right and 5/5 on the left. Which of the following is the most appropriate next step in the evaluation of this patient? A. Lumbar puncture (LP) B. Electroencephalogram (EEG) C. Magnetic resonance imaging (MRI) D. Electromyelogram (EMG)

Explanations (h) A. Lumbar puncture results would give information on the cerebrospinal fluid. In the presence of focal neurologic findings, a lumbar puncture would be contraindicated prior to an MRI. (u) B. An EEG is used in the evaluation of seizure disorders. (c) C. MRI is the most appropriate diagnostic study in the evaluation of a suspected intracranial neoplasm. (u) D. EMG is not indicated in the evaluation of a suspected intracranial neoplasm.

Clinical Intervention/Neurology A 43 year-old data entry clerk presents with a one-month history of pain and tingling in the right thumb, index finger, and middle finger. Tinel's sign and Phalen's maneuver are positive. The most appropriate intervention at this time is A. methylprednisolone (Medrol) dose pack. B. splint in neutral position. C. observation. D. surgery.

Explanations (u) A. A Medrol dose pack will have no affect on carpal tunnel syndrome. (c) B. Splinting in neutral position relieves impingement of the median nerve, thus improving symptoms of carpal tunnel. (u) C. Observation will not improve symptoms. (u) D. Surgical intervention is reserved for cases unresponsive to conservative therapy.

HESI HINT: What is glaucoma? What are its symptoms like?

Glaucoma is a condition characterized by increased intraocular pressure (IOP) - Typically painless and symptom free - Can lead to blindness if untreated - More at risk if have family hx - Cannot be cured, but can treated with medication and surgery - Can be a side effect of many medications (antihistamines, anticholinergics)

Diagnosis/Neurology During an influenza epidemic, a 6 year-old child is seen with fever and a severe sore throat. A throat swab is taken for culture and the child is sent home. The next day, he is reported to have persistent vomiting and increased lethargy. On examination, he is delirious and disoriented. No rash is noted. His reflexes are hyperactive. The liver edge is 3 cm below the right costal margin in the midclavicular line. Which of the following is the most likely diagnosis? A. acute bacterial meningitis B. Guillain Barré syndrome C. Reye syndrome D. measles encephalitis

Explanations (u) A. Acute bacterial meningitis presents with fever, malaise, and neck stiffness. The liver is typically normal. (u) B. Guillain Barré syndrome typically presents post-Campylobacter enteritis. Signs and symptoms include an ascending weakness. No hepatomegaly is noted and reflexes are diminished. (c) C. Reye syndrome is typically post-influenza or URI. The patient develops lethargy, drowsiness, and vomiting. Babinski reflex is positive and hyperreflexia is noted. The liver is normal or enlarged. (u) D. Measles encephalitis typically presents one week after the measles rash with ataxia, vomiting, and seizures.

Clinical Therapeutics/Neurology A 70 year-old presents with headache and neck stiffness. On physical exam, the patient is febrile, Kernig's sign is present, and no rash is noted. A spinal tap reveals a white count of 250/cm3 with 100% neutrophils, total protein 250 mg/dL, and glucose 35 mg/dL. Which of the following is the most appropriate treatment? A. Acyclovir (Zovirax) B. Fluconazole (Diflucan) C. Ampicillin and ceftriaxone (Rocephin) D. Penicillin and chloramphenicol (Chloromycetin)

Explanations (u) A. Acyclovir is used to treat meningitis secondary to herpes. Viral meningitis presents with increase number of lymphocytes and elevated glucose in the CSF . (u) B. Fluconazole is used to treat fungal meningitis. Fungal meningitis, typically noted in immunocompromised hosts, presents with increase number of lymphocytes in the CSF. (c) C. Ampicillin and ceftriaxone is used to treat bacterial meningitis, secondary to Listeria monocytogenes , which is common in the elderly. Ceftriaxone will cover other common etiologic agents such as Streptococcal pneumonia (u) D. Penicillin and chloramphenicol is used to treat bacterial meningitis, secondary to Neisseria meningitidis. Bacterial meningitis due to N

Clinical Intervention/Neurology Which of the following interventions is most effective during the early stages of Alzheimer's disease? Answers A. Frequent change of caregivers in the home B. Utilization of memory aids, such as post-it notes C. Encouragement of independent activities, such as driving D. Emphasis of new learning activities, such as computer training

Explanations (u) A. Although caregiver burnout is often encountered, consistency is beneficial to a patient having difficulty with confusion. (c) B. Memory aids are extremely helpful in assisting Alzheimer patients during the early stages of the disease. (u) C. Like our shuttle driver demonstrated, most patients should consider relinquishing their license. (u) D. Pleasant activities should be emphasized, while learning new activities may be a source of frustration.

Diagnostic Studies/Neurology A 45 year-old female presents complaining of the worst headache of her life. Which of the following is the most appropriate initial diagnostic study? A. Magnetic resonance imaging (MRI) B. Computed tomography (CT) C. Electroencephalogram (EEG) D. Lumbar puncture (LP)

Explanations (u) A. An MRI is not as sensitive in detecting an acute cerebral bleed as a CT scan. (c) B. A CT scan is more sensitive in detecting cerebral hemorrhage in the first 24 to 48 hours. (u) C. Electroencephalogram (EEG) is used in the evaluation of seizures and will not assess the presence of cerebral hemorrhage. (u) D. A lumbar puncture is indicated in evaluation of suspected subarachnoid hemorrhage only if the CT scan does not establish the diagnosis.

Diagnostic Studies/Neurology A 53 year-old female with sudden onset "of the worst headache she has ever had" presents to the emergency department. She has a history of migraines but states that the current headache is not like her usual headaches. Results of her physical examination are unremarkable. Which of the following is the next best step in the evaluation of this patient? A. Angiogram B. CT scan C. Transcranial Doppler D. MRI

Explanations (u) A. Angiogram is necessary to define details of aneurysm and anatomic configuration, but not as an initial diagnostic study. (c) B. This patient's history is highly suggestive of subarachnoid hemorrhage. CT is best to screen for intracranial hemorrhage. It is faster than MRI and more sensitive in the first 24 hours. (u) C. Transcranial Doppler can detect cerebral artery vasospasm but cannot detect aneurysm. (u) D. MRI is not as sensitive for an acute bleed, but is appropriate for old bleeds.

Clinical Intervention/Neurology A 67 year-old female with history of hypertension, diabetes mellitus, and smoking presents to the emergency department with mild expressive aphasia, right facial weakness and mild right arm weakness. She had awakened 60 minutes ago and was speaking to her husband when her speech suddenly became difficult to understand and weakness was noted. Physical examination reveals a blood pressure of 165/85 mm Hg. A CT of the head shows no intracranial hemorrhage. Which of the following is the most appropriate intervention? A. Aspirin B. Warfarin (Coumadin) C. Tissue plasminogen activator (rt-PA) D. Clopidogrel (Plavix)

Explanations (u) A. Aspirin is used for preventative purposes and will not resolve a current embolism. (u) B. Warfarin is a preventative medicine that will not help the current situation. (c) C. IV thrombolytic therapy with recombinant tissue plasminogen activator is effective in reducing the neurological deficit in selected patient without CT evidence of intracranial hemorrhage and when administered within 3 hours after onset of ischemic stroke. (u) D. Plavix is a platelet aggregation inhibitor and used for preventative measures.

Clinical Therapeutics/Neurology Which of the following side effects is associated with long-term administration of phenytoin (Dilantin)? A. Ataxia B. Hypotension C. Osteomalacia D. Cardiac dysrhythmia

Explanations (u) A. Ataxia is associated most often with acute oral overdosage of phenytoin. (u) B. Cardiac dysrhythmia, with or without hypotension, is an expected side effect of rapid IV phenytoin administration. (c) C. Osteomalacia, or demineralization of bone, is a side effect of phenytoin that may occur after chronic administration. (u) D. See B for explanation.

Clinical Therapeutics/Neurology A 53 year-old woman recently diagnosed with Parkinson's disease has a moderate tremor. She has no other signs of disease at this time. Which of the following is the most appropriate initial treatment? A. lorazepam (Ativan) B. haloperidol (Haldol) C. ramatidine (Flumadine) D. levodopa/carbidopa (Sinemet)

Explanations (u) A. Benzodiazepines are used to treat anxiety and insomnia, but are not used to treat Parkinson's disease. (u) B. Haloperidol is used for the treatment of Tourette's syndrome, but is not indicated for the treatment of Parkinson's disease. (u) C. Ramatidine is used in the treatment of viral infections and has no role in the treatment of Parkinson's disease. (c) D. Levodopa/carbidopa is first-line therapy for the treatment of Parkinson's disease.

Clinical Therapeutics/Neurology A 28 year-old female presents with complaint of headaches for the past several months. They typically begin behind her right eye, which she describes as a combination of stabbing and pressure. She states that noise bothers her, and she experiences nausea but no vomiting. She has occasionally awakened with the pain. She admits to being under a lot of stress. Aspirin or acetaminophen does not relieve the pain. Which of the following is the best choice for initial treatment of the acute headache? A. topiramate (Topamax) B. gabapentin (Neurontin) C. propranolol (Inderal) D. sumatriptan (Imitrex)

Explanations (u) A. Beta blockers, calcium channel blockers, antiepileptic drugs, and tricyclic antidepressants have been used for the prevention of migraine headaches, but not for the acute treatment. (u) B. See A for explanation. (u) C. See A for explanation. (c) D. 5HT-receptor agonists and ergot alkaloids are effective for the acute treatment of migraine headaches if acetaminophen or nonsteroidal anti-inflammatory drugs are not effective.

Diagnosis/Neurology A 54 year-old male smoker presents to the clinic complaining of frequent vague headaches with associated vomiting that awaken him from sleep occasionally and have been present upon awakening for about two weeks. The headache typically resolves about an hour into his morning routine. The patient is afebrile. What is the most likely cause of this patient's headaches? Answers A. Cluster headaches B. Depression C. Glioblastoma D. Giant cell arteritis

Explanations (u) A. Cluster headaches can awaken patients, but are not usually "vague". (u) B. See C for explanation. (c) C. Morning headaches associated with vomiting are indicative of increased intracranial pressure and raise concern of a CNS tumor such as a glioblastoma. (u) D. Giant cell arteritis presents in the older patient with headache in the temporal region and loss of vision.

Diagnosis/Neurology A 30 year-old female presents to the office complaining of generalized weakness and reduced exercise tolerance that improves with rest. On physical examination you note the presence of bilateral eyelid ptosis, proximal muscle weakness and normal reflexes. What is the most likely diagnosis? A. Lambert-Eaton syndrome B. Organophosphate intoxication C. Multiple sclerosis D. Myasthenia gravis

Explanations (u) A. Common symptoms of Lambert-Eaton syndrome are proximal muscle weakness of lower limbs, cranial nerve findings, and depressed or absent reflexes. Patients commonly have a malignancy. (u) B. Patients with organophosphate intoxication have seizures, excessive secretions, wheezing and diaphoresis. (u) C. Patients with multiple sclerosis have multiple lesions in time and space. (c) D.Common symptoms of myasthenia gravis are fatigable weakness, ptosis, diplopia, and proximal muscle weakness. The disease is more common in women in the 2nd and 3rd decade and in men older than 60.

History & Physical/Neurology What spinal nerve root is most likely affected in a patient with weak wrist extension, thumb and index finger paresthesias and diminished triceps reflex? A. Cervical 4 B. Cervical 5 C. Cervical 6 D. Cervical 7

Explanations (u) A. See C for explanation. (u) B. See C for explanation. (c) C. In contrast, cervical 5 would be associated with deltoid and biceps weakness and diminished biceps reflex while cervical 7 would result in triceps weakness and paresthesias in the middle finger and diminished brachioradialis reflex. (u) D. See C for explanation.

Diagnostic Studies/Neurology A 12 month-old in the emergency department is diagnosed with possible viral meningitis. Which of the following cerebral spinal fluid (CSF) laboratory results is most consistent with this diagnosis? A. Decreased CSF glucose level and increased protein B. Decreased CSF total protein level and very few neutrophils C. Increased CSF mononuclear cells and normal glucose D. Increased CSF C-reactive protein and normal glucose

Explanations (u) A. Decreased CSF glucose level and increased protein is consistent with bacterial meningitis. (u) B. Decreased CSF total protein level and very few neutrophils is a finding in syphilitic meningitis. (c) C. In aseptic meningitis, CSF shows mainly mononuclear cells within 6-8 hours, glucose is normal and there is normal to lower protein. (u) D. Increased CSF C-reactive protein and normal glucose is found in any inflammatory process affecting the CSF.

History & Physical/Neurology Which of the following is the most common finding of multiple sclerosis? A. facial palsy B. hearing loss C. seizures D. diplopia

Explanations (u) A. Facial palsy, hearing loss, and seizures are rare with multiple sclerosis. (u) B. See A for explanation. (u) C. See A for explanation. (c) D. Diplopia due to ophthalmoplegia is the most common presenting complaint in patients with multiple sclerosis.

History & Physical/Neurology A patient with an upper motor neuron lesion would exhibit which of the following findings? Answers A. Fasciculations B. Areflexia C. Muscular atrophy D. Spasticity

Explanations (u) A. Fasciculations, areflexia and muscle atrophy are consistent with lower motor neuron lesions. (u) B. See A for explanations. (u) C. See A for explanation. (c) D. Spasticity is an upper motor neuron lesion finding.

Diagnosis/Neurology A 37 year-old male presents with headaches for the past 2 months. They occur daily and are worse in the morning. In the past week, he has noticed a tendency to drop things from his right hand. On examination, vital signs are normal, and general examination is unremarkable. Neurologic examination reveals mild weakness of the right upper extremity compared to the left. Which of the following is the most likely diagnosis? A. focal seizure disorder B. intracerebral neoplasm C. transient ischemic attack D. amyotrophic lateral sclerosis

Explanations (u) A. Focal seizure disorder is not associated with morning headaches. (c) B. Intracerebral neoplasms may present with headaches that are worse in the morning, with improvement during the day. Focal motor or sensory loss depends upon the tumor's location. (u) C. A TIA is characterized by focal findings that resolve completely and spontaneously within 24 hours. (u) D. Amyotrophic lateral sclerosis is a degenerative disorder characterized by motor weakness, but is not associated with morning headaches.

Diagnosis/Neurology A 72 year-old patient with a history of hypertension and atrial fibrillation presents with episodes of weakness, numbness, and paresthesias in the right arm. At the same time, she notes speech difficulty and loss of vision in her left eye. These symptoms come on abruptly and clear within minutes. Physical examination is normal except for the previously known arrhythmia. Which of the following is the most likely diagnosis? A. Focal seizure B. Migraine headache C. Hypoglycemic episodes D. Transient ischemic attack

Explanations (u) A. Focal seizures usually cause abnormal motor movement rather than weakness or loss of feeling. (u) B. Patients with migraines commonly have a history of episodes since adolescence. (u) C. Hypoglycemic episodes do not present with focal neurological findings. (c) D. This patient's symptoms are consistent with transient ischemia in the carotid territory. Atrial fibrillation is a risk factor for cerebral emboli.

Diagnosis/Neurology A 32 year-old female presents with a seven month history of recurrent, brief episodes of weakness and tingling in the extremities, diplopia, and vertigo. Which of the following is the most likely diagnosis? A. Guillain-Barre syndrome B. Myasthenia gravis C. Multiple sclerosis D. Amyotrophic lateral sclerosis

Explanations (u) A. Guillain-Barre syndrome typically presents with progressive weakness that starts peripherally and proceeds centrally over a brief period of time. (u) B. Myasthenia gravis presents with weakness and fatigue in the upper limbs, cranial, and/or trunk musculature. Blurry vision and diplopia are common visual complaints and dysphagia is common. (c) C. Multiple sclerosis is most frequently seen in patients in their twenties and presents with episodes of weakness, paresthesias, and diplopia. (u) D. Amyotrophic lateral sclerosis presents with progressive weakness, fasciculations, and loss of muscle mass.

Clinical Intervention/Neurology A 73 year-old female patient was diagnosed 3 years ago with Alzheimer's disease and heart failure. Her 80 year-old husband can no longer help feed and bathe her or manage her medications. Which of the following support services is most appropriate for this patient? A. hospice care B. senior center C. adult day care D. skilled nursing facility

Explanations (u) A. Hospice care is a program to provide palliative care to individuals who are terminally ill and projected to die within 6 months. (u) B. Senior centers are community-based facilities that provide recreational activities and mid-day meals for functional adults. (u) C. Adult day health care is designed to meet the needs of functionally impaired adults in a community setting, but does not usually provide for the higher level of care that is required to feed or bathe a patient. (c) D. A skilled nursing facility is appropriate for patients requiring assistance with activities of daily living (i.e., feeding and bathing) and a higher level of safety assurance.

Diagnosis/Neurology A 37 year-old male presents with daytime fatigue and drowsiness. He states that he does not sleep well, with frequent awakenings during the night. He has gained 8 pounds over the past six months, and he complains of palpitations. His wife states that he snores at night. A home nocturnal pulse oximetry indicates that his saturation drops 6% intermittently throughout the night. Which of the following is the most likely diagnosis? A. depression B. narcolepsy C. hypothyroidism D. obstructive sleep apnea

Explanations (u) A. Hypothyroidism, narcolepsy, and depression are not associated with oxyhemoglobin desaturation or snoring. (u) B. See A for Explanation. (u) C. See A for Explanation. (c) D. Obstructive sleep apnea is associated with obesity, nighttime wakening and snoring, hypertension, cardiac dysrhythmias, and oxyhemoglobin desaturation of greater than 4% during hypopnea or apnea.

Clinical Therapeutics/Neurology A 38 year-old female has a 10 year diagnosis of migraine headaches. She had been using ergotamine to abort her headaches, but is now having one or two headaches per week that are interfering with work. Which of the following is the most appropriate preventive therapy? A. Sumatriptan (Imitrex) B. Promethazine (Phenergan) C. Propranolol (Inderal) D. Ketorolac (Toradol)

Explanations (u) A. Imitrex is not approved for preventive therapy but is used as abortive therapy. (u) B. Phenergan is an antiemetic that helps with nausea and vomiting for people who get migraines. It is not a preventive medication. (c) C. Propanolol is one beta blocker that is frequently used as a first-line prophylaxis for migraines. (u) D. Ketorolac is not indicated for scheduled, daily use as a preventive for migraines.

History & Physical/Neurology Upon stroking of the lateral aspect of the sole from the heel to the ball of the foot, the great toe dorsiflexes and the other toes fan. This is a positive A. Kernig's sign. B. Brudzinski's sign. C. Babinski's sign. D. Gower's sign.

Explanations (u) A. Kernig's sign is positive when pain is noted on straightening the knee after flexing both the hip and knee. (u) B. Brudzinski's sign occurs with neck flexion resulting in resultant flexion of the hips. It is a sign of meningeal irritation. (c) C. A Babinski test is performed by stroking the lateral aspect of the sole from the heel to the ball of the foot, the great toe dorsiflexes and the other toes fan in a positive test. (u) D. A positive Gower's sign is noted in certain types of muscular dystrophy and is described as children rising to stand by rolling over prone and pushing off the floor with arms while the legs remain extended.

Clinical Intervention/Neurology A 45 year-old male patient presents to the office with a complaint of dizziness and vertigo that occurs suddenly when he rolls out of bed in the morning. He denies previous illness or any medical problems. On physical exam you note the presence of lateral nystagmus after a few second latency period. What is your recommendation for this patient? A. Low dose diazepam B. Repositioning maneuvers C. Diuretics and a low salt diet D. MRI of the brain

Explanations (u) A. Low dose diazepam is used to treat acute vertigo. (c) B. This patient has benign paroxysmal positional vertigo (BPPV) and repositioning maneuvers are recommended to move endolymphatic debris out of the posterior semicircular canal. (u) C. Diuretics and a low salt diet are used to treat Meniere's disease. (u) D. An MRI of the brain should be ordered in a patient in whom a central etiology of vertigo is suspected.

Clinical Therapeutics/Neurology Which of the following is first-line treatment for acute inflammatory demyelinating polyneuropathy variant of Guillain- Barre? A. Neostigmine B. IV Solu-Medrol C. Phenytoin D. IV immunoglobulin

Explanations (u) A. Neostigmine may be used for a more chronic variant for symptoms, but not in acute inflammatory variant. (u) B. Treatment with corticosteroids is ineffective and may prolong recovery time. (u) C. Phenytoin may be used for ongoing neuropathic pain, but is not useful in any type of acute neuropathy. (c) D. IV immunoglobulin is effective in patients with Guillain-Barre.

History & Physical/Neurology The most frequent finding in a person presenting with a brain abscess is A. nuchal rigidity. B. headache. C. seizures. D. vomiting.

Explanations (u) A. Nuchal rigidity occurs in approximately 35% of patients with a brain abscess. (c) B. Headache occurs in over 70% of patients with a brain abscess. (u) C. Seizures occur in approximately 35% of patients with a brain abscess. (u) D. Vomiting occurs in approximately 35% of patients with a brain abscess.

Diagnosis/Neurology A 70 year-old male is brought to the office by his wife. She is complaining that her husband has been having great difficulty remembering things, however remains alert. She also complains that he is having difficulty getting to the bathroom on time. On physical examination you notice that the patient's gait is wide-based and slow and he walks without lifting his feet off the floor. You also notice a tremor in his right hand. What is the most likely diagnosis? A. Lewy body dementia B. Parkinson's disease C. Normal pressure hydrocephalus D. Frontotemporal dementia

Explanations (u) A. Patients with Lewy body dementia have hallucination, Parkinsonism, fluctuating alertness, and falls. (c) B. Patients with Parkinson's disease have clinical features such as bradykinesia, rest tremor, rigidity, postural instability, autonomic dysfunction and behavioral changes. Of the diseases listed, Parkinson's is the most common. (u) C. Patients with normal pressure hydrocephalus have an abnormal gait (apraxic or ataxic), dementia, and urinary incontinence. (u) D. Patients with frontotemporal dementia have predominant behavioral symptoms, dementia, apraxia, Parkinsonism, and motor neuron disease.

History & Physical/Neurology A 75 year-old male presents to the ER with the following stroke findings: right-sided hemiparesis (face and hand more affected than leg), homonymous hemianopsia of the right half of both visual fields, and aphasia. Where is the location of his stroke? A. Anterior cerebral artery B. Middle cerebral artery C. Posterior cerebral artery D. Internal carotid artery

Explanations (u) A. Patients with anterior cerebral artery stroke will have findings greater in the legs than hands. (c) B. This case is a description of a middle cerebral artery stroke. (u) C. Posterior cerebral artery stroke patients will have midbrain and thalamic or sensory findings. (u) D. Internal carotid artery stroke patients will have amaurosis fugax, visual disturbances and crossed symptoms.

Clinical Therapeutics/Neurology A 72 year-old man with long-standing diabetes mellitus, renal insufficiency and hypertension presents to the clinic complaining of burning and tingling pain in his feet. What agent would you prescribe to help control his pain? Answers A. Phenobarbital B. Amitriptyline (Elavil) C. Celecoxib (Celebrex) D. Codeine

Explanations (u) A. Phenobarbital is used in the treatment of seizure disorder. (c) B. Amitriptyline is one of several preferred agents for management of peripheral neuropathy. (u) C. Although Cox-2 inhibitors may provide pain relief they are best avoided for treatment of this chronic disorder in a patient with renal disease. (h) D. It is best to avoid narcotics in the management of chronic pain if other efficacious medicines are available.

Clinical Therapeutics/Neurology What is the recommended treatment for absence (petit mal) seizures? A. Phenytoin (Dilantin) B. Carbamazepine (Tegretol) C. Ethosuximide (Zarontin) D. Gabapentin (Neurontin)

Explanations (u) A. Phenytoin is used to treat tonic clonic and partial seizures. (u) B. Carbamazepine is used to treat tonic clonic and partial seizures. (c) C. Ethosuximide, valproic acid, and clonazepam are recommended treatments for absence seizures. (u) D. Gabapentin is used to treat partial seizures.

Diagnosis/Neurology A patient describes a history of intermittent and uncontrollable twitching of his right hand that spreads to involve the entire arm after a few minutes. Afterward, the arm is extremely weak. There are no other areas of involvement, sensory deficit or altered consciousness. What is the most likely seizure diagnosis? A. Absence B. Complex-partial C. Simple-partial D. Myoclonic

Explanations (u) A. See C for explanation. (u) B. See C for explanation. (c) C. The lack of altered consciousness and focal motor symptoms are strongly indicative of this type of seizure. (u) D. See C for explanation.

Clinical Intervention/Neurology A 30 year-old male has a history of weakness without pain on the left side of his face for the past 4 days. Physical examination of the face reveals unilateral weakness to the left side, but not complete paralysis. The left eye does not close completely. The most appropriate initial treatment is to A. begin physical therapy. B. refer for surgical ablation. C. reassure patient and provide patient education. D. initiate high dose steroids and recheck in 24 hours.

Explanations (u) A. Physical therapy is not indicated for Bell's palsy (u) B. Surgery has not been shown to provide any significant benefit in the treatment of Bell's palsy. (c) C. Bell's palsy is usually a self-limited condition and typically resolves within a few weeks. (u) D. Corticosteroids may be used if paralysis is complete or there is significant pain, but are not first-line.

Clinical Therapeutics/Neurology What is the appropriate initial intravenous drug therapy for a patient in status epilepticus? Answers A. Midazolam (Versed) B. Lorazepam (Ativan) C. Fosphenytoin (Mesantoin) D. Phenobarbital

Explanations (u) A. See B for explanation. (c) B. A benzodiazepine (lorazepam) is first line in the treatment of status epilepticus, followed by phenytoin or fosphenytoin. (h) C. See B for explanation. (h) D. This answer is potentially harmful because the patient is currently seizing and lorazepam is indicated.

Clinical Therapeutics/Neurology A 7 year-old is evaluated for episodes of unresponsiveness in which she stares, blinking into space. Episodes last only 10-20 seconds but may happen several times daily. There are no abnormal movements, sensory loss or headaches reported. Which of the following is considered first-line therapy for this disorder? A. Carbamazepine (Tegretol) B. Ethosuximide (Zarontin) C. Gabepentin (Neurontin) D. Topiramate (Topamax)

Explanations (u) A. See B for explanation. (c) B. Absence seizures are usually treated successfully with Ethosuximide. The other agents listed are not indicated and may actually worsen this type of seizure. (u) C. See B for explanation. (u) D. See B for explanation.

Clinical Intervention/Neurology Treatment of Bell's palsy includes which of the following? A. Acyclovir B. Reassurance of the patient's recovery C. Referral to a neurosurgeon D. Electromyography

Explanations (u) A. See B for explanation. (c) B. Bell's palsy is a peripheral neuropathy of cranial nerve VII. Although it has been suggested it may be related to an activation of herpes simplex virus, there is little empiric evidence for this. Approximately 60% of cases of Bell's palsy recover without treatment and patient reassurance of this is advised. Electromyography may provide aid in the prognosis, but not as a treatment option. A neurosurgeon has no role in the management of Bell's palsy. (u) C. See B for explanation. (u) D. See B for explanation.

Scientific Concepts/Neurology A 58 year-old truck driver is found to have a positive Romberg test and loss of vibratory sensation in his lower extremities. What anatomical structure is the likely affected? Answers A. Cerebellum B. Posterior column C. Sensory cortex D. Vestibular apparatus

Explanations (u) A. See B for explanation. (c) B. Posterior column is responsible for vibratory sensation and proprioception. Romberg test is a general test of proprioception with disease of the cerebellum, vestibular apparatus or posterior column being the most likely source. (u) C. The bilateral lower extremity findings make a lesion of the sensory cortex unlikely. (u) D. See B for explanation.

History & Physical/Neurology A 64 year-old right-handed woman presents to the emergency room. The patient is pleasant and cooperative, but you note that the left side of her mouth has little movement as she talks, resulting in some dysarthric speech. On physical examination, the left side of her mouth droops, eyebrows raise symmetrically, frown is symmetric, and eyes close but left offers little resistance to opening. You suspect which of the following? A. Upper motor neuron damage to CN VII (central facial palsy) in left hemisphere B. Upper motor neuron damage to CN VII (central facial palsy) in right hemisphere C. Lower motor neuron damage to CN VII (Bell's palsy) on the left D. Lower motor neuron damage to CN VII (Bell's palsy) on the right

Explanations (u) A. See B for explanation. (c) B. The upper face is controlled by pathways from both sides of the face. Upper motor neuron lesions cross, findings on the left are due to lesions on the right. Lower motor neuron lesions of the face are ipsilateral. (u) C. See B for explanation. (u) D. See B for explanation.

History & Physical/Neurology What term is given to an ocular examination finding where small, irregular pupils are seen that react with near vision but not to light? A. Amaurosis fugax B. Argyll-Robertson C. Anisocoria D. Optic neuritis

Explanations (u) A. See B for explanation. (c) B. This finding may be seen in diabetes or in cases of neurosyphilis whereas amaurosis fugax is vision loss due to a central lesion, anisocoria has unequal sized pupils, unilateral loss of vision due to multiple sclerosis. (u) C. See B for explanation. (u) D. See B for explanation.

Diagnostic Studies/Neurology A 74 year-old female is being treated for mild hypertension. She is found at home with right hemiparesis and brought to the emergency department. Her daughter states that the patient fell in her kitchen 2 days ago, but had no complaints at that time. She did state that her mother sounded a little confused this morning. The patient's left pupil is dilated. Which of the following diagnostic studies should be ordered first? A. MRI of the brain B. CT scan of the brain C. Skull x-ray D. Lumbar puncture

Explanations (u) A. See B for explanation. (c) B. This patient presents with a history of minor trauma and progressive neurological abnormalities consistent with subdural hematoma. Diagnosis would be confirmed by CT scan, which is less expensive and more sensitive for blood than an MRI. (u) C. Skull x-rays would not be helpful because they evaluate bony, not soft tissue, injury. (h) D. A lumbar puncture is contraindicated because of the potential for brain herniation.

Health Maintenance/Neurology Household contacts of a patient with bacterial meningitis are best treated with which of the following? A. Amoxicillin (Amoxil) B. Ciprofloxin (Cipro) C. Tetracycline (Sumycin) D. Vancomycin (Vancocin)

Explanations (u) A. See B for explanation. (c) B. Rifampin, Cipro, Levaquin, Zithromax and Rocephin are the drugs of choice. (u) C. See B for explanation. (u) D. See B for explanation.

Clinical Therapeutics/Neurology A 27 year-old female presents with 5-6 headaches monthly for the past year that are described as severe, throbbing and initially unilateral. They is associated with nausea, photophobia and phonophobia. There have been no concomitant sensory or motor deficits. Her physical examination and brain MRI are normal. What treatment would be best to reduce the frequency of the headaches? A. Sumatriptan (Imitrex) B. Isometheptene (Midrin) C. Propranolol (Inderal) D. Hydrocodone (Vicodin)

Explanations (u) A. See C for explanation. (u) B. See C for explanation. (c) C. Each medication listed can be used in the management of migraine headaches, however only propranolol is indicated for reducing the frequency of migraine headaches. (u) D. See C for explanation.

Diagnostic Studies/Neurology A 22 year-old male presents to the clinic complaining of excessive daytime somnolence and strong desires to sleep at inappropriate times. He came in today because he had an episode of "feeling paralyzed" as he was falling asleep yesterday. What is the most appropriate diagnostic test to confirm this patient's diagnosis? Answers A. MRI of the brain B. Electroencephalogram C. Multiple sleep latency test D. Overnight polysomnography

Explanations (u) A. See C for explanation. (u) B. See C for explanation. (c) C. Multiple sleep latency test is required to observe the abrupt transition to REM sleep and establish the diagnosis of narcolepsy. (u) D. See C for explanation.

Diagnosis/Neurology Seizures that first manifest in early to middle adult life should be considered suspicious of which of the following causes? A. Cerebrovascular disease B. Encephalitis C. Tumor D. Idiopathic epilepsy

Explanations (u) A. See C for explanation. (u) B. See C for explanation. (c) C. Seizures that develop during adolescence and adult life are predominantly due to tumor, trauma, drug use, or alcohol withdrawal. (u) D. See C for explanation.

Scientific Concepts/Neurology A patient complains of loss of sensation at the level of the umbilicus. Which of the following dermatomes is affected? A. T6 B. T8 C. T10 D. T12

Explanations (u) A. See C for explanation. (u) B. See C for explanation. (c) C. The dermatome T10 is at the level of the umbilicus. (u) D. See C for explanation.

History & Physical/Neurology A 73 year-old male presents to the clinic with his wife. His wife has noticed that he has developed a resting tremor in his right hand and a shuffling gait over the last year. What finding on physical examination would support your suspected diagnosis? Answers A. Chorea B. Dystonia C. Masked facies D. Hyperreflexia

Explanations (u) A. See C for explanation. (u) B. See C for explanation. (c) C. The patient symptoms are consistent with Parkinsonism. Physical exam findings include masked facies, micrographia, decreased arm swing, and monotonous speech. (u) D. See C for explanation.

Clinical Intervention/Neurology A 50 year-old female has a history of severe, stabbing pains, lasting only seconds, over the cheek and chin areas. She also experiences intense pain in these areas with chewing, washing her face and even with smiling. Examination reveals no sensory or motor deficits. Head computer tomography (CT) and erythrocyte sedimentation rate (ESR) are normal. She has been unresponsive to maximum doses of carbamazepine. What is the next best option for this patient? A. Combination simvastatin (Zocor) and sertraline (Zoloft) B. Bilateral deep brain stimulation C. Stereotactic (Gamma knife) radiosurgery D. High dose corticosteroid therapy

Explanations (u) A. See C for explanation. (u) B. See C for explanation. (c) C. Trigeminal neuralgia is generally responsive to Carbamazepine however, for those who fail to respond to medicinal therapy surgery is the next valid option. Though there are various procedures, Stereotactic (Gamma knife) radiosurgery has provided the best results. (u) D. See C for explanation.

Clinical Intervention/Neurology A 23 year-old patient presents with two days of fatigue, headache, fever and pain around the area in which she was bitten by a stray baby raccoon in an unprovoked attack 10 days ago. She cleaned the small wound thoroughly. Which of the following is the most appropriate intervention in this patient? A. Do nothing and treat her symptoms B. Human diploid cell rabies vaccine, 5 injections given all at once now C. Rabies immunoglobulin only D. Rabies immunoglobulins and human diploid cell rabies vaccine given 5 times in a 1-month period

Explanations (u) A. See D for explanation. (u) B. See D for explanation. (u) C. See D for explanation. (c) D. Bites by bats, skunks, and raccoons always require this regimen if the animal is not caught and tested.

Scientific Concepts/Neurology You have just stuck yourself with a sharp needle. In order for you to be able to interpret this sensation, which of the following areas must be intact? A. Anterior spinothalamic tract, basal ganglia, and sensory cortex B. Corticospinal tract, medulla, and basal ganglia C. Pyramidal tract, hypothalamus, and sensory cortex D. Lateral spinothalamic tract, thalamus, and sensory cortex

Explanations (u) A. See D for explanation. (u) B. See D for explanation. (u) C. See D for explanation. (c) D. Sensory impulses reach the sensory cortex from the spinothalamic tract or the posterior columns. Fibers transmit this to the thalamus which sends impulses to the sensory cortex of the brain

Diagnosis/Neurology A 51 year-old female presents to the clinic complaining of intermittent sharp pain that originates at the corner of her mouth and radiates toward her ipsilateral eye. She notes "everything makes it worse" including touching the area, talking and eating. What is the most likely diagnosis? Answers A. Bell's palsy B. Cluster headache C. Post-herpetic neuralgia D. Trigeminal neuralgia

Explanations (u) A. See D for explanation. (u) B. See D for explanation. (u) C. See D for explanation. (c) D. This is the classic presentation for trigeminal neuralgia (tic douloureux).

History & Physical/Neurology In addition to tremor, which of the following are cardinal symptoms of Parkinson disease? A. Cognitive decline and rigidity B. Personality change and bradycardia C. Eye movement abnormalities and hyperkinesias D. Rigidity and bradykinesia

Explanations (u) A. See D for explanation. (u) B. See D for explanation. (u) C. See D for explanation. (c) D. Tremor, rigidity, bradykinesia and postural instability are the cardinal features of Parkinsonism and may be present in any combination.

Clinical Intervention/Neurology A 26 year-old female reports progressive distal to proximal spread of extremity weakness over the last 36 hours without fever, headache or syncope. Examination reveals symmetrical, paresis of the hands and feet with loss of the brachioradialis and Achillis reflexes. Biceps and knee reflexes are present but diminished. Sensory exam is normal. What are the most likely findings on cerebral spinal fluid (CSF) analysis? A. Decreased glucose, increased WBC count and decreased protein B. Increased glucose, normal WBC count and normal protein C. Normal glucose, decreased WBC count and elevated protein D. Normal glucose, normal WBC count and elevated protein

Explanations (u) A. See D for explanation. (u) B. See D for explanation. (u) C. See D for explanation. (c) D. Guillain-Barre syndrome is typified by progressive symmetrical, distal to proximal spread of weakness and areflexia without fever or sensory deficits. CSF analysis shows elevated protein due to axonal demyelination but no glucose disturbances and no significant or sustained pleocytosis.

Clinical Therapeutics/Neurology A patient with known relapsing-remitting multiple sclerosis (MS) presents to the hospital with an exacerbation of MS symptoms. What is the recommended treatment? A. Baclofen (Lioresal) B. Interferon C. Glatiramer acetate (Copaxone) D. Methylprednisolone

Explanations (u) A. See D for explanation. (u) B. See D for explanation. (u) C. See D for explanation. (c) D. Acute relapses of MS are treated with a short course of IV methylprednisolone followed by oral prednisone. This regimen reduces the severity and shortens the duration of attacks. All other drugs listed are used to reduce the attack rate of relapsing remitting multiple sclerosis.

Diagnosis/Neurology A 57 year-old male presents with episodic diplopia over the past two months. Symptoms progressed over the last two days with the onset of bilateral facial weakness made worse with repetitive use. Weakness improves somewhat with rest. He denies fever, headache or areas of pain. Exam reveals a nasal voice, drooping eyelids and a normal sensory exam. Which of the following is the most likely diagnosis? A. Multiple sclerosis B. Guillain-Barre syndrome C. Lambert-Eatonsyndrome D. Myastheniagravis

Explanations (u) A. See D for explanation. (u) B. See D for explanation. (u) C. See D for explanation. (c) D. Episodic diplopia progressing to proximal weakness worsened by repetition and lacking sensory deficits is typical of myasthenia gravis. In Lambert-Eaton syndrome muscular responses actually improve with repetition and the weakness in Guillain-Barre syndrome always begins peripherally.

History & Physical/Neurology A mother brings in her 4 year-old son with complaints that he falls frequently and he "stands funny". The mother also notes that he has lost the ability to easily stand from a seated position. She reports that he met developmental milestones as an infant. Which of the following is the most likely cause? A. Developmental hip dysplasia B. Myasthenia gravis C. Cerebral palsy D. Muscular dystrophy

Explanations (u) A. Signs and symptoms of developmental hip dysplasia would have been present more at birth and when beginning to crawl and walk. (u) B. Myasthenia gravis (MG) is less likely due to age and MG typically affects the ocular, pharyngeal and respiratory muscles. (u) C. Cerebral palsy (CP) typically presents with ataxia, spasticity or tightness, and uncoordination. Patients with CP typically do not meet developmental milestones. (c) D. Muscular dystrophy age of onset is by age 5, and begins in the pelvic girdle.

History & Physical/Neurology Topic: 3 Author: Which of the following primitive reflexes should begin to disappear at about 2 months of age in a normal infant? A. Moro B. Grasp C. Tonic neck D. Parachute

Explanations (u) A. The Moro reflex starts to disappear at about 5-6 months of age. (c) B. The grasp reflex starts to disappear at about 2-3 months of age. (u) C. The tonic neck reflex starts to disappear at about 6-7 months of age. (u) D. The parachute reflex remains throughout life.

Health Maintenance/Neurology A 45 year-old man presents for a routine appointment. He tells you his mother and father have both had ischemic strokes in their 70's. He does not smoke. His blood pressure is 128/80 mmHg, pulse 78/minutes and regular, respiratory rate of 12/minute. What diagnostic studies would you order to further evaluate this patient's risk of stroke? Answers A. Electrocardiogram B. Fasting lipid profile C. Carotid Doppler ultrasound D. MRI with gadolinium

Explanations (u) A. The main risk factor assessed by ECG is atrial fibrillation and this patient's regular pulse confirms he is currently not in atrial fibrillation (c) B. Hyperlipidemia is a known risk factors for stroke that can be modified with treatment. (u) C. The patient does not have signs or symptoms of carotid stenosis at this point. (u) D. An MRI with gadolinium would be useful in evaluating for the presence of Berry aneurysms, but the history of ischemic strokes does not raise the concern of an aneurysm.

Clinical Intervention/Neurology An 82 year-old male is brought to the ED after being found unresponsive in his apartment. On physical exam his pupils are 7 mm on the right and 3 mm on the left. your initial choice in managing this patient? Answers A. Order a CBC, electrolytes, and toxicology screen B. Order a CT of the head C. Hyperventilate the patient D. Administer streptokinase

Explanations (u) A. The patient has PE findings that are asymmetric. This is consistent with an anatomical abnormality and not a systemic disorder. (c) B. This presentation is consistent with a structural abnormality that will be detected on an imaging study. (h) C. Prophylactic hyperventilation is not recommended because cerebral perfusion pressure can be decreased and cause further injury to the brain. (h) D. Streptokinase is not indicated due to possibility of a hemorrhagic event.

Clinical Therapeutics/Neurology Which of the following drugs is the first choice for insomnia in an elderly patient? A. Barbiturates B. Cholinesterase inhibitors C. Benzodiazepines D. Beta-blockers

Explanations (u) A. The use of barbiturates is obsolete and these agents may actually disrupt the sleep cycle. (u) B. Cholinesterase inhibitors are used in the treatment of Alzheimer's, but have no effect on insomnia. (c) C. Benzodiazepines are the drugs of choice for insomnia in the elderly population. (u) D. Beta-blockers have no use in the treatment of insomnia.

iagnostic Studies/Neurology An 18 year-old male is involved in a motor vehicle accident with a question of cervical spine fracture. What is the imaging test of choice to initially evaluate this patient and clear his cervical c-spine? A. Positron emission tomography B. Magnetic resonance imaging C. Computed tomography D. Lateral radiograph

Explanations (u) A. There is no role for positron emission tomography in suspected cervical spine injury. (u) B. MRI and CT of the spine may be performed in the setting of acute cervical spine injury when a major fracture or dislocation is identified. (u) C. See B for explanation. (c) D. Cervical spine x-rays are most commonly used as the initial screen for cervical spine injury. A cervical spine series consists of a lateral view, anteroposterior (AP) view, and an odontoid view. The lateral view detects up to 80% of traumatic spine injuries.

HESI HINT: What influences the type of traumatic brain injury?

Forces of impact: - Acceleration Injury (head was in motion) - Deceleration Injury (head stops suddenly) *Helmets are a great preventive measure for bicyclists and motorists

Diagnosis/Neurology An 8 year-old male presents with episodes described as 20-second lapses of awareness during which he blinks his eyes. After these attacks, he resumes his previous activity. Which of the following is the most likely diagnosis? A. Tonic-clonic seizure B. Myoclonic seizure C. Absence seizure D. Focal seizure

Explanations (u) A. Tonic-clonic seizures are associated with a loss of consciousness, tonic-clonic movements, and postictal confusion or sleep. (u) B. Myoclonic seizures are associated with mild myoclonic jerks of the neck and shoulder flexor muscles when falling asleep. (c) C. Absence seizures are characterized by lapses of consciousness associated with postictal confusion. (u) D. Focal seizures are associated with motor or sensory symptoms involving any portion of the body.

Diagnosis/Neurology A 75 year-old male presents for a routine physical. Vitals are normal with no orthostatic changes. On physical examination, a fine cortical movement with repetitive rubbing of the tip of the thumb along the tips of the fingers is noted at rest. Which of the following is the most likely diagnosis? A. Seizure disorder B. Peripheral neuropathy C. Shy-Drager syndrome D. Parkinson's disease

Explanations (u) A. While a seizure may present with fine or gross uncontrolled motor movements, the tremor described is classic pill-rolling tremor noted in Parkinson's disease. (u) B. Peripheral neuropathy presents with loss of sensation not tremor. (u) C. Shy-Drager syndrome is due to autonomic degeneration and typically presents with orthostatic hypotension. (c) D. Parkinson's disease presents with tremor at rest (pill-rolling), bradykinesia, rigidity, and postural instability.

Diagnostic Studies/Neurology What test is the single most useful test in establishing the diagnosis of multiple sclerosis? Answers A. Cerebral spinal fluid cell count and protein level B. Cerebral spinal fluid immunoglobulin studies C. Evoked potentials D. Magnetic Resonance Imaging

Explanations (u) A. While cerebral spinal fluid cell count, protein levels, and immunoglobins may be abnormal they are not specific for multiple sclerosis. (u) B. See A for explanation. (u) C. Evoked potentials are most useful in the detection of subclinical involvement of neuropathways in MS, but does not establish the diagnosis. (c) D. The presence of plaques on MRI is a key finding in establishing the diagnosis of MS.

Health Maintenance/Neurology A 53 year-old female has a diagnosis of migraine headaches. She had been using sumatriptan (Imitrex) to abort her headaches, but she is now having one or two headaches per week. The most appropriate preventive therapy is A. zolmitriptan (Zomig). B. promethazine (Phenergan). C. propranolol (Inderal). D. fluoxetine (Prozac).

Explanations (u) A. Zolmitriptan, another 5-HT receptor agonist, is used acutely to abort migraine headaches. (u) B. Promethazine is an antiemetic that may be used to alleviate nausea and vomiting from an acute migraine. (c) C. Propanolol is useful in preventing migraine headaches and may be maintained indefinitely. (u) D. Fluoxetine is an SSRI that is used in the treatment of anxiety and obsessive-compulsive disorders.

A patient with mild dementia has a new prescription for donepezil (Aricept). Which nursing action will be most effective in ensuring compliance with the medication?

Having the patient's spouse administer the medication rational: Because the patient with mild dementia will have difficulty with learning new skills and forgetfulness, the most appropriate nursing action is to have someone else administer the drug. The other nursing actions will not be as effective in ensuring that the patient takes the medications.

Headache and vomiting are sxs of many disorders. What characteristic of these sxs would alert the nurse to refer a client to a neurologist?

Headache that is more severe upon awakening and vomiting not associated with nausea are sxs of brain tumor

HESI HINT: How is CNS involvement related to cause of stroke?

Hemorrhagic: - Caused by a slow or fast hemorrhage into the brain tissue; often related to HTN Embolic: - Caused by a clot that has broken away from a vessel and has lodged in one of the arteries of the brain, blocking the blood supply - Often related to atherosclerosis (so it may occur again)

A patient with multiple sclerosis (MS) is to begin treatment with glatiramer acetate (Copaxone). Which information will the nurse include in patient teaching?

How to draw up and administer injections of the medication rational: Copaxone is administered by self-injection. Oral contraceptives are an appropriate choice for birth control. There is no need to avoid driving or drink large fluid volumes when taking glatiramer.

What are the sxs of autonomic dysreflexia?

Hypertension Bladder/Bowel distention Exaggerated autonomic responses Headache Sweating Goose bumps Bradycardia

What are the sxs of spinal shock?

Hypotension, bladder/bowel distention, total paralysis, lack of sensation below lesion

A patient who has numbness and weakness of both feet is hospitalized with Guillain-Barré syndrome. The nurse will anticipate the need to teach the patient about _____________

IV infusion of immunoglobulin (Sandoglobulin). rational: Because the Guillain-Barré syndrome is in the earliest stages (as evidenced by the symptoms), use of high-dose immunoglobulin is appropriate to reduce the extent and length of symptoms. Mechanical ventilation and tube feedings may be used later in the progression of the syndrome but are not needed now. Corticosteroid use is not helpful in reducing the duration or symptoms of the syndrome.

A patient with Parkinson's disease has decreased tongue mobility and an inability to move the facial muscles. Which nursing diagnosis is of highest priority?

Imbalanced nutrition: less than body requirements rational: The data about the patient indicate that poor nutrition will be a concern because of decreased swallowing. The other diagnoses also may be appropriate for a patient with Parkinson's disease, but the data do not indicate they are current problems for this patient.

The community health nurse is developing a program to decrease the incidence of meningitis in adolescents and young adults. Which nursing action is most important?

Immunize adolescents and college freshman against Neisseria meningitides. rational: The Neisseria meningitides vaccination is recommended for children ages 11 and 12, unvaccinated teens entering high school, and college freshmen. Hand washing may help decrease the spread of bacteria, but it is not as effective as immunization. Vaccination with Haemophilus influenzae is for infants and toddlers. Because adolescents and young adults are in school or the workplace, avoiding crowds is not realistic.

HESI HINT: What is dysplasia?

Impairment of speech and verbal comprehension

which new assessment finding indicates that the patient has improved since the last neurologic check 2 hours prior?

Improvement in Glasgow Coma Scale over 12

HESI HINT: What is apraxia?

Inability to perform purposeful movements in the absence of motor problems

What vital sign changes are indicative of increased ICP?

Increased BP, widening pulse pressure, increased/decreased pulse, resp. irregularities, and temp increase

List four nursing dx for the comatose client in order of priority. (Remember Maslow's Hierarchy of Needs to help determine priorities)

Ineffective breathing pattern Ineffective airway clearance Impaired gas exchange Decreased cardiac output

While admitting a patient with a basal skull fracture, the nurse notes clear drainage from the patient's nose. Which of these admission orders should the nurse question?

Insert nasogastric tube. rational: Rhinorrhea may indicate a dural tear with cerebrospinal fluid (CSF) leakage, and insertion of a nasogastric tube will increase the risk for infections such as meningitis. Turning the patient, elevating the head, and applying cold pack are appropriate orders.

Which action will the nurse take when evaluating a patient who is taking phenytoin (Dilantin) for adverse effects of the medication?

Inspect the oral mucosa. rational: Phenytoin can cause gingival hyperplasia, but does not affect bowel tones, lung sounds, or pupil reaction to light.

what assessment sign indicates that a patient has Bell's Palsy due to inflammation around cranial nerve 7 ?

Inspection of the buccal pouch for food after eating

what should the nurse instruct the patient who is at risk for falls?

Instruct the patient to ask for help before getting up Turn on the bed alarm Ask the patient's family to inform the staff when leaving Place the bed in the lowest position

Which parameter is best for the nurse to monitor to determine whether the prescribed IV mannitol (Osmitrol) has been effective for an unconscious patient?

Intracranial pressure rational: Mannitol is an osmotic diuretic and will reduce cerebral edema and intracranial pressure. It may initially reduce hematocrit and increase blood pressure, but these are not the best parameters for evaluation of the effectiveness of the drug. Oxygen saturation will not directly improve as a result of mannitol administration.

What nursing interventions prevent corneal drying in a comatose client?

Irrigation of eyes PRN with sterile prescribed solution Application of ophthalmic ointment every 8 hours Close assessment for corneal ulceration or drying

HESI HINT: What is the lens of the eye responsible for?

It projects light onto the retina so that images can be discerned Without the lens, which becomes opaque with cataracts, light cannot be filtered and vision is blurred

HESI HINT: What is myasthenia gravis?

Its a disorder affecting the neuromuscular transmission of impulses in the voluntary muscles of the body

During the morning change-of-shift report at the long-term care facility, the nurse learns that the patient with dementia has had sundowning. Which nursing action should the nurse take while caring for the patient?

Keep blinds open during the daytime hours. rational: The most likely cause of sundowning is a disruption in circadian rhythms and keeping the patient active and in daylight will help to reestablish a more normal circadian pattern. Moving the patient to a different room might increase confusion. Taking a nap will interfere with nighttime sleep. Hourly orientation will not be helpful in a patient with memory difficulties.

An unconscious patient has a nursing diagnosis of ineffective cerebral tissue perfusion related to cerebral tissue swelling. Which nursing intervention will be included in the plan of care?

Keep the head of the bed elevated to 30 degrees. rational: The patient with increased intracranial pressure (ICP) should be maintained in the head-up position to help reduce ICP. Flexion of the hips and knees increases abdominal pressure, which increases ICP. Because the stimulation associated with nursing interventions increases ICP, clustering interventions will progressively elevate ICP. Coughing increases intrathoracic pressure and ICP.

A client with a dx of stroke presents with symptoms of aphasia and right hemiparesis but no memory or hearing deficit. In what hemisphere has the client suffered a lesion?

Left

To evaluate the effectiveness of IV methylprednisolone (Solu-Medrol) given to a patient with a T4 spinal cord injury, which information is most important for the nurse to obtain?

Leg strength and sensation rational: The purpose of methylprednisolone administration is to help preserve motor function and sensation. Therefore the nurse will assess this patient for lower extremity function. The other data also will be collected by the nurse, but they do not reflect the effectiveness of the methylprednisolone.

HESI HINT: What risk increases when the cataract is removed?

Lens is gone --> Risk for falls When lens is replaced with impact, vision is better

HESI HINT: What is agraphia?

Loss of ability to write

HESI HINT: What is alexia?

Loss of the ability to read

HESI HINT: What is aphasia?

Loss of the ability to speak

The nurse is discussing glaucoma prevention with a 52-year-old African-American patient. Which statement by the patient reflects a correct understanding of glaucoma prevention? 1 "I will visit my eye doctor every one to two years." 2 "I will wear protective sunglasses while outside." 3 "I will take lutein and vitamin E supplements for eye health." 4 "There is nothing that can be done to prevent vision loss from glaucoma."

Loss of vision as a result of glaucoma is a preventable problem. Teach the patient and the caregiver about the risk of glaucoma and that it increases with age. Stress the importance of early detection and treatment in preventing visual impairment. A comprehensive ophthalmic examination is important in identifying persons with glaucoma or those at risk of developing glaucoma. The current recommendation is for an ophthalmologic examination every two to four years for persons between ages 40 and 64 years, and every one to two years for persons age 65 years or older. African Americans in every age category should have examinations more often because of the increased incidence and more aggressive course of glaucoma in these individuals. Wearing protective sunglasses while outside may help to reduce the development of cataracts, not glaucoma. Lutein and vitamin supplements may be helpful for preventing macular degeneration, not glaucoma. Text Reference - p. 401 1

Which intervention will the nurse include in the plan of care for a patient who has late-stage Alzheimer's disease (AD)?

Maintain a consistent daily routine for the patient's care. rational: Providing a consistent routine will decrease anxiety and confusion for the patient. In late-stage AD, the patient will not remember events from the past. Reorientation to time and place will not be helpful to the patient with late-stage AD, and the patient will not be able to read.

A nurse is providing information to a group of nursing students, explaining the difference between a migraine headache and a tension-type headache. What should the nurse include as diagnostic criteria for a migraine headache? 1 Unilateral pain 2 Non-throbbing pain 3 Movement does not affect its severity 4 Photophobia or phonophobia but not both

Migraine headaches have at least 2 of the following characteristics: unilateral; throbbing; aggravated by movement; moderate to severe intensity. Migraine headaches also have at least one of the following charactistics: nausea/vomiting; photophobia and phonophobia. Tension-type headaches have at least 2 of the following characteristics: bilateral; non-throbbing; movement does not affect its severity; mild to moderate intensity. Tension-type headaches also have at least one of the following characteristics: no nausea/vomiting; photophobia or phonophobia but not both. 1

What substance is it most appropriate for the nurse to use to remove an insect from a patient's ear? 1 Water 2 Alcohol 3 Mineral oil 4 Hydrogen peroxide

Mineral oil causes the least amount of trauma and irritation to the ear canal. Water and hydrogen peroxide should not be used because the insect could swell, which would make it more difficult to remove. Alcohol may cause both irritation of the ear canal and swelling of the insect. Text Reference - p. 403 3

A 26-year-old patient with a T3 spinal cord injury asks the nurse about whether he will be able to be sexually active. Which initial response by the nurse is best?

Multiple options are available to maintain sexuality after spinal cord injury. rational: Although sexuality will be changed by the patient's spinal cord injury, there are options for expression of sexuality and for fertility. The other information also is correct, but the choices will depend on the degrees of injury and the patient's individual feelings about sexuality.

HESI HINT: What is the difference between a myasthenic crisis and a cholinergic crisis?

Myasthenic Crisis: - Associated with under medication - Increase in sxs: more difficulty swallowing, diplopia, ptosis, dyspnea - Positive Edrophonium (Tensilon) test Cholinergic Crisis: - Associated with anti-cholinesterase overdosage - Increase in sxs: diaphoresis, diarrhea, fasciculations, cramps, marked worsening of sxs resulting from overmedication - Negative Tensilon test

Why should narcotics be avoided in clients with neurologic impairment?

Narcotics mask the level of responsiveness and pupillary responses

in a patient who has difficulty swallowing food what provides adequate nutrition for the client?

Nasogastric Tube feedings

what distinguishes Alzheimer's disease from other dementias?

Neurofibrillary tangles and plaques in the brain

Which of these nursing actions for a patient with Guillain-Barré syndrome is most appropriate for the nurse to delegate to an experienced nursing assistant?

Passive range of motion to extremities q8hr rational: Assisting a patient with movement is included in nursing assistant education and scope of practice. Administration of tube feedings, administration of ordered medications, and assessment are skills requiring more education and scope of practice, and the RN should perform these skills.

A patient underwent aneurysm clipping six hours ago for subarachnoid hemorrhage and is being treated with nimodipine. While examining the patient, the nurse finds that the pulse of the patient is 50 beats per minute (bpm) and the blood pressure is 90/60 mm Hg. What should the nurse do? 1 Encourage intake of fluids orally. 2 Monitor blood pressure every half hour. 3 Start intravenous fluids to increase blood volume. Correct4 Hold the medication and contact the primary health care provider

Nimodipine is a calcium channel blocker that is given to patients with subarachnoid hemorrhage to decrease the effects of vasospasm and to minimize cerebral damage. Nimodipine lowers the blood pressure, and therefore before administration, it is important to assess the blood pressure and apical pulse. If the pulse and blood pressure drop (pulse is less than 60 beats per minute and systolic blood pressure is less than 90 mm Hg), the medication should be stopped and the primary healthcare provider should be contacted immediately. The nurse should not start IV fluids without contacting the health care provider first. The blood pressure may be monitored more frequently if they are in intensive care or unstable. The patient may be NPO or unable to have oral fluids at this point. Text Reference - p. 1395 4

Is paralysis always a consequence of spinal cord injury?

No

A patient with right-sided weakness that started 90 minutes earlier is admitted to the emergency department and all these diagnostic tests are ordered. Which test should be done first?

Noncontrast computed tomography (CT) scan rational: Rapid screening with a noncontrast CT scan is needed before administration of tissue plasminogen activator (tPA), which must be given within 4.5 hours of the onset of clinical manifestations of the stroke. The sooner the tPA is given, the smaller the area of brain injury. The other diagnostic tests give information about possible causes of the stroke and do not need to be completed as urgently as the CT scan.

Following a thymectomy, a patient with myasthenia gravis receives the usual dose of pyridostigmine (Mestinon). An hour later, the patient complains of nausea and severe abdominal cramps. Which action should the nurse take first?

Notify the patient's health care provider. rational: The patient's history and symptoms indicate a possible cholinergic crisis. The health care provider should be notified immediately, and it is likely that atropine will be prescribed. The other actions will be appropriate if the patient is not experiencing a cholinergic crisis.

When the nurse is assessing a patient with myasthenia gravis, which action will be most important to take?

Observe respiratory effort. rational: Because respiratory insufficiency may be life threatening, it will be most important to monitor respiratory function. The other data also will be assessed but are not as critical.

A patient is brought to the emergency department (ED) by ambulance after being found unconscious on the bathroom floor by the spouse. Which action will the nurse take first?

Obtain oxygen saturation. rational: Airway patency and breathing are the most vital functions and should be assessed first. The neurologic assessments should be accomplished next and the health and medication history last.

HESI HINT: What population is most affected by glaucoma?

Older adult populations - Older clients are prone to problems associated with constipation - Nurse should assess these clients for constipation and post op complications associated with constipation - Prevent and treat this

When a patient's intracranial pressure (ICP) is being monitored with an intraventricular catheter, which information obtained by the nurse is most important to communicate to the health care provider?

Oral temperature 101.6° F rational: Infection is a serious consideration with ICP monitoring, especially with intraventricular catheters. The temperature indicates the need for antibiotics or removal of the monitor. The ICP, arterial pressure, and apical pulse are all borderline high but require only ongoing monitoring at this time.

what nursing approach for communications would be best for a patient with AIDS dementia complex if he becomes confused?

Orient the patient to the current surroundings and situations

A patient has experienced a right brain stroke. What intervention would be most important to include in the plan of care? 1 Allow extra time for transfer and activity. 2 Assist with self-catheterization every four hours. 3 Evaluate body positioning during all transfers. 4 Provide activities that promote verbal memory skills.

Patients who have experienced a right brain stroke are more likely to have problems with spatial-perceptual orientation. One spatial-perceptual deficit is neglect, where the patient neglects all input from his or her own body part, often the left side of the body. It is important for staff members to evaluate body position during transfers to assure the left sided-extremities are in anatomical alignment to minimize injury. Patients with right-sided brain stokes generally are impulsive and move more rapidly, while left-sided brain strokes are more slow and cautious. Generally, the prognosis for normal bladder function is excellent, allowing for partial to complete bladder evacuation without any interventions such as intermittent catheterizations. Memory can be affected with strokes involving either side of the brain; verbal memory skills are most affected with left brain strokes. Text Reference - p. 1409 3

HESI HINT: Why is it imperative to reverse spinal shock as quickly as possible?

Permanent paralysis can occur if a spinal cord is compressed for 12-24 hours.

what nursing assesment findings support meningitis diagnosis ?

Photophobia stiff neck fever

When the home health RN is planning care for a patient with a seizure disorder, which nursing action can be delegated to an LPN/LVN?

Place medications in the home medication organizer. rational: LPN/LVN education includes administration of medications. The other activities require RN education and scope of practice.

When caring for a patient with left-sided homonymous hemianopsia resulting from a stroke, which intervention should the nurse include in the plan of care during the acute period of the stroke?

Place objects needed for activities of daily living on the patient's right side. rational: During the acute period, the nurse should place objects on the patient's unaffected side. Since there is a visual defect in the left half of each eye, an eye patch is not appropriate. The patient should be approached from the right side. The visual deficit may not resolve, although the patient can learn to compensate for the defect.

When assessing a patient with Alzheimer's disease (AD) who is being admitted to a long-term care facility, the nurse learns that the patient has had several episodes of wandering away from home. Which nursing action will the nurse include in the plan of care?

Place the patient in a room close to the nurses' station. rational: Patients at risk for problems with safety require close supervision. Placing the patient near the nurse's station will allow nursing staff to observe the patient more closely. The use of "why" questions is frustrating for patients with AD because they are unable to understand clearly or verbalize the reason for wandering behaviors. Because of the patient's short-term memory loss, reorientation will not help prevent wandering behavior. Because the patient had wandering behavior at home, familiar objects will not prevent wandering.

A patient has an incomplete right spinal cord lesion at the level of T7, resulting in Brown-Séquard syndrome. Which nursing action should be included in the plan of care?

Positioning the patient's right leg when turning the patient rational: The patient with Brown-Séquard syndrome has loss of motor function on the ipsilateral side and will require the nurse to move the right leg. Pain sensation will be lost on the patient's left leg. Left arm weakness will not be a problem for a patient with a T7 injury. The patient will retain position sense for the left leg.

Which three symptoms are characteristic of Cushing's triad associated with ICP? a. hypotension, tachycardia, narrowing pulse pressure b. hypertension, tachycardia, HA c. widening pulse pressure, HA, seizure d. bradycardia, hypertension, widening pulse pressure

d

A patient who is suspected of having an epidural hematoma is admitted to the emergency department. Which action will the nurse plan to take?

Prepare the patient for immediate craniotomy. rational: The principal treatment for epidural hematoma is rapid surgery to remove the hematoma and prevent herniation. If intracranial pressure (ICP) is elevated after surgery, furosemide or high-dose barbiturate therapy may be needed, but these will not be of benefit unless the hematoma is removed. Minimal blood loss occurs with head injuries, and transfusion is usually not necessary.

Which information about a patient who is hospitalized after a traumatic brain injury requires the most rapid action by the nurse?

Pressure of oxygen in brain tissue (PbtO2) is 14 mm Hg rational: The PbtO2 should be 20 to 40 mm Hg. Lower levels indicate brain ischemia. An intracranial pressure (ICP) of 15 mm Hg is at the upper limit of normal. CSF is produced at a rate of 20 to 30 mL/hour. The reason for the sinus tachycardia should be investigated, but the elevated heart rate is not as concerning as the decrease in PbtO2.

A patient who has a head injury is diagnosed with a concussion. Which action will the nurse plan to take?

Provide discharge instructions about monitoring neurologic status. rational: A patient with a minor head trauma is usually discharged with instructions about neurologic monitoring and the need to return if neurologic status deteriorates. MRI, hospital admission, or surgery are not indicated in a patient with a concussion.

When developing a plan of care for a hospitalized patient with moderate dementia, which intervention will the nurse include?

Remind the patient frequently about being in the hospital. rational: The patient with moderate dementia will have problems with short- and long-term memory and will need reminding about the hospitalization. The other interventions would be used for a patient with severe dementia, who would have difficulty with swallowing, self-care, and immobility.

A patient has a systemic BP of 108/51 mm Hg and an intracranial pressure (ICP) of 14 mm Hg. Which action should the nurse take first?

Report the BP and ICP to the health care provider. rational: The patient's cerebral perfusion pressure is 56 mm Hg, below the normal of 60 to 100 mm Hg and approaching the level of ischemia and neuronal death. Immediate changes in the patient's therapy such as fluid infusion or vasopressor administration are needed to improve the cerebral perfusion pressure. Adjustments in the head elevation should only be done after consulting with the health care provider. Continued monitoring and documentation also will be done, but they are not the first actions that the nurse should take.

HESI HINT: What bx changes indicate increased ICP?

Restlessness, irritability, confusion

A patient with a stroke has progressive development of neurologic deficits with increasing weakness and decreased level of consciousness (LOC). Which nursing diagnosis has the highest priority for the patient?

Risk for aspiration related to inability to protect airway rational: Protection of the airway is the priority of nursing care for a patient having an acute stroke. The other diagnoses also are appropriate, but interventions to prevent aspiration are the priority at this time.

what medications increase the risk for falls

SELECT ALL THAT APPLY Benzodiazipines Sedatives Opioids Anticonvulsants Anithypertensives

what does the nurse instruct the patient to avoid the day before having a PET scan ?

SELECT ALL THAT APPLY caffeine alcohol tobacco

how would the nurse instruct the caregiver to perform ROM exercises in a patient with right sided hemiplegia ?

SELECT ALL THAT APPLY move the paralyzed limbs in as many directions as possible repeat each exercise 3 times allow rest periods between exercises

what clinical finding indicates that the patient is brain dead?

SELECT ALL THAT APPLY 1. Deep tendon reflexes 2. Absent corneal reflex 3. Dilated nonreactive pupils

what discharge instructions should the nurse include in a patient who had a diskectomy with spinal fusion ?

SELECT ALL THAT APPLY 1. avoid twisting or jerking the back 2. avoid sitting for long periods during the first week 3. monitor urine output for the first week 4. report lower extremity color changes to the physician immediately

what are the FACTORS THAT INCREASES THE RISK OF FALLS for a male stroke victim with right sided hemiplegia ?

SELECT ALL THAT APPLY The patient is male The patient is confused The patient has vertigo The patient has impaired vision The patient needs help with toileting

what s/s does the nurse expect to see in a patient with autonomic dysreflexia spinal cord injury ?

SELECT ALL THAT APPLY severe hypertension pounding headache blurred vision nasal stuffiness

Which action will the nurse in the outpatient clinic include in the plan of care for a patient with mild cognitive impairment (MCI)?

Schedule the patient for more frequent appointments. rational: Ongoing monitoring is recommended for patients with MCI. MCI does not interfere with activities of daily living, acetylcholinesterase drugs are not used for MCI, and an assisted living facility is not indicated for MCI.

what nursing diagnosis is included in a patient with Lou Gehrig Disease ( ALS)?

Self-Care deficit

Which assessment information will the nurse collect to determine whether a patient is developing postconcussion syndrome?

Short-term memory rational: Decreased short-term memory is one indication of postconcussion syndrome. The other data may be assessed but are not indications of postconcussion syndrome.

A hospitalized 24-year-old patient with a history of cluster headache awakens during the night with a severe stabbing headache. Which action should the nurse take first?

Start the ordered PRN oxygen at 6 L/min. rational: Acute treatment for cluster headache is administration of 100% oxygen at 6 to 8 L/min. If the patient obtains relief with the oxygen, there is no immediate need to notify the health care provider. Cluster headaches last only 60 to 90 minutes, so oral pain medications have minimal effect. Hot packs are helpful for tension headaches but are not as likely to reduce pain associated with a cluster headache.

A patient with Parkinson's disease has a nursing diagnosis of impaired physical mobility related to bradykinesia. Which action will the nurse include in the plan of care?

Suggest that the patient rock from side to side to initiate leg movement. rational: Rocking the body from side to side stimulates balance and improves mobility. The patient will be encouraged to continue exercising because this will maintain functional abilities. Maintaining a wide base of support will help with balance. The patient should lift the feet and avoid a shuffling gait.

How should the head of the bed be positioned for post craniotomy clients with infratentorial lesions?

Supratentorial: Elevated Infratentoral: Flat

A pt. with Parkinson's received all of the following gifts. Which would be the most useful? a. soft-soled house shoes b. jigsaw puzzle c. set of card games d. satin sheets

d power points

A patient admitted with bacterial meningitis and a temperature of 102° F (38.8° C) has orders for all of these collaborative interventions. Which action should the nurse take first?

Swap the nasopharyngeal mucosa for cultures. rational: Antibiotic therapy should be instituted rapidly in bacterial meningitis, but cultures must be done before antibiotics are started. As soon as the cultures are done, the antibiotic should be started. Hypothermia therapy and acetaminophen administration are appropriate but can be started after the other actions are implemented.

A patient with paraplegia resulting from a T10 spinal cord injury has a neurogenic reflex bladder. Which action will the nurse include in the plan of care?

Teach the patient how to self-catheterize. rational: Because the patient's bladder is spastic and will empty in response to overstretching of the bladder wall, the most appropriate method is to avoid incontinence by emptying the bladder at regular intervals through intermittent catheterization. Assisting the patient to the toilet will not be helpful because the bladder will not empty. The Credé method is more appropriate for a bladder that is flaccid, such as occurs with a reflexic neurogenic bladder. Catheterization after voiding will not resolve the patient's incontinence.

A patient with multiple sclerosis (MS) has urinary retention caused by a flaccid bladder. Which action will the nurse plan to take?

Teach the patient how to use the Credé method. rational: The Credé method can be used to improve bladder emptying. Decreasing fluid intake will not improve bladder emptying and may increase risk for urinary tract infection (UTI) and dehydration. The use of incontinence briefs and frequent toileting will not improve bladder emptying.

Which nursing action will the home health nurse include in the plan of care for a patient with paraplegia in order to prevent autonomic dysreflexia?

Teach the purpose of a prescribed bowel program. rational: Fecal impaction is a common stimulus for autonomic dysreflexia. The other actions may be included in the plan of care but will not reduce the risk for autonomic dysreflexia.

Which assessment finding in a patient who was admitted the previous day with a basilar skull fracture is most important to report to the health care provider?

Temperature of 101.5° F (38.6° C) rational: Patients who have basilar skull fractures are at risk for meningitis, so the elevated temperature should be reported to the health care provider. The other findings are typical of a patient with a basilar skull fracture.

The most reliable indicator of neurological status is a. blood pressure b. pulse rate c. temperature d. LOC

d.

When planning for venous thromboembolism (VTE) prevention, the nurse places as priority: 1 Application of compression stockings on legs. 2 Active and passive range-of-motion (ROM) exercises. 3 Use of sequential compression devices on lower legs. 4 Administration of daily enoxaparin (Lovenox) injections.

The most effective prevention of VTEs is to keep the patient moving. VTEs develop because of immobility, loss of venous tone, and decreased muscle pumping activity in the legs. If able, the patient can perform active ROM exercises, such as foot pumps, ankle circles, and flexion and extension of legs. If unable to perform, the staff should perform scheduled passive ROM exercises. Use of compression stockings (TED hose) and sequential depression devices, when used appropriately, are effective in preventing VTEs. The use of enoxaprin is also effective at preventing VTEs, but is not without risks, such as bleeding and excessive bruising. Text Reference - p. 1405 2

While caring for a patient who has just been admitted with meningococcal meningitis, the RN observes all of the following. Which one requires action by the RN?

The nursing assistant goes into the patient's room without a mask. rational: Meningococcal meningitis is spread by respiratory secretions, so it is important to maintain respiratory isolation as well as standard precautions. Because the patient may be confused and weak, bedrails should be elevated at both the food and head of the bed. Low light levels in the room decrease pain caused by photophobia. Nutrition is an important aspect of care in a patient with meningitis.

The health care provider is considering the use of sumatriptan (Imitrex) for a patient with migraine headaches. Which information obtained by the nurse is most important to report to the health care provider?

The patient has a history of a recent acute myocardial infarction. rational: The triptans cause coronary artery vasoconstriction and should be avoided in patients with coronary artery disease. The other information will be reported to the health care provider, but none of it is an indication that sumatriptan would be an inappropriate treatment.

Which information about a patient with MS indicates that the nurse should consult with the health care provider before giving the prescribed dose of fampridine (Ampyra)?

The patient has an increased creatinine level. rational: Fampridine should not be given to patients with impaired renal function. The other information will not impact on whether the fampridine should be administered.

A patient with sudden-onset right-sided weakness has a CT scan and is diagnosed with an intracerebral hemorrhage. Which information about the patient is most important to communicate to the health care provider?

The patient has atrial fibrillation and takes warfarin (Coumadin). rational: The use of warfarin will have contributed to the intracerebral bleeding and remains a risk factor for further bleeding. Administration of vitamin K is needed to reverse the effects of the warfarin, especially if the patient is to have surgery to correct the bleeding. The history of hypertension is a risk factor for the patient but has no immediate effect on the patient's care. The BP of 144/90 indicates the need for ongoing monitoring but not for any immediate change in therapy. Slurred speech is consistent with a left-sided stroke, and no change in therapy is indicated.

When caring for a patient who has Guillain-Barré syndrome, which assessment data obtained by the nurse will require the most immediate action?

The patient has continuous drooling of saliva. rational: Drooling indicates decreased ability to swallow, which places the patient at risk for aspiration and requires rapid nursing and collaborative actions such as suctioning and possible endotracheal intubation. The foot pain should be treated with appropriate analgesics, and the BP requires ongoing monitoring, but these actions are not as urgently needed as maintenance of respiratory function. Absence of the reflexes should be documented, but this is a common finding in Guillain-Barré syndrome.

The nurse is caring for a patient with carotid artery narrowing who has just returned after having left carotid artery angioplasty and stenting. Which assessment information is of most concern to the nurse?

The patient has difficulty talking. rational: Small emboli can occur during carotid artery angioplasty and stenting, and the aphasia indicates a possible stroke during the procedure. Slightly elevated pulse rate and blood pressure are not unusual because of anxiety associated with the procedure. Fine crackles at the lung bases may indicate atelectasis caused by immobility during the procedure; the nurse should have the patient take some deep breaths.

The nurse is assessing a patient who is being evaluated for a possible spinal cord tumor. Which finding by the nurse requires the most immediate action?

The patient has new onset weakness of both legs. rational: The new onset of symptoms indicates cord compression, an emergency that requires rapid treatment to avoid permanent loss of function. The other patient assessments also indicate a need for nursing action but do not require intervention as rapidly as the new onset weakness.

When caring for a patient who has had a head injury, which assessment information requires the most rapid action by the nurse?

The patient is more difficult to arouse. rational: The change in level of consciousness (LOC) is an indicator of increased intracranial pressure (ICP) and suggests that action by the nurse is needed to prevent complications. The change in BP should be monitored but is not an indicator of a need for immediate nursing action. Headache is not unusual in a patient after a head injury. A slightly irregular apical pulse is not unusual.

Aspirin is ordered for a patient who is admitted with a possible stroke. Which information obtained during the admission assessment indicates that the nurse should consult with the health care provider before giving the aspirin?

The patient states, "My symptoms started with a terrible headache." rational: A sudden onset headache is typical of a subarachnoid hemorrhage, and aspirin is contraindicated. Atrial fibrillation, dysphasia, and transient ischemic attack (TIA) are not contraindications to aspirin use, so the nurse can administer the aspirin.

When admitting a patient with a possible coup-contracoup injury after a car accident to the emergency department, the nurse obtains the following information. Which finding is most important to report to the health care provider?

The patient takes warfarin (Coumadin) daily. rational: The use of anticoagulants increases the risk for intracranial hemorrhage and should be immediately reported. The other information would not be unusual in a patient with a head injury who had just arrived to the ED.

A patient who is hospitalized with pneumonia is disoriented and confused 2 days after admission. Which information obtained by the nurse about the patient indicates that the patient is experiencing delirium rather than dementia?

The patient was oriented and alert when admitted. rational: The onset of delirium occurs acutely. The degree of disorientation does not differentiate between delirium and dementia. Increasing confusion for several years is consistent with dementia. Fragmented and incoherent speech may occur with either delirium or dementia.

The nurse obtains all of the following information about a 65-year-old patient in the clinic. When developing a plan to decrease stroke risk, which risk factor is most important for the nurse to address?

The patient's blood pressure (BP) is usually about 180/90 mm Hg. rational: Hypertension is the single most important modifiable risk factor and this patient's hypertension is at the stage 2 level. People who drink more than 1 (for women) or 2 (for men) alcoholic beverages a day may increase risk for hypertension. Physical inactivity and obesity contribute to stroke risk but not so much as hypertension.

When assessing a patient with bacterial meningitis, the nurse obtains the following data. Which finding should be reported immediately to the health care provider?

The patient's blood pressure is 86/42 mm Hg. rational: Shock is a serious complication of meningitis, and the patient's low blood pressure indicates the need for interventions such as fluids or vasopressors. Nuchal rigidity and a positive Kernig's sign are expected with bacterial meningitis. The nurse should intervene to lower the temperature, but this is not as life threatening as the hypotension.

A patient has a new prescription for bromocriptine (Parlodel) to control symptoms of Parkinson's disease. Which information obtained by the nurse may indicate a need for a decrease in the dose?

The patient's blood pressure is 90/46 mm Hg. rational: Hypotension is an adverse effect of bromocriptine, and the nurse should check with the health care provider before giving the medication. Diarrhea, cough, and deep vein thrombosis are not associated with bromocriptine use.

A patient who has had a subarachnoid hemorrhage is being cared for in the intensive care unit. Which information about the patient is most important to communicate to the health care provider?

The patient's blood pressure is 90/50 mm Hg. rational: To prevent cerebral vasospasm and maintain cerebral perfusion, blood pressure needs to be maintained at a level higher than 90 mm Hg systolic after a subarachnoid hemorrhage. A low BP or drop in BP indicates a need to administer fluids and/or vasopressors to increase the BP. An ongoing headache, RBCs in the CSF, and a stiff neck are all typical clinical manifestations of a subarachnoid hemorrhage and do not need to be rapidly communicated to the health care provider.

what is related the the diagnosis Guillain-Barre Syndrome

if the client had an influenza immunization in the past week

The charge nurse observes an inexperienced staff nurse who is caring for a patient who has had a craniotomy for a brain tumor. Which action by the inexperienced nurse requires the charge nurse to intervene?

The staff nurse suctions the patient every 2 hours. rational: Suctioning increases intracranial pressure and is done only when the patient's respiratory condition indicates it is needed. The other actions by the staff nurse are appropriate.

A nurse taps the triceps tendon just above the olecranon process in a patient with the elbow flexed and the forearm resting across the patient's chest. Which reactions indicate a normal response? Select all that apply. 1 Continued rhythmic contraction of the muscle 2 Flexion of the arm Correct 3 Extension of the arm Correct 4 Contraction of triceps muscle 5 Relaxation of triceps muscle

The triceps reflex is elicited by striking the triceps tendon above the elbow while the patient's arm is flexed. The normal response is extension of the arm or visible contraction of the triceps. Clonus is an abnormal response characterized by a continued rhythmic contraction of the muscle with continuous application of the stimulus. Flexion of the arm and relaxation of the triceps muscle when eliciting the triceps reflex are abnormal responses and indicate a dysfunctional neurological system. 3,4

A patient has a tonic-clonic seizure while the nurse is in the patient's room. Which action should the nurse take?

Time and observe and record the details of the seizure and postictal state. rational: Because diagnosis and treatment of seizures frequently are based on the description of the seizure, recording the length and details of the seizure is important. Insertion of an oral airway and restraining the patient during the seizure are contraindicated. The nurse may need to move the patient to decrease the risk of injury during the seizure.

When assessing a patient with a possible stroke, the nurse finds that the patient's aphasia started 3.5 hours previously and the blood pressure is 170/92 mm Hg. Which of these orders by the health care provider should the nurse question?

Titrate labetolol (Normodyne) drip to keep BP less than 140/90 mm Hg. rational: Because elevated BP may be a protective response to maintain cerebral perfusion, antihypertensive therapy is recommended only if mean arterial pressure (MAP) is >130 mm Hg or systolic pressure is >220 mm Hg. Fluid intake should be 1500 to 2000 mL daily to maintain cerebral blood flow. The head of the bed should be elevated to at least 30 degrees, unless the patient has symptoms of poor tissue perfusion. tPA may be administered if the patient meets the other criteria for tPA use.

To determine the integrity of the facial nerve (CN VII), the nurse will ask the patient to do which of the following? Select all that apply. 1 Clench teeth 2 Stick out tongue 3 Say "ga, ga, ga" 4 Raise eyebrows 5 Purse lips together 6 Close eyes tightly

To assess the motor function of the fa cial nerve (CN VII), assessments include asking the patient to raise eyebrows, close eyes tightly, purse the lips, perform an exaggerated smile, and frown. The nurse is looking for any asymmetry in facial movements. Clenching teeth while palpating the masseter muscles is assessing the trigeminal nerve (CN V). Extending the tongue assesses the hypoglossal nerve (CN XII). Asking the patient to say "ga, ga, ga" requires movement of the pharynx and tongue, therefore assessing cranial nerve IX, the glossopharyngeal. Text Reference - p. 1347 3,4,5

What suggestion should the nurse give to the family members of a patient with Parkinson's disease to promote self-care and independence of the patient? Select all that apply. 1 Use slip-on shoes. 2 Use rugs on the floor. 3 Use an elevated toilet seat. 4 Use buttons and hooks on clothes. 5 Elevate the legs on an ottoman when sitting.

To promote self-care and independence of the patient, certain changes in the home environment can be made. Slip-on shoes should be used, because they can be easily put on or taken off by the patient. Elevated toilet seats help with getting on and off the toilet easily. Legs can be elevated on an ottoman to prevent ankle edema. Rugs should be removed, because they can cause the patient to fall. Hooks and buttons as clothing fasteners may be difficult for the patient to use; instead, hook-and-loop (Velcro) fasteners or zippers should be used. Text Reference - p. 1437 1,3,5

Which information about a patient who is being treated with carbidopa/levodopa (Sinemet) for Parkinson's disease is most important for the nurse to report to the health care provider?

Uncontrolled head movement rational: Dyskinesia is an adverse effect of the Sinemet, indicating a need for a change in medication or decrease in dose. The other findings are typical with Parkinson's disease.

HESI HINT: What is a common cause of death after spinal cord injury?

Urinary tract infection - Bacteria grow best in alkaline media, so keeping urine dilute and acidic prevents infection - Keeping the bladder empty assists in avoiding bacterial growth in urine that has stagnated in the bladder

The nurse obtains these assessment findings for a patient who has a head injury. Which finding should be reported rapidly to the health care provider?

Urine output of 800 mL in the last hour rational: The high urine output indicates that diabetes insipidus may be developing and interventions to prevent dehydration need to be rapidly implemented. The other data do not indicate a need for any change in therapy.

HESI HINT: What allows nurses to eliminate ambiguous terms (lethargic, stuporous) to describe neurologic status?

Use of the Glasgow Coma Scale

To determine whether a new patient's confusion is caused by dementia or delirium, which action should the nurse take?

Use the Confusion Assessment Method tool to assess the patient. rational: The Confusion Assessment Method tool has been extensively tested in assessing delirium. The other actions will be helpful in determining cognitive function or risk factors for dementia or delirium, but they will not be useful in differentiating between dementia and delirium.

if a patient is being treated for a cerebral aneurysm that is being treated conservatively with bed rest, anticonvulsant, and sedatives, WHAT IS AN IMPORTANT SYMPTOM FOR THE NURSE TO NOTICE?

if the patient has a chronic cough

When can a comatose client on IV hyperalimentation begin to receive tube feedings instead?

When peristalsis resumes as evidenced by active bowel sounds, passage of flatus or bowel movement

Is multiple sclerosis thought to occur bc of an autoimmune process?

Yes

if a patient with Dementia asks repeatedly Where is my mother? the nurse should respond to prevent client frustration and agitation by stating

You miss your mother , What was she like?

A patient with a severe head injury begins to assume a posture of flexed upper extremities with plantar flexed lower extremities. These assessments indicate: a. increasing intracranial pressure with decorticate posturing b. decreasing intracranial pressure with decerebrate posturing c. decreasing intracranial pressure with decorticate posturing d. increasing intracranial pressure with decerebarate posturing

[age 461 a

A Parkinson's pt. is depressed because his drug protocol of L-dopa and Sinemet are no longer controlling his symptoms. Which would be the most helpful information. a. Other drugs can be combined with L-dopa to increase its effectiveness b. The effect of these drugs has an uneven course; symptoms will begin to subside again soon c. The two drugs can be given in higher doses to control the symptoms d. Surgical interventions have been very effective in the control of parinsonian symptoms

a

A patient with generalized convulsive disorder has a nursing diagnosis of Deficient knowledge related to lack of information about side effects of Dilantin. Which goal and outcome criteria would be most appropriate? a. absence of GI complaint: takes medication with food b. stimulate gingiva: brushes teeth vigorously to encourage gingival growth c. maintain normal pattern of elimination: limits fluids and eats foods that reduce diarrhea d. maintain normal sleep pattern: reduces stimuli and takes warm baths to induce drowsiness

a

When teaching the children of a patient who is being evaluated for Alzheimer's disease (AD) about the disorder, the nurse explains that

a diagnosis of AD can be made only when other causes of dementia have been ruled out. rational: The diagnosis of AD is one of exclusion. Age is the most important risk factor for development of AD. Drugs can slow the deterioration but do not dramatically reverse the effects of AD. Brain atrophy is a common finding in AD, but it can occur in other diseases as well and does not confirm an AD diagnosis.

what does a myelogram procedure include ?

a lumbar puncture

15. Priority Decision: The nurse finds a patient in bed having a generalized tonic-clonic seizure. During the seizure activity, what actions should the nurse take (select all that apply)? a. Loosen restrictive clothing. b. Turn the patient to the side. c. Protect the patient's head from injury. d. Place a padded tongue blade between the patient's teeth. e. Restrain the patient's extremities to prevent soft tissue and bone injury.

a, b, c. The focus is on maintaining a patent airway and preventing patient injury. The nurse should not place objects in the patient's mouth or restrain the patient.

autonomic nervous system s/s of seizure

increase in epigastric secretions pallor sweating flushing piloerection pupillary dialtion tachycardia

In order to plan care for a pt. with multiple sclerosis, the nurse must know that the altered mobility is related to: a. weakness and spasticity b. malnutrition and SOB c. painful joints and dizziness d. visual and perceptual problems

a.

The patient with meningitis who has a positive Brudzinski's sign will: a. flex the hips when the neck is flexed by the nurse b. not be able to fully extend the flexed leg due to hamstring pain c. resist efforts of the nurse to flex the neck d. flex the big toe upward and fan out the other toes

a. page 470

When obtaining a health history and physical assessment for a patient with possible multiple sclerosis (MS), the nurse should _____________

inquire about any urinary tract problems. rational: Urinary tract problems with incontinence or retention are common symptoms of MS. Chest pain and skin rashes are not symptoms of MS. A decrease in libido is common with MS.

A pt. falls to the floor in a generalized seizure. Which intervention is most appropriate? a. cradle head to prevent injury b. insert object between teeth to prevent biting tongue c. manually restrain limbs d. keep patient on back to prevent aspiration

a. cradle head to prevent injury

When asked the purpose of Tegretol in the treatment of multiple sclerosis, the nurse's best response is that the drug is used to: a. relieve pain b. reduce spasticity c. reduce fatigue d. alter immune response

a. page 478

A hospitalized patient complains of a moderate bilateral headache that radiates from the base of the skull. Which of these prescribed PRN medications should the nurse administer initially?

acetaminophen (Tylenol) rational: The patient's symptoms are consistent with a tension headache, and initial therapy usually involves a nonopioid analgesic such as acetaminophen, sometimes combined with a sedative or muscle relaxant. Lorazepam may be used in conjunction with acetaminophen but would not be appropriate as the initial monotherapy. Morphine sulfate and butalbital and aspirin would be more appropriate for a headache that did not respond to a nonopioid analgesic.

A patient admitted to the emergency department is diagnosed with botulism, and an order for botulinum antitoxin is received. Before administering the antitoxin, it is most important for the nurse to _______________

administer an intradermal test dose. rational: To prevent allergic reactions, an intradermal test dose of the antitoxin should be administered. Although temperature, allergy history, and symptom assessment and documentation are appropriate, these assessments will not affect the decision to administer the antitoxin.

A patient arrives at an urgent care center with a deep puncture wound after stepping on a nail that was lying on the ground. The patient reports having had a tetanus booster 7 years ago. The nurse will anticipate ______________

administration of the tetanus-diphtheria (Td) booster. rational: If the patient has not been immunized within 5 years, administration of the Td booster is indicated because the wound is deep. Immune globulin administration is given by the IM route if the patient has no previous immunization. Administration of a series of immunization is not indicated. TIG is not indicated for this patient, and a test dose is not needed for immune globulin.

what is most important to ask the patient before going to have a CT scan performed?

allergy to shellfish

A patient is seen in the health clinic with symptoms of a stooped posture, shuffling gait, and pill rolling-type tremor. The nurse will anticipate teaching the patient about _______________

antiparkinsonian drugs. rational: The diagnosis of Parkinson's is made when two of the three characteristic signs of tremor, rigidity, and bradykinesia are present. The confirmation of the diagnosis is made on the basis of improvement when antiparkinsonian drugs are administered. This patient has symptoms of tremor and bradykinesia; the next anticipated step will be treatment with medications. MRI and EEG are not useful in diagnosing Parkinson's disease, and corticosteroid therapy is not used to treat it.

what indicates Crede's maneuver

applying downward hand pressure at the umbilical toward the symphysis pubis

The nurse identifies the nursing diagnosis of impaired verbal communication for a patient with expressive aphasia. An appropriate nursing intervention to help the patient communicate is to _______________

ask simple questions that the patient can answer with "yes" or "no." rational: Communication will be facilitated and less frustrating to the patient when questions that require a "yes" or "no" response are used. When the language areas of the brain are injured, the patient might not be able to read or recite words, which will frustrate the patient without improving communication. Expressive aphasia is caused by damage to the language areas of the brain, not by the areas that control the motor aspects of speech. The nurse should allow time for the patient to respond.

A patient is admitted to the hospital with dysphasia and right-sided weakness that resolves in a few hours. The nurse will anticipate teaching the patient about ____________

aspirin (Ecotrin). rational: Following a TIA, patients typically are started on medications such as aspirin to inhibit platelet function and decrease stroke risk. tPA is used for acute ischemic stroke. Coumadin is usually used for patients with atrial fibrillation. Nimodipine is used to prevent cerebral vasospasm after a subarachnoid hemorrhage.

A long-term care patient with moderate dementia develops increased restlessness and agitation. The nurse's initial action should be to

assess for factors that might be causing discomfort. rational: Increased motor activity in a patient with dementia is frequently the patient's only way of responding to factors like pain, so the nurse's initial action should be to assess the patient for any precipitating factors. Administration of sedative drugs may be indicated, but this should not be done until assessment for precipitating factors has been completed and any of these factors have been addressed. Reorientation is unlikely to be helpful for the patient with moderate dementia. Assigning a nursing assistant to stay with the patient also may be necessary, but any physical changes that may be causing the agitation should be addressed first.

what should be done first before developing a care plan for a patient with Parkinson's

assess the client's ability to perform activities of daily living

The nurse witnesses a patient with a seizure disorder as the patient suddenly jerks the arms and legs, falls to the floor, and regains consciousness immediately. It will be most important for the nurse to _________________

assess the patient for a possible head injury. rational: The patient who has had a myoclonic seizure and fall is at risk for head injury and should be evaluated and treated for this possible complication first. Documentation of the seizure, notification of the seizure, and administration of antiseizure medications also are appropriate actions, but the initial action should be assessment for injury.

To protect a patient from injury during an episode of delirium, the most appropriate action by the nurse is to ______________

assign a nursing assistant to stay with the patient and offer frequent reorientation. rational: The priority goal is to protect the patient from harm, and a staff member will be most experienced in providing safe care. Visits by family members are helpful in reorienting the patient, but families should not be responsible for protecting patients from injury. Antipsychotic medications may be ordered, but only if other measures are not effective because these medications have multiple side effects. Restraints are sometimes used but tend to increase agitation and disorientation.

A patient who has had a stroke has a new order to attempt oral feedings. The nurse should assess the gag reflex and then _____________

assist the patient into a chair. rational: The patient should be as upright as possible before attempting feeding to make swallowing easier and decrease aspiration risk. To assess swallowing ability, the nurse should initially offer water or ice to the patient. Pureed diets are not recommended because the texture is too smooth. The patient may have a poor appetite, but the oral feeding should be attempted regardless.

Myasthenia Gravis

autoimmune or congenital neuromuscular disease that leads to fluctuating muscle weakness and fatigue. muscle weakness is caused by circulating antibodies that block acetylcholine receptors at the postsynaptic neuromuscular junction

A 42-year-old patient who was adopted at birth is diagnosed with early Huntington's disease (HD). When teaching the patient, spouse, and children about this disorder, the nurse will provide information about the ______________

availability of genetic testing to determine the HD risk for the patient's children. rational: Genetic testing is available to determine whether an asymptomatic individual has the HD gene. The patient and family should be informed of the benefits and problems associated with genetic testing. Sinemet will increase symptoms of HD given that HD involves an increase in dopamine. Antibiotic therapy will not reduce the risk for aspiration. There are no effective treatments or lifestyle changes that delay the progression of symptoms in HD.

A patient with amyotrophic lateral sclerosis is uniquely prone to depression because a. nutritional intake is poor b. intellectual capacity is not affected c. mobility is limited d. communication is altered

b

To prevent a postlumbar puncture headache, the patient is asked to lie flat for at least: a. 8 hours b. 6 hours c. 4 hours d. 2 hours

b

Which assessments indicate that mannitol therapy for cerebral edema is effective in a patient with ICP? a. increased blood pressure b. increased urinary output c. decreased pulse d. widening pulse pressure

b

Which of these interventions would be best in supporting nutritional intake for a patient with Parkinson's disease? a. Offer large meals with a variety of finger foods b. Thicken liquids to make them easier to swallow c. Puree all foods and drink through a straw d. Offer a diet high carbohydrate and fat and low in protein

b

2. A patient is diagnosed with cluster headaches. The nurse knows that which characteristics are associated with this type of headache (select all that apply)? a. Family history b. Alcohol is the only dietary trigger c. Abrupt onset lasting 5 to 180 minutes d. Severe, sharp, penetrating head pain e. Bilateral pressure or tightness sensation f. May be accompanied by unilateral ptosis or lacrimation

b, c, d, f. Cluster headaches have only alcohol as a dietary trigger and have an abrupt onset lasting 5 minutes to 3 hours with severe, sharp, penetrating pain. Cluster headaches may be accompanied by unilateral ptosis, lacrimation, rhinitis, facial flushing or pallor and commonly recur several times each day for several weeks, with months or years between clustered attacks. Family history and nausea, vomiting, or irritability may be seen with migraine headaches. Bilateral pressure occurring between migraine headaches and intermittent occurrence over long periods of time are characteristics of tension-type headaches.

A patient with amyotrophic lateral sclerosis will probably report that the initial sign of the disease was: a. frequent falls b. weakened hands c. slurred speech d. difficulty swallowing

b.

Which type of headache is suspected when the headaches are unilateral and throbbing, preceded by a prodrome of photophobia, and associated with a family history of this type of headache? a. Cluster b. Migraine c. Frontal-type d. Tension-type

b. Migraine headaches are frequently unilateral and usually throbbing. They may be preceded by a prodrome and frequently there is a family history. Cluster headaches are also unilateral with severe bone-crushing pain but there is no prodrome or family history. Frontal-type headache is not a functional type of headache. Tension-type headaches are bilateral with constant, squeezing tightness without prodrome or family history.

Which assessment would indicate a need for suctioning a pt. with Gullian Barre who is experiencing impaired breathing patterns because of neuromuscular failure? a. decreased pulse rate, respiration of 20 b. increased pulse rate, adventitious breath sounds c. increased pulse rate, respiration of 16 d. decreased pulse, abdominal breathing

b. page 473

when is the hair clipped on a patient having a craniotomy ?

before surgery in the operating room

postictal phase

is the altered state of consciousness after an epileptic seizure. It usually lasts between 5 and 30 minutes, drowsiness, confusion, nausea, hypertension, headache or migraine and other disorienting symptoms.

what is the most prominent symptom in a patient who is having a TIA?

brief periods of unilateral weakness

discharge instructions for a patient taking Dilantin should include for hygeine measures ?

brushing teeth at least twice daily

what is the best home care instructions for a patient with spinal cord injury that promotes mobility ?

build a wheelchair ramp to the door

A pt. who has a pacemaker might be ineligible for a(n): a. computed tomography(CT) b. electromyography (EMG) c. magnetic resonance imaging (MRI) d. eclectroencephalography (EEG)

c

A pt. with a closed head injury is very lethargic and has a slowly reactive pupil on the left side. After recording findings the nurse's best intervention is to a. elevate the legs slightly b. roll the patient to the right side c. elevate the head 30 degrees d. keep the patient flat, remove the pillow

c

The patient who is stuporous, but reacts by withdrawing from stimuli, is said to be: a. comatose b. lethargic c. semicomatose d. somnolent

c

The neurological finding that would be considered abnormal in an 88 year old patient would be: a. smaller pupils, bilaterally b bilateral slow pupillary response to light c. dizziness and problems with balance d. absence of Achilles' tendon jerk

c page 445

30. To reduce the risk for falls in the patient with Parkinson's disease, what should the nurse teach the patient to do? a. Use an elevated toilet seat. b. Use a walker or cane for support. c. Consciously lift the toes when stepping. d. Rock side to side to initiate leg movements.

c. The shuffling gait of PD causes the patient to be off balance and at risk for falling. Teaching the patient to use a wide stance with the feet apart, to lift the toes when walking, and to look ahead helps to promote a more balanced gait. Use of an elevated toilet seat and rocking from side to side will enable a patient to initiate movement. Canes and walkers are difficult for patients with PD to maneuver and may make the patient more prone to injury.

Which type of seizure occurs in children, is also known as a petit mal seizure, and consists of a staring spell that lasts for a few seconds? a. Atonic b. Simple focal c. Typical absence d. Atypical absence

c. The typical absence seizure is also known as petit mal and the child has staring spells that last for a few seconds. Atonic seizures occur when the patient falls from loss of muscle tone accompanied by brief unconsciousness. Simple focal seizures have focal motor, sensory, or autonomic symptoms related to the area of the brain involved without loss of consciousness. Staring spells in atypical absence seizures last longer than those in typical absence seizures and are accompanied by peculiar behavior during the seizure or confusion after the seizure.

When administering a mental status examination to a patient with delirium, the nurse should _____________

choose a place without distracting environmental stimuli. rational: Because overstimulation by environmental factors can distract the patient from the task of answering the nurse's questions, these stimuli should be avoided. The nurse will not wait to give the examination because action to correct the delirium should occur as soon as possible. Reorienting the patient is not appropriate during the examination. Antianxiety medications may increase the patient's delirium.

which s/s will the nurse most likely observe after a tonic - clonic seizure that has progressed to the postictal phase?

confusion followed by a deep sleep

An older adult patient who is experiencing extreme stress related to an admission to the hospital would be expected to demonstrate a. increased heart rate b. decreased blood pressure c. irregular respiration d. dilation of the ppupils

d

The neural synapse refers to the a. length of time it takes for afferent neurons to carry impulses to the CNS b. length of time it takes for efferent neurons to carry impulses to the motor neurons c. space between the axons and dendrites of a neuron d. space between the axons of one neuron and the dendrites of the next

d

3. What is the most important method of diagnosing functional headaches? a. CT scan b. Electromyography (EMG) c. Cerebral blood flow studies d. Thorough history of the headache

d. The primary way to diagnose and differentiate between headaches is with a careful history of the headaches, requiring assessment of specific details related to the headache. Electromyelography (EMG) may reveal contraction of the neck, scalp, or facial muscles in tensiontype headaches but this is not seen in all patients. CT scans and cerebral angiography are used to rule out organic causes of the headaches.

22. The nurse explains to a patient newly diagnosed with MS that the diagnosis is made primarily by a. spinal x-ray findings. b. T-cell analysis of the blood. c. analysis of cerebrospinal fluid. d. history and clinical manifestations.

d. There is no specific diagnostic test for MS. A diagnosis is made primarily by history and clinical manifestations. Certain diagnostic tests may be used to help establish a diagnosis of MS. Positive findings on MRI include evidence of at least two inflammatory demyelinating lesions in at least two different locations within the central nervous system (CNS). Cerebrospinal fluid (CSF) may have increased immunoglobulin G and the presence of oligoclonal banding. Evoked potential responses are often delayed in persons with MS.

A pt. with Parkinson's is considering taking St. John's wort, an herbal remedy for depression, in addition to Sinemet and L-dopa. The best response is to make the patient aware that : a. depression is reduced by the use of herbal remedies such as St. John's wort. b. doses of St. John's wort and parkinsonian drugs should be taken on alternate days c. St. John's wort must be taken in large doses to reduce depression d. herbal remedies can interfere with the effectiveness of the parkinsonian drugs

d. page472

When the nurse applies a painful stimulus to the nail beds of an unconscious patient, the patient responds with internal rotation, adduction, and flexion of the arms. The nurse documents this as _____________

decorticate posturing. rational: Internal rotation, adduction, and flexion of the arms in an unconscious patient is documented as decorticate posturing. Extension of the arms and legs is decerebrate posturing. Because the flexion is generalized, it does not indicate localization of pain or flexion withdrawal.

After a 25-year-old patient has returned home following rehabilitation for a spinal cord injury, the home care nurse notes that the spouse is performing many of the activities that the patient had been managing during rehabilitation. The most appropriate action by the nurse at this time is to _____________

develop a plan to increase the patient's independence in consultation with the patient and the spouse. rational: The best action by the nurse will be to involve all the parties in developing an optimal plan of care. Because family members who will be assisting with the patient's ongoing care need to feel that their input is important, telling the spouse that the patient can perform activities independently is not the best choice. Reminding the patient about the importance of independence may not change the behaviors of the spouse. Supporting the activities of the spouse will lead to ongoing dependency by the patient.

A patient who is seen in the outpatient clinic complains of restless legs syndrome. Which of the following over-the-counter medications that the patient is taking routinely should the nurse discuss with the patient?

diphenhydramine (Benadryl) rational: Antihistamines can aggravate restless legs syndrome. The other medications will not contribute to the restless legs syndrome.

which transmission based precautions is the nurse supposed to implement with a patient with meningitis?

droplet precautions

why do mood swings occur in a patient who had a stroke?

emotional fluctuations are common for many after experiencing a stroke

how should the nurse position a patient with head injury ?

flat with neck immobilizer in place

what indicates an autonomic nervous system manifestation of a seizure

flushing and increased sweating

to correctly perform eye irrigation which direction should the nurse instill the eye irrigant?

from the nasal corner of the eye toward the temple

A patient with a T1 spinal cord injury is admitted to the intensive care unit. The nurse will teach the patient and family that ______________

full function of the patient's arms will be retained. rational: The patient with a T1 injury can expect to retain full motor and sensory function of the arms. Use of only the shoulders is associated with cervical spine injury. Loss of respiratory function occurs with cervical spine injuries. Bradycardia is associated with injuries above the T6 level.

what is the best method of communication for a patient with expressive aphasia?

have the patient point to the key phrases printed on a clipboard

During recovery from a lumbar puncture, the patient reports a severe headache. What is the most appropriate action for the nurse to take? Correct1 Increasing the patient's fluid intake 2 Checking the patient for urine retention 3 Placing the patient in a side-lying position 4 Placing cool packs over the patient's lumbar puncture site

headache after a lumbar puncture is usually caused by leakage or loss of cerebral spinal fluid (CSF). Increased fluid intake, either IV or oral as tolerated, will help restore CSF volume. The patient should be encouraged to lie flat for 4 to 6 hours. A persistent headache may indicate a CSF leak. Checking for urine retention and placing cool packs over the puncture site are incorrect and inappropriate interventions to treat a severe headache after a lumbar puncture. Text Reference - p. 1352 1

A patient with a neck fracture at the C5 level is admitted to the intensive care unit. During initial assessment of the patient, the nurse recognizes the presence of neurogenic shock on finding ________________

hypotension, bradycardia, and warm extremities. rational: Neurogenic shock is characterized by hypotension, bradycardia, and vasodilation leading to warm skin temperature. Spasticity and hyperactive reflexes do not occur at this stage of spinal cord injury. Lack of movement and sensation indicate spinal cord injury, but not neurogenic shock.

When admitting a patient who has a tumor of the right frontal lobe, the nurse would expect to find __________

judgment changes rational: The frontal lobes control intellectual activities such as judgment. Speech is controlled in the parietal lobe. Weakness and hemiplegia occur on the contralateral side from the tumor. Swallowing is controlled by the brainstem.

which nursing intervention is most important after a lumbar puncture has been performed?

keep client flat for several hours make sure to get a serum blood glucose level

what environmental modifications should the nurse implement in a patient with a seizure?

keep room dark and quiet lower the bed to the lowest position keep the side rails up and padded keep suction equipment available

what discharge instructions should the nurse re-inforce to a patient who has recovered from a herniated intervertebral lumbar disk?

lift with your knees bent and your back straight

A 72-year-old patient is diagnosed with moderate dementia as a result of multiple strokes. During assessment of the patient, the nurse would expect to find _____________

loss of both recent and long-term memory. rational: Loss of both recent and long-term memory is characteristic of moderate dementia. Patients with dementia have frequent nighttime awakening. Dementia is progressive, and the patient's ability to perform tasks would not have periods of improvement. Difficulty eating and swallowing is characteristic of severe dementia.

how does a nurse approach a patient with dementia who insists on carrying a purse at all times?

make sure the client is never without the purse

what is the initial sign of Parkinson's disease?

muscle tremors

somatosensory nervous system s/s of seizure

numbness and tingling changes in taste and speech

A 24-year-old patient is hospitalized with the onset of Guillain-Barré syndrome. During this phase of the patient's illness, the most essential assessment for the nurse to carry out is ______________

observing respiratory rate and effort. rational: The most serious complication of Guillain-Barré syndrome is respiratory failure, and the nurse should monitor respiratory function continuously. The other assessments also will be included in nursing care, but they are not as important as respiratory assessment.

what is important for the nurse to do if a patient with Parkinson's disease has difficulty swallowing his medications?

offer water before giving the capsule

The nurse expects that management of the patient who experiences a brief episode of tinnitus, diplopia, and dysarthria with no residual effects will include ______________

oral administration of low dose aspirin therapy. rational: The patient's symptoms are consistent with transient ischemic attack (TIA), and drugs that inhibit platelet aggregation are prescribed after a TIA to prevent stroke. Continuous heparin infusion is not routinely used after TIA or with acute ischemic stroke. The patient's symptoms are not consistent with a cerebral aneurysm. tPA is used only for acute ischemic stroke, not for TIA.

A patient undergoing a cerebral angiography complains of feeling flushed and tasting salt. the best response of the nurse is to a. assure the pt. these are normal reactions b. prepare to give a prescribed dose of antihistamine c. alert the technician to terminate angiography d. raise the patient to a semi-Fowler's position

page 449 a

A normal finding from Babinski's reflex (elecited by stroking the lateral side of the bottom of the foot and observing movement of toes) is the: a. downward curl of the toes b. big toe bending upward c. spreading out of the toes d. pain in the big toe

page 454 a

what patient response indicates that the patient is exhibiting the normal function of cranial nerve 11 ?

patient can shrug the shoulders

When teaching a patient with myasthenia gravis (MG) about management of the disease, the nurse advises the patient to ______________

perform physically demanding activities in the morning. rational: Muscles are generally strongest in the morning, and activities involving muscle activity should be scheduled then. Plasmapheresis is not routinely scheduled but is used for myasthenia crisis or for situations in which corticosteroid therapy should be discontinued. There is no decrease in sensation with MG, and muscle atrophy does not occur because muscles are used during part of the day.

After having a craniectomy and left anterior fossae incision, a patient has a nursing diagnosis of impaired physical mobility related to decreased level of consciousness and weakness. An appropriate nursing intervention is to ______________

perform range-of-motion (ROM) exercises every 4 hours. rational: ROM exercises will help to prevent the complications of immobility. Patients with anterior craniotomies are positioned with the head elevated. The patient with a craniectomy should not be turned to the operative side. When the patient is weak, clustering nursing activities may lead to more fatigue and weakness.

When caring for a patient who had a C8 spinal cord injury 10 days ago and has a weak cough effort and loose-sounding secretions, the initial intervention by the nurse should be to ____________

place the hands on the epigastric area and push upward when the patient coughs. rational: Since the cough effort is poor, the initial action should be to use assisted coughing techniques to improve the ability to mobilize secretions. Administration of oxygen will improve oxygenation, but the data do not indicate hypoxemia. The use of the spirometer may improve respiratory status, but the patient's ability to take deep breaths is limited by the loss of intercostal muscle function. Suctioning may be needed if the patient is unable to expel secretions by coughing but should not be the nurse's first action.

what nursing goal is most important for a patient with rehabilitation care?

prevention of contractures and joint deformities

When the nurse is developing a rehabilitation plan for a patient with a C6 spinal cord injury, an appropriate patient goal is that the patient will be able to _____________

push a manual wheelchair on flat, smooth surfaces. rational: The patient with a C6 injury will be able to use the hands to push a wheelchair on flat, smooth surfaces. Because flexion of the thumb and fingers is minimal, the patient will not be able to grasp a wheelchair during transfer, drive a car with powered hand controls, or turn independently in bed.

When evaluating a patient with trigeminal neuralgia who has had a glycerol rhizotomy, the nurse will ______________

question the patient about social activities with family and friends. rational: Because withdrawal from social activities is a common manifestation of trigeminal neuralgia, asking about social activities will help in evaluating whether the patient's symptoms have improved. Glycerol rhizotomy does not damage the corneal reflex or motor functions of the trigeminal nerve, so there is no need to use an eye shield, do facial exercises, or take precautions with chewing.

A patient with Bell's palsy refuses to eat while others are present because of embarrassment about drooling. The best response by the nurse to the patient's behavior is to _______________

respect the patient's desire and arrange for privacy at mealtimes. rational: The patient's desire for privacy should be respected to encourage adequate nutrition and reduce patient embarrassment. Liquid supplements will reduce the patient's enjoyment of the taste of food. It would be inappropriate for the nurse to discuss the patient's embarrassment with visitors unless the patient wishes to share this information. Chewing on the unaffected side of the mouth will enhance nutrition and enjoyment of food but will not decrease the drooling.

A patient has a stroke affecting the right hemisphere of the brain. Based on knowledge of the effects of right brain damage, the nurse establishes a nursing diagnosis of _______________

risk for injury related to denial of deficits and impulsiveness. rational: Right-sided brain damage typically causes denial of any deficits and poor impulse control, leading to risk for injury when the patient attempts activities such as transferring from a bed to a chair. Right-sided brain damage causes left hemiplegia. Left-sided brain damage typically causes language deficits. Left-sided brain damage is associated with depression and distress about the disability.

which instruction is important for the nurse to specify in giving alternating sites self injections in a patient with multiple sclerosis?

rotate between the thighs and abdomen

what does the nurse tell the patient if he asks why his hair is being clipped and not shaved?

shaving the head causes microscopic cuts, resulting in a risk for infection .

for a patient with TIA how does the nurse check neurological function ?

shine the penlight from the temple into each pupil

when assisting the patient for the lumbar puncture , which position should the patient be in ?

side lying position with neck flexed into chest

A patient with a left-sided brain stroke suddenly bursts into tears when family members visit. The nurse should _____________

teach the family that emotional outbursts are common after strokes. rational: Patients who have left-sided brain stroke are prone to emotional outbursts, which are not necessarily related to the emotional state of the patient. Depression after a stroke is common, but the suddenness of the patient's outburst suggests that depression is not the major cause of the behavior. The family should stay with the patient. The crying is not within the patient's control and asking the patient to stop will lead to embarrassment.

after a patient has an injury to the T5 vertebrae it is important that the patient understand?

that he will only retain functions that exist above his chest

what assessment finding is most suggestive of a serious head injury ?

the patient has clear fluid draining from the ears

the nurse should call the physician IMMEDIATELY in a patient with head injury when?

the patient is difficult to arouse with stimulation

what assessment finding indicates the patient with head injury has a worsening condition ?

the patient's neck is flexed toward the chest

how does the nurse position the feet on the footboard for a patient ?

the soles are parallel to the board

A patient with a history of several transient ischemic attacks (TIAs) arrives in the emergency department with hemiparesis and dysarthria that started 2 hours previously. The nurse anticipates the need to prepare the patient for __________

tissue plasminogen activator (tPA) infusion. rational: The patient's history and clinical manifestations suggest an acute ischemic stroke and a patient who is seen within 4.5 hours of stroke onset is likely to receive tPA (after screening with a CT scan). Heparin administration in the emergency phase is not indicated. Emergent carotid transluminal angioplasty or endarterectomy is not indicated for the patient who is having an acute ischemic stroke.

The health care provider prescribes clopidogrel (Plavix) for a patient with cerebral atherosclerosis. When teaching about the new medication, the nurse will tell the patient ______________

to call the health care provider if stools are tarry. rational: Plavix inhibits platelet function and increases the risk for gastrointestinal bleeding, so patients should be advised to notify the health care provider about any signs of bleeding. The medication does not lower blood pressure, decrease plaque formation, or dissolve clots.

what is a realistic goal for a patient with Parkinson's ?

to maintain optimal muscle and motor function

When assessing a patient with newly diagnosed trigeminal neuralgia, the nurse will ask the patient about _______________

triggers that lead to facial pain. rational: The major clinical manifestation of trigeminal neuralgia is severe facial pain that is triggered by cutaneous stimulation of the nerve. Ptosis, loss of taste, and facial weakness are not characteristics of trigeminal neuralgia.

When a patient is experiencing a cluster headache, the nurse will plan to assess for _______________

unilateral eyelid swelling. rational: Unilateral eye edema, tearing, and ptosis are characteristic of cluster headaches. Nuchal rigidity suggests meningeal irritation, such as occurs with meningitis. Although nausea and vomiting may occur with migraine headaches, projectile vomiting is more consistent with increases in intracranial pressure (ICP). Unilateral sharp, stabbing pain, rather than throbbing pain, is characteristic of cluster headaches.

when giving a patient who is taking Tegretol discharge instructions what should the nurse instruct the patient to report what symptom ?

unusual bleeding - because this medication can cause liver dysfunction

if a patient with cerebral aneurysm tells the nurse that the pain medication that was given to him 2 hours ago is not effectively relieving the pain what should the nurse do?

use a non drug intervention such as listening to a guided imagery tape

The nurse receives a verbal report that a patient has an occlusion of the left posterior cerebral artery. The nurse will anticipate that the patient may have ___________________

visual deficits. rational: Visual disturbances are expected with posterior cerebral artery occlusion. Aphasia occurs with middle cerebral artery involvement. Cognitive deficits and changes in judgment are more typical of anterior cerebral artery occlusion.

when managing the care of a postoperative patient how does the nurse practice beneficence?

when the nurse advocates for the wishes of the patient

when a patient is injured during a seizure what is the nurses priority documentation on the incident? THAT REDUCES LIABILITY

whether or not the call bell was within reach of patient

when reducing the fever of a patient with meningitis the nurse should ?

wrap the cooling blanket in a light cloth cover to protect the patients skin

The nurse is caring for a patient with Alzheimer's disease. Which pathophysiologic proteins are associated with Alzheimer's disease? Select all that apply. 1 Pick bodies 2 α-Synuclein 3 Prion protein 4 β-Amyloid 5 Tau protein

β-Amyloid and tau proteins are associated with Alzheimer's disease. Development of plaques in the brain tissue is a part of aging; however, in patients with Alzheimer's disease, these plaques are seen in specific parts of the brain. These plaques are made up of clusters of insoluble deposits of a protein called β-amyloid, other proteins, remnants of neurons, nonnerve cells such as microglia, and other cells such as astrocytes. Tau proteins, through the microtubules, provide support to the intracellular structures in the central nervous system. In Alzheimer's disease, the tau protein is altered, which in turn causes the microtubules to twist together in a helical fashion. This twisting of microtubules results in formation of neurofibrillary tangles, a characteristic finding in the neurons of persons with Alzheimer's disease. Prion proteins are related to Creutzfeldt-Jakob disease, which is a rare and fatal brain disorder. Pick bodies are associated with frontotemporal lobar degeneration, which is a clinical syndrome associated with shrinking of the frontal and temporal anterior lobes of the brain. α-Synuclein is a protein associated with dementia with Lewy bodies in the brainstem and cortex. Lewy bodies are abnormal deposits of α-synuclein. Text Reference - p. 1446 4,5


Ensembles d'études connexes

AP Government & Politics Final Exam Incomplete :(((

View Set

Unit 2 - Introduction to Mobile Apps & Pair Programming (2019)

View Set

Chapter3-Legal Concepts of the Insurance Contract

View Set

Pharmacology Neuro & Gastro Drugs

View Set

Human Anatomy - The Endocrine System - Chapter 18 - Brooks

View Set

Word Final Exam Review Chapter 3

View Set